You are on page 1of 68

EMPLOYER RECOGNITION backwages from the time they were illegally

Pangilinan v General Milling Corp dismissed in the aggregate amount of


434 SCRA 159 (July 12, 2004) P15,328,594.04;

Facts: The respondent General Milling Corporation is 2. Ordering respondent corporation to


a domestic corporation engaged in the production pay the said complainants their 13th month pay,
and sale of livestock and poultry.It is, likewise, the holiday pay and service incentive leave pay in the
distributor of dressed chicken to various restaurants aggregate amount of P1,979,148.23;
and establishments nationwide. As such, it employs
hundreds of employees, some on a regular basis
and others on a casual basis, as emergency 3. Ordering respondent corporation to
workers. pay said complainants the amount of P1,730,744.22
by way of attorneys fees, representing ten (10%)
The petitioners were employed by the respondent percentum of the total judgment awards.
on different dates as emergency workers at its
poultry plant in Cainta, Rizal, under separate The case against individual respondent Medardo
temporary/casual contracts of employment for a Quiambao is hereby dismissed.
period of five months. Most of them worked as
chicken dressers, while the others served as packers Issue: Whether or not respondents are regular
or helpers. Upon the expiration of their respective employees of GMC?
contracts, their services were terminated. They later
filed separate complaints for illegal dismissal and Ruling:
non-payment of holiday pay, 13th month pay, No, the petitioners were employees with a fixed
night-shift differential and service incentive leave period, and, as such, were not regular employees.
pay against the respondent before the Arbitration
Branch of the National Labor Relations Commission Article 280 of the Labor Code comprehends three
kinds of employees: (a) regular employees or those
The petitioners alleged that their work as chicken whose work is necessary or desirable to the usual
dressers was necessary and desirable in the usual business of the employer; (b) project employees or
business of the respondent, and added that those whose employment has been fixed for a
although they worked from 10:00 p.m. to 6:00 a.m., specific project or undertaking the completion or
they were not paid night-shift differential. They termination of which has been determined at the
stressed that based on the nature of their work, they time of the engagement of the employee or where
were regular employees of the respondent; hence, the work or services to be performed is seasonal in
could not be dismissed from their employment nature and the employment is for the duration of
unless for just cause and after due notice. In support the season; and, (c) casual employees or those
thereof, the petitioners cited the decision of the who are neither regular nor project employee.
Honorable Labor Arbiter Perlita B. Velasco in NLRC
Case No. NCR-6-2168-86, entitled Estelita Jayme, et A regular employee is one who is engaged
al. vs. General Milling Corporation; and NLRC Case to perform activities which are necessary and
No. NCR-9-3726-86, entitled Marilou Carino, et al. vs. desirable in the usual business or trade of the
General Milling Corporation. They asserted that the employer as against those which are undertaken
respondent GMC terminated their contract of for a specific project or are seasonal. There are two
employment without just cause and due notice. separate instances whereby it can be determined
They further argued that the respondent could not that an employment is regular: (1) if the particular
rely on the nomenclature of their employment as activity performed by the employee is necessary or
temporary or casual. desirable in the usual business or trade of the
employer; and, (2) if the employee has been
On August 18, 1997, Labor Arbiter (LA) Voltaire A. performing the job for at least a year.
Balitaan rendered a decision in favor of the
petitioners declaring that they were regular In the case of St. Theresas School of
employees. Finding that the termination of their Novaliches Foundation vs. NLRC,[43] we held that
employment was not based on any of the just Article 280 of the Labor Code does not proscribe or
causes provided for in the Labor Code, the LA prohibit an employment contract with a fixed
declared that they were allegedly illegally period. We furthered that it does not necessarily
dismissed. The decretal portion of the decision follow that where the duties of the employee
reads: consist of activities usually necessary or desirable in
the usual business of the employer, the parties are
WHEREFORE, judgment is hereby rendered in these forbidden from agreeing on a period of time for the
cases, as follows: performance of such activities. There is thus nothing
essentially contradictory between a definite period
1. Declaring respondent corporation of employment and the nature of the employees
guilty of illegally dismissing complainants, except duties.
Rosalina Basan and Filomena Lanting whose
complaints are hereby dismissed on ground of Indeed, in the leading case of Brent School
prescription, and as a consequence therefor Inc. v. Zamora, we laid down the guideline before a
ordering the said respondent corporation to contract of employment may be held as valid, to
reinstate them to their former positions without loss wit:
of seniority rights and other privileges and with full
Page 1 of 5
[S]tipulations in employment G.R. No. 70705 August 21, 1989
contracts providing for term
employment or fixed period MOISES DE LEON, petitioner, vs. NATIONAL LABOR
employment are valid when the RELATIONS COMMISSION and LA TONDEA INC.,
period were agreed upon knowingly respondents
and voluntarily by the parties without
force, duress or improper pressure, Facts:
being brought to bear upon the Moises de Leon petitioner was employed by private
employee and absent any other respondent La Tonde;a Inc. on December 11,
circumstances vitiating his consent, 1981, at the Maintenance Section of its Engineering
or where it satisfactorily appears that Department in Tondo, Manila. 1 His work consisted
the employer and employee dealt mainly of painting company building and
with each other on more or less equipment, and other odd jobs relating to
equal terms with no moral maintenance. He was paid on a daily basis through
dominance whatever being petty cash vouchers.
exercised by the former over the
latter. In the early part of January, 1983, after a service of
more than one (1) year, petitioner requested from
An examination of the contracts entered respondent company that he be included in the
into by the petitioners showed that their payroll of regular workers, instead of being paid
employment was limited to a fixed period, usually through petty cash vouchers. Private respondent's
five or six months, and did not go beyond such response to this request was to dismiss petitioner
period. from his employment on January 16, 1983. Having
been refused reinstatement despite repeated
demands, petitioner filed a complaint for illegal
The records reveal that the stipulations in the dismissal, reinstatement and payment of
employment contracts were knowingly and backwages before the Office of the Labor Arbiter
voluntarily agreed to by the petitioners without of the then Ministry now Department of Labor and
force, duress or improper pressure, or any Employment.
circumstances that vitiated their consent. Similarly,
nothing therein shows that these contracts were Petitioner alleged that he was dismissed following
used as a subterfuge by the respondent GMC to his request to be treated as a regular employee;
evade the provisions of Articles 279 and 280 of the that his work consisted of painting company
Labor Code. buildings and maintenance chores like cleaning
and operating company equipment, assisting
The petitioners were hired as emergency Emiliano Tanque Jr., a regular maintenance man;
workers and assigned as chicken dressers, packers and that weeks after his dismissal, he was re-hired
and helpers at the Cainta Processing Plant. The by the respondent company indirectly through the
respondent GMC is a domestic corporation Vitas-Magsaysay Village Livelihood Council, a labor
engaged in the production and sale of livestock agency of respondent company, and was made to
and poultry, and is a distributor of dressed chicken. perform the tasks which he used to do. Emiliano
While the petitioners employment as chicken Tanque Jr. corroborated these averments of
dressers is necessary and desirable in the usual petitioner in his affidavit. 2
business of the respondent, they were employed on
a mere temporary basis, since their employment On the other hand, private respondent claimed
was limited to a fixed period. As such, they cannot that petitioner was not a regular employee but only
be said to be regular employees, but are merely a casual worker hired allegedly only to paint a
contractual employees. Consequently, there was certain building in the company premises, and that
no illegal dismissal when the petitioners services his work as a painter terminated upon the
were terminated by reason of the expiration of their completion of the painting job.
contracts.[47] Lack of notice of termination is of no
consequence, because when the contract On April 6, 1984, Labor Arbiter Bienvenido S.
specifies the period of its duration, it terminates on Hernandez rendered a decision 3 finding the
the expiration of such period. A contract for complaint meritorious and the dismissal illegal; and
employment for a definite period terminates by its ordering the respondent company to reinstate
own term at the end of such period.[48] petitioner with full backwages and other benefits.
Labor Arbiter Hernandez ruled that petitioner was
In sum, we rule that the appeal was filed not a mere casual employee as asserted by private
within the ten (10)-day reglementary respondent but a regular employee. He concluded
period. Although the petitioners who mainly worked that the dismissal of petitioner from the service was
as chicken dressers performed work necessary and prompted by his request to be included in the list of
desirable in the usual business of the respondent, regular employees and to be paid through the
they were not regular employees payroll and is, therefore, an attempt to circumvent
therein. Consequently, the termination of their the legal obligations of an employer towards a
employment upon the expiry of their respective regular employee.
contracts was valid.
Labor Arbiter Hernandez found as follows:

EMPLOYER DETERMINATION/ DESIGNATION


Page 2 of 5
After a thorough examination of the records of the least one year of service, whether such service is
case and evaluation of the evidence and versions continuous or broken, shall be considered a regular
of the parties, this Office finds and so holds that the employee with respect to the activity in which he is
dismissal of complainant is illegal. Despite the employed and his employment shall continue while
impressive attempt of respondents to show that the such actually exists.
complainant was hired as casual and for the work
on particular project, that is the repainting of Mama This provision reinforces the Constitutional mandate
Rosa Building, which particular work of painting and to protect the interest of labor. Its language
repainting is not pursuant to the regular business of evidently manifests the intent to safeguard the
the company, according to its theory, we find tenurial interest of the worker who may be denied
differently. Complainant's being hired on casual the rights and benefits due a regular employee by
basis did not dissuade from the cold fact that such virtue of lopsided agreements with the
painting of the building and the painting and economically powerful employer who can
repainting of the equipment and tools and other maneuver to keep an employee on a casual status
things belonging to the company and the odd jobs for as long as convenient. Thus, contrary
assigned to him to be performed when he had no agreements notwithstanding, an employment is
painting and repainting works related to deemed regular when the activities performed by
maintenance as a maintenance man are the employee are usually necessary or desirable in
necessary and desirable to the better operation of the usual business or trade of the employer. Not
the business company. Respondent did not even considered regular are the so-called "project
attempt to deny and refute the corroborating employment" the completion or termination of
statements of Emiliano Tanque Jr., who was which is more or less determinable at the time of
regularly employed by it as a maintenance man employment, such as those employed in
doing same jobs not only of painting and repainting connection with a particular construction project 9
of building, equipment and tools and machineries and seasonal employment which by its nature is
or machines if the company but also other odd jobs only desirable for a limited period of time. However,
in the Engineering and Maintenance Department any employee who has rendered at least one year
that complainant Moises de Leon did perform the of service, whether continuous or intermittent, is
same odd jobs and assignments as were assigned deemed regular with respect to the activity he
to him during the period de Leon was employed for performed and while such activity actually exists.
more than one year continuously by Id respondent
company. We find no reason not to give credit and The primary standard, therefore, of determining a
weight to the affidavit and statement made therein regular employment is the reasonable connection
by Emiliano Tanque Jr. This strongly confirms that between the particular activity performed by the
complainant did the work pertaining to the regular employee in relation to the usual business or trade
business in which the company had been of the employer. The test is whether the former is
organized. Respondent cannot be permitted to usually necessary or desirable in the usual business
circumvent the law on security of tenure by or trade of the employer. The connection can be
considering complainant as a casual worker on determined by considering the nature of the work
daily rate basis and after working for a period that performed and its relation to the scheme of the
has entitled him to be regularized that he would be particular business or trade in its entirety. Also, if the
automatically terminated. ... . employee has been performing the job for at least
Issue: Whether or not de Leon is a regular employee one year, even if the performance is not continuous
of La Tzondena Inc. or merely intermittent, the law deems the repeated
and continuing need for its performance as
Ruling: sufficient evidence of the necessity if not
Yes, de Leon is a regular employee. indispensability of that activity to the business.
The law on the matter is Article 281 of the Labor Hence, the employment is also considered regular,
Code which defines regular and casual but only with respect to such activity and while
employment as follows: such activity exists.

Art. 281. Regular and casual employment. The In the case at bar, the respondent company, which
provisions of a written agreement to the contrary is engaged in the business of manufacture and
notwithstanding and regardless of the oral distillery of wines and liquors, claims that petitioner
agreements of the parties, an employment shall be was contracted on a casual basis specifically to
deemed to be regular where the employee has paint a certain company building and that its
been engaged to perform activities which are completion rendered petitioner's employment
usually necessary or desirable in the usual business terminated. This may have been true at the
or trade of the employer, except where the beginning, and had it been shown that petitioner's
employment has been fixed for a specific project or activity was exclusively limited to painting that
undertaking the completion or termination of which certain building, respondent company's theory of
has been determined at the time of the casual employment would have been worthy of
engagement of the employee or where the work or consideration.
services to be performed is seasonal in nature and
the employment is for the duration of the season. However, during petitioner's period of employment,
the records reveal that the tasks assigned to him
An employment shall be deemed to be casual if it is included not only painting of company buildings,
not covered by the preceding paragraph: equipment and tools but also cleaning and oiling
Provided, That any employee who has rendered at machines, even operating a drilling machine, and
Page 3 of 5
other odd jobs assigned to him when he had no been no need for the regular Maintenance Section
painting job. A regular employee of respondent of respondent company's Engineering Department,
company, Emiliano Tanque Jr., attested in his manned by regular employees like Emiliano Tanque
affidavit that petitioner worked with him as a Jr., whom petitioner often worked with.
maintenance man when there was no painting job.
Furthermore, the petitioner performed his work of
It is noteworthy that, as wisely observed by the painting and maintenance activities during his
Labor Arbiter, the respondent company did not employment in respondent's business which lasted
even attempt to negate the above averments of for more than one year, until early January, 1983
petitioner and his co- employee. Indeed, the when he demanded to be regularized and was
respondent company did not only fail to dispute this subsequently dismissed. Certainly, by this fact alone
vital point, it even went further and confirmed its he is entitled by law to be considered a regular
veracity when it expressly admitted in its comment employee. And considering further that weeks after
that, "The main bulk of work and/or activities his dismissal, petitioner was rehired by the company
assigned to petitioner was painting and other through a labor agency and was returned to his
related activities. Occasionally, he was instructed to post in the Maintenance Section and made to
do other odd things in connection with perform the same activities that he used to do, it
maintenance while he was waiting for materials he cannot be denied that as activities as a regular
would need in his job or when he had finished early painter and maintenance man still exist.
one assigned to him. 10
It is of no moment that petitioner was told when he
The respondent Commission, in reversing the was hired that his employment would only be
findings of the Labor Arbiter reasoned that casual, that he was paid through cash vouchers,
petitioner's job cannot be considered as necessary and that he did not comply with regular
or desirable in the usual business or trade of the employment procedure. Precisely, the law overrides
employer because, "Painting the business or factory such conditions which are prejudicial to the interest
building is not a part of the respondent's of the worker whose weak bargaining position
manufacturing or distilling process of wines and needs the support of the State. That determines
liquors. 11 whether a certain employment is regular or casual
is not the will and word of the employer, to which
The fallacy of the reasoning is readily apparent in the desperate worker often accedes, much less the
view of the admitted fact that petitioner's activities procedure of hiring the employee or the manner of
included not only painting but other maintenance paying his salary. It is the nature of the activities
work as well, a fact which even the respondent performed in relation to the particular business or
Commission, like the private respondent, also trade considering all circumstances, and in some
expressly recognized when it stated in its decision cases the length of time of its performance and its
that, 'Although complainant's (petitioner) work was continued existence.
mainly painting, he was occasionally asked to do
other odd jobs in connection with maintenance Finally, considering its task to give life and spirit to
work. 12 It misleadingly assumed that all the the Constitutional mandate for the protection of
petitioner did during his more than one year of labor, to enforce and uphold our labor laws which
employment was to paint a certain building of the must be interpreted liberally in favor of the worker in
respondent company, whereas it is admitted that case of doubt, the Court cannot understand the
he was given other assignments relating to failure of the respondent Commission to perceive
maintenance work besides painting company the obvious attempt on the part of the respondent
building and equipment. company to evade its obligations to petitioner by
dismissing the latter days after he asked to be
It is self-serving, to say the least, to isolate treated as a regular worker on the flimsy pretext
petitioner's painting job to justify the proposition of that his painting work was suddenly finished only to
casual employment and conveniently disregard the rehire him indirectly weeks after his dismissal and
other maintenance activities of petitioner which assign him to perform the same tasks he used to
were assigned by the respondent company when perform. The devious dismissal is too obvious to
he was not painting. The law demands that the escape notice. The inexplicable disregard of
nature and entirety of the activities performed by established and decisive facts which the
the employee be considered. In the case of Commission itself admitted to be so, in justifying a
petitioner, the painting and maintenance work conclusion adverse to the aggrieved laborer clearly
given him manifest a treatment consistent with a spells a grave abuse of discretion amounting to
maintenance man and not just a painter, for if his lack of jurisdiction.
job was truly only to paint a building there would
have been no basis for giving him other work Private respondent is ordered to reinstate petitioner
assignments In between painting activities. as a regular maintenance man and to pay
petitioner 1) backwages equivalent to three years
It is not tenable to argue that the painting and from January 16,1983, in accordance with the
maintenance work of petitioner are not necessary Aluminum Wage Orders in effect for the period
in respondent's business of manufacturing liquors covered, 2) ECOLA 3) 13th Month Pay, 4) and other
and wines, just as it cannot be said that only those benefits under pertinent Collective Bargaining
who are directly involved in the process of Agreements, if any.
producing wines and liquors may be considered as
necessary employees. Otherwise, there would have
Page 4 of 5
EMPLOYER DETERMINATION/ DESIGNATION period and specific works stated in their respective
[G.R. No. 119523. October 10, 1997] appointments, in addition to their admission that
ISABELO VIOLETA and JOVITO BALTAZAR, petitioners, they are project employees who are subject to the
vs. NATIONAL LABOR RELATIONS COMMISSION, Fifth provisions of Policy Instruction No. 20.
Division, and DASMARINAS INDUSTRIAL AND Labor Arbiter Guardson A. Siao dismissed the claims
STEELWORKS CORPORATIONS, respondents. of petitioners for lack of merit but ordered private
respondent to grant them separation pay. The
Facts: labor arbiter concluded that petitioners are project
Petitioners Isabelo Violeta and Jovito Baltazar were employees based on their admission that they are
former employees of private respondent Dasmarias regular project employees. Thus, their employment
Industrial and Steelworks Corporation (DISC). Their was deemed coterminous with the project for
records of service and employment, insofar as the which their employer engaged them. Their
same are material to this case, are not in dispute. separation was declared valid and their claims for
reinstatement and back wages were denied. The
Petitioner Violeta worked in Construction and award of separation pay was based on the findings
Development Corporation of the Philippines of the labor arbiter that it is the policy of private
(CDCP), a sister corporation of private respondent, respondent to pay employees who have rendered
at its project in CDCP Mines, Basay, Negros Oriental at least one year of continuous service.
from December 15, 1980 up to February 15, 1981. Issue: Whether or not Violeta is a regular employee
Private respondent then hired him as Erector II at of Dasmarinas Industrial and Steel
the formers project for Philphos in Isabel, Leyte on Ruling: Yes,
November 10, 1982 until the termination of the The source of the definition of a regular employee
project on December 3, 1984. On January 21, 1985, vis--vis a project employee is found in Article 280 of
he was reassigned as Erector II for Five Stand TCM the Labor Code which provides:
Project, with vacation and sick leaves, and was Art. 280. Regular and casual employment. - The
designated as a regular project employee at provisions of written agreement to the contrary
private respondents project for National Steel notwithstanding and regardless of the oral
Corporation (NSC) in Iligan City. After receiving a agreement of the parties, an employment shall be
salary adjustment, he was again hired on June 6, deemed to be regular where the employee has
1989 as Handyman for the civil works of a been engaged to perform activities which are
construction project for NSC. On February 10, 1992, usually necessary or desirable in the usual business
he was appointed for project employment, again or trade of the employer, except where the
as Handyman, to NSC ETL #3 Civil Works by private employment has been fixed for a specific project or
respondent. Due to the completion of the particular undertaking the completion or termination of which
item of work he was assigned to, private has been determined at the time of the
respondent terminated the services of petitioner engagement of the employee or where the work or
Violeta on March 15, 1992. service to be performed is seasonal in nature and
Petitioner Baltazar started in the employ of CDCP the employment is for the duration of the season.
on June 23, 1980. He was hired by private An employee shall be deemed to be casual if it is
respondent as Lead Carpenter for project Agua VII not covered by the preceding paragraph:
on October 1, 1981. Like petitioner Violeta, he was Provided, That any employee who has rendered at
transferred from one project to another as a regular least one year of service, whether such service is
project employee. On November 28, 1991, he was continuous or broken, shall be considered a regular
hired as Leadman II in ETL #3 Civil Works by private employee with respect to the activity in which he is
respondent in its project for NSC, but he was employed and his employment shall continue while
separated from such employment on December such activity exists. (Emphases ours).
20, 1991 as a result of the completion of said item of Article 280 was emplaced in our statute books to
work. prevent the circumvention of the employees right
Upon their separation, petitioners executed a to be secure in his tenure by indiscriminately and
quitclaim wherein they declared that they have no completely ruling out all written and oral
claim against private respondent and supposedly agreements inconsistent with the concept of
discharged private respondent from any liability regular employment defined therein. Where an
arising from their employment. employee has been engaged to perform activities
Contending that they are already regular which are usually necessary or desirable in the usual
employees who cannot be dismissed on the ground business of the employer, such employee is
of completion of the particular project where they deemed a regular employee and is entitled to
are engaged, petitioners filed two separate security of tenure notwithstanding the contrary
complaints for illegal dismissal against private provisions of his contract of employment.
respondent, with a prayer for reinstatement and The principal test for determining whether particular
back wages plus damages. employees are properly characterized as project
Private respondent admitted that it is engaged in employees, as distinguished from regular
the development and construction of infrastructure employees, is whether or not the project employees
projects and maintained that Violeta was hired on were assigned to carry out a specific project or
June 6, 1989 to March 15, 1992 as Handyman while undertaking, the duration (and scope) of which
Baltazar was employed on June 6, 1989 to were specified at the time the employees were
December 20, 1991 as Leadman II. It argued that engaged for that project. As defined, project
both are project employees based on their employees are those workers hired (1) for a specific
declaration in their Appointments for Project project or undertaking, and (2) the completion or
Employment that they are employed only for the termination of such project or undertaking has
Page 5 of 5
been determined at the time of engagement of thoroughly read the De Jesus decision, it would
the employee have discovered that the PNCC also raised as a
defense the admission of therein petitioner De Jesus
As Handyman and Erector II, respectively, that he was a project employee, but to no avail.
petitioners services are both necessary and vital to There is another reason why we should rule in favor
the operation of the business of private respondent. of petitioners. Nowhere in the records is there any
This is not at all traversed, but is even confirmed by showing that private respondent reported the
the fact that they were continually and successively completion of its projects and the dismissal of
assigned to the different projects of private petitioners in its finished projects to the nearest
respondent and its sister company, CDCP. Public Employment Office in compliance with
Policy Instruction No. 20 of then Labor Secretary Blas
Following the rule on precedents, we once again F. Ople.
hold that the respective employments of the Jurisprudence abounds with the consistent rule that
present petitioners is not subject to a term but the failure of an employer to report to the nearest
rather to a condition, that is, progress Public Employment Office the termination of its
accomplishment. As we have stated in De Jesus, it workers services every time a project or a phase
cannot be said that their employment had been thereof is completed indicates that said workers are
pre-determined because, firstly, the duration of not project employees.[19] In the case at bar, only
their work is contingent upon the progress the last and final termination of petitioners was
accomplishment and, secondly, the contract gives reported to the aforementioned labor office.
private respondent the liberty to determine the Private respondent should have filed as many
personnel and the number as the work progresses. reports of termination as there were construction
It is ineluctably not definite so as to exempt private projects actually finished if petitioners were indeed
respondent from the strictures and effects of Article project employees, considering that petitioners
280. were hired and again rehired for various projects or
To add our own observation, the appointments of the phases of work therein. Its failure to submit
petitioners herein were not coterminous with NSC reports of termination cannot but sufficiently
ETL #3 Civil Works but with the need for such convince us further that petitioners are truly regular
particular items of work as were assigned to them, employees. Just as important, the fact that
as distinguished from the completion of the project. petitioners had rendered more than one year of
With such ambiguous and obscure words and service at the time of their dismissal overturns
conditions, petitioners employment was not co- private respondents allegations that petitioners
existent with the duration of their particular work were hired for a specific or a fixed undertaking for a
assignments because their employer could, at any limited period of time.[20]
stage of such work, determine whether their Even if we disregard the stints of petitioners with
services were needed or not. Their services could CDCP, it cannot be disclaimed that they have
then be terminated even before the completion of rendered long years of service in private
the phase of work assigned to them. respondents business affairs. Beginning his service in
We find this explication necessary and in accord 1982, petitioner Violeta served in the employ of
with the principle that in controversies between a private respondent up to 1992. In the case of
laborer and his master, doubts reasonably arising petitioner Baltazar, he worked for private
from the evidence, or in the interpretation of respondent from 1981 to 1991. Private respondent
agreements and writings should be resolved in the repeatedly appointed petitioners to new projects
formers favor.[18] after the completion of every project or item of
To be exempted from the presumption of regularity work in which they were previously employed, each
of employment, therefore, the agreement between over a span of about 10 years.
a project employee and his employer must strictly Public respondent contends that the gaps in the
conform with the requirements and conditions employment of petitioners, consisting of the periods
provided in Article 280. It is not enough that an in between the completion of one project and the
employee is hired for a specific project or phase of engagement of petitioners in the next, show that
work. There must also be a determination of or a they could not have been regular employees under
clear agreement on the completion or termination the control of private respondent, and that
of the project at the time the employee is engaged petitioners could have applied for or accepted
if the objective of Article 280 is to be achieved. employment from other employers during those
Since this second requirement was not met in periods. This is puerile and speculative.
petitioners case, they should be considered as In the first place, Article 280 of the Labor Code
regular employees despite their admissions and contemplates both continuous and broken
declarations that they are project employees services. In the second place, there is absolutely no
made under circumstances unclear to us. evidence of petitioners having applied for or
Parenthetically, it is relevant to observe that the accepted such other or outside employment
similarities in the stipulations of the during the brief interregna in the continuity of their
employment/appointment contracts can be work with private respondent. Their undertaking in
explained by the indirect relationship of the the Employment Terms and Conditions of their
Philippine National Construction Corporation service to private respondent bound them to work
(PNCC) and private respondent. CDCP was the in such place of work or project as DISC may assign
predecessor of PNCC which, in turn, is an existing or transfer them, with the further agreement that
sister company of private respondent. Apparently, they would so work during rest day, holidays, night
private respondent ignored the mistake committed time and night shift or during emergencies.[21]
by its said sister company. Also, if only the NLRC had
Page 6 of 5
These are self-evident refutations of private project employee. The position of a helper does not
respondents theory and further bolster petitioners fall within the classification of regular employees.
position that they were not mere employees Hence, complainant never attained regular
engaged for a single or particular project.They employment status. Moreover, his silence for more
were thus removed from the scope of project than three (3) years without any reasonable
employment and considered as regular employees explanation tended to weaken his claim.[8]
since their employment as so-called project Issue: Whether or not de Guzman is a regular
employees was extended long after the employee of San Miguel Corp.?
termination of different projects.[22] Ruling:
The fact that petitioners signed quitclaims will not No, Art. 280 of the Labor Code defines regular,
bar them from pursuing their claims against private project and casual employment as follows:
respondent because quitclaims executed by ART. 280. Regular and Casual Employment. The
laborers are frowned upon as contrary to public provisions of written agreement to the contrary
policy, and are ineffective to bar claims for the full notwithstanding and regardless of the oral
measure of the workers legal rights.[23] The so- agreement of the parties, an employment shall be
called quitclaims signed by petitioners were deemed to be regular where the employee has
actually pro forma provisions printed in the been engaged to perform activities which are
clearance certificate they had to get from private usually necessary or desirable in the usual business
respondent. These were not in the nature of a or trade of the employer, except where the
compromise but a compulsory general release employment has been fixed for a specific project or
required from them, for which no consideration was undertaking the completion or termination of which
either given or even stated. has been determined at the time of the
engagement of the employee or where the work or
EMPLOYER DETERMINATION/ DESIGNATION services to be performed is seasonal in nature and
[G.R. No. 125606. October 7, 1998] the employment is for the duration of the season.
SAN MIGUEL CORPORATION, petitioner, vs. An employment shall be deemed to be casual if it is
NATIONAL LABOR RELATIONS COMMISSION, THIRD not covered by the preceding paragraph:
DIVISION, and FRANCISCO DE GUZMAN, JR., Provided, That, any employee who has rendered at
respondents. least one year of service, whether such service is
continuous or broken, shall be considered a regular
Facts: employee with respect to the activity in which he is
The facts on record show that in November 1990, employed and his employment shall continue while
private respondent was hired by petitioner as such actually exists.
helper/bricklayer for a specific project, the repair The above mentioned provision reinforces the
and upgrading of furnace C at its Manila Glass Constitutional mandate to protect the interest of
Plant. His contract of employment provided that labor as it sets the legal framework for ascertaining
said temporary employment was for a specific ones nature of employment, and distinguishing
period of approximately four (4) months. different kinds of employees. Its language manifests
On April 30, 1991, private respondent was able to the intent to safeguard the tenurial interest of
complete the repair and upgrading of furnace C. worker who may be denied the enjoyment of the
Thus, his services were terminated on that same day rights and benefits due to an employee, regardless
as there was no more work to be done. His of the nature of his employment, by virtue of
employment contract also ended that day. lopsided agreements with the economically
On May 10, 1991, private respondent was again powerful employer who can maneuver to keep an
hired for a specific job or undertaking, which employee on a casual or contractual status for as
involved the draining/cooling down of furnace F long as it is convenient to the employer.
and the emergency repair of furnace E. This project Thus, under Article 280 of the Labor Code, an
was for a specific period of approximately three (3) employment is deemed regular when the activities
months. performed by the employee are usually necessary
After the completion of this task, namely the or desirable in the usual business or trade of the
draining/cooling down of furnace F and the employer even if the parties enter into an
emergency repair of furnace E, at the end of July agreement stating otherwise. But considered not
1991, private respondents services were terminated. regular under said Article are (1) the so-called
On August 1, 1991, complainant saw his name in a project employment the termination of which is
Memorandum posted at the Companys Bulletin more or less determinable at the time of
Board as among those who were considered employment, such as those connected with a
dismissed. particular construction project; and (2) seasonal
On August 12, 1994, or after the lapse of more than employment, which by its nature is only for one
three (3) years from the completion of the last season of the year and the employment is limited
undertaking for which private respondent was for the duration of that season, such as the
hired, private respondent filed a complaint for Christmas holiday season. Nevertheless, an
illegal dismissal against petitioner, docketed as exception to this exception is made: any employee
NLRC NCR Case No. 08-05954-94.[7] who has rendered at least one (1) year of service,
Both parties submitted their respective position whether continuous or intermittent, with respect to
papers, reply and rejoinder to Labor Arbiter Felipe the activity he performed and while such activity
Garduque II. On June 30, 1995, he rendered the actually exists, must be deemed regular.
decision dismissing said complaint for lack of merit. Following Article 280, whether one is employed as a
In his ruling Labor Arbiter Garduque sustained project employee or not would depend on whether
petitioners argument that private respondent was a he was hired to carry out a specific project or
Page 7 of 5
undertaking, the duration and scope of which were were also identifiably separate and distinct from the
specified at the time his services were engaged for usual, ordinary or regular business operations of
that particular project.[11] Another factor that may petitioner, which is glass manufacturing. These
be considered is the reasonable connection undertakings, the duration and scope of which had
between the particular activity undertaken by the been determined and made known to private
employee in relation to the usual trade or business respondent at the time of his employment, clearly
of the employer; if without specifying the duration indicated the nature of his employment as a
and scope, the work to be undertaken is usually project employee. Thus, his services were
necessary or desirable in the usual business or trade terminated legally after the completion of the
of the employer, then it is regular employment and project.[15]
not just project much less casual employment. Public respondent NLRCs decision, if upheld, would
Thus, the nature of ones employment does not amount to negating the distinctions made in Article
depend on the will or word of the employer. Nor on 280 of the Labor Code. It would shunt aside the rule
the procedure of hiring and the manner of that since a project employees work depends on
designating the employee, but on the nature of the the availability of a project, necessarily, the
activities to be performed by the employee, duration of his employment is coterminous with the
considering the employers nature of business[12] project to which he is assigned.[16] It would
and the duration and scope of the work to be become a burden for an employer to retain an
done. employee and pay him his corresponding wages if
Note that the plant where private respondent was there was no project for him to work on. Well to
employed for only seven months is engaged in the remember is the language of the Court in the case
manufacture of glass, an integral component of the of Mamansag v. NLRC:[17]
packaging and manufacturing business of While the Constitution is committed to the policy of
petitioner. The process of manufacturing glass social justice and the protection of the working
requires a furnace, which has a limited operating class, it should not be supposed that every dispute
life. Petitioner resorted to hiring project or fixed term will be automatically decided in favor of labor.
employees in having said furnaces repaired since Management has also rights, which, as such, are
said activity is not regularly performed. Said entitled to respect and enforcement in the interest
furnaces are to be repaired or overhauled only in of fair play. Although the Supreme Court has
case of need and after being used continuously for inclined more often than not toward the worker
a varying period of five (5) to ten (10) years. and has upheld his cause in his conflicts with the
In 1990, one of the furnaces of petitioner required employer, such favoritism has not blinded the Court
repair and upgrading. This was an undertaking to the rule that justice is in every case for the
distinct and separate from petitioners business of deserving, to be dispensed in the light of the
manufacturing glass. For this purpose, petitioner established facts and the applicable law and
must hire workers to undertake the said repair and doctrine.
upgrading. Private respondent was, thus, hired by Considering that private respondent was a project
petitioner on November 28, 1990 on a temporary employee whose employment, the nature of which
status for a specific job for a determined period of he was fully informed, related to a specific project,
approximately four months. work or undertaking, we find that the Labor Arbiter
Upon completion of the undertaking, or on April 30, correctly ruled that said employment legally ended
1991, private respondents services were terminated. upon completion of said project. Hence the
A few days, thereafter, two of petitioners furnaces termination of his employment was not tantamount
required draining/cooling down and emergency to an illegal dismissal; and it was a grave abuse of
repair. Private respondent was again hired on May discretion on public respondent's part to order his
10, 1991 to help in the new undertaking, which reinstatement by petitioner.
would take approximately three (3) months to
accomplish. Upon completion of the second MANAGEMENT PREROGATIVE
undertaking, private respondents services were
likewise terminated.[14] He was not hired a third [G.R. No. 114129. October 24, 1996]
time, and his two engagements taken together did MANILA ELECTRIC COMPANY, petitioner, vs.
not total one full year in order to qualify him as an NATIONAL LABOR RELATIONS COMMISSIONS and
exception to the exception falling under the cited JEREMIAS G. CORTEZ, respondents
proviso in the second paragraph of Art. 280 of the
Labor Code. Facts:
Clearly, private respondent was hired for a specific Private respondent Jeremias C. Cortez, Jr. was
project that was not within the regular business of employed on probationary status by petitioner
the corporation. For petitioner is not engaged in the Manila Electric Company (Meralco) on September
business of repairing furnaces.Although the activity 15, 1975 as a lineman driver. Six months later, he
was necessary to enable petitioner to continue was regularized as a 3rd class lineman-driver
manufacturing glass, the necessity therefor arose assigned at petitioners North Distribution Division. In
only when a particular furnace reached the end of 1977, and until the time of his dismissal, he worked
its life or operating cycle. Or, as in the second as 1st class lineman-driver whose duties and
undertaking, when a particular furnace required an responsibilities among others, includes the
emergency repair. In other words, the undertakings maintenance of Meralcos distribution facilities
where private respondent was hired primarily as (electric lines) by responding to customers
helper/bricklayer have specified goals and complaints of power failure, interruptions, line
purposes which are fulfilled once the designated trippings and other line troubles.
work was completed. Moreover, such undertakings
Page 8 of 5
Characteristics, however, of private respondents a) Serious misconduct or willful disobedience by the
service with petitioner is his perennial suspension employee of the lawful orders of his employers or
from work, viz: representatives in connection with his work.
Date of Memorandum Penalty b) Gross and habitual neglect by the employee of
Meted/Description his duties.
a. May 25, 1977 - Suspension of five (5) xxx xxx xxx.
working days without pay for violation of Company This cause includes gross inefficiency, negligence
Code on Employee Discipline, i.e., drinking of and carelessness. Such just causes is derived from
alcoholic beverages during working time xxx. the right of the employer to select and engage his
b. March 28, 1984 - Suspension of three (3) employees. For indeed, regulation of manpower by
working days without pay for failure or refusal to the company clearly falls within the ambit of
report to J.F. cotton Hospital [where petitioner management prerogative. This court had defined a
maintains a medical clinic] as instructed by a valid exercise of management prerogative as one
company physician, while on sick leave. which covers: hiring work assignment, working
C. June 13, 1984 - Suspension of ten (10) methods, time, place and manner of work, tools to
working days without pay for unauthorized be used, processes to be followed, supervision of
extension of sick leave. workers, working regulations, transfer of employees,
d. June 5, 1987 - Suspension of three (3) working work supervision, lay-off of workers, and the
days without pay for failure or refusal to report to discipline, dismissal and recall of workers. Except as
J.F. Cotton Hospital [where petitioner maintains a provided for, or limited by, special laws, an
medical clinic] as instructed by a company employer is free to regulate, according to his own
physician, while on sick leave. discretion and judgment, all aspects of
employment.5
[Private respondents failed to report for work from Moreover, this Court has upheld a companys
Sept. 18, 1986 to Nov. 10, 1986]. management prerogatives so long as they are
e. December 16, 1988 - Preventive suspension for exercised in good faith for the advancement of the
failure to submit the required Medical Certificate employers interest and not for the purpose of
within 48 hours from the first date of the sick leave. defeating or circumventing the rights of the
employees under special laws or under valid
[Private respondent failed to report for work from agreements.6
Nov. 28, 1988 to the time such Memorandum was In the case at bar, the service record of private
issued on December 16, 1988]. respondent with petitioner is perpetually
f. February 22, 1989 - After formal administrative characterized by unexplained absences and
investigation, suspension of five (5) working days unauthorized sick leave extensions. The nature of his
without pay for unauthorized absences on job i.e. as a lineman-driver requires his physical
November 28, 1988 to December 2, 1988. Absences presence to minister to incessant complaints often
from December 2, 1988. Absences from December faulted with electricity. As aptly stated by the
9-19, 1988 were charged to private respondents Solicitor General:
vacation leave credits for the calendar year 1989. Habitual absenteeism of an errant employee is not
g. May 30, 1989 - Suspension of ten (10) concordant with the public service that petitioner
working days without pay for unauthorized has to assiduously provide. To have delayed power
absences from May 17-19 1989, with warning that failure in a certain district simply because a
penalty of dismissal will be imposed upon MERALCO employee assigned to such area was
commission of similar offense in the future.1 absent and cannot immediately be replaced is a
breach of public service of the highest order. A
Due to his numerous infractions, private respondent deep sense of duty would, therefore, command
was administratively investigated for violation of that private respondent should, at the very least,
Meralcos Code on Employee Discipline, particularly limit his absence for justifiable reasons.7
his repeated and unabated absence from work The penchant of private respondent to continually
without prior notice his superior specifically from incur unauthorized absences and/or a violation of
August 2 to September 19, 1989. petitioners sick leave policy finally rendered his
After such administrative investigation was dismissal as imminently proper.Private respondent
conducted by petitioner, it concluded that private cannot expect compassion from this Court by
respondent was found to have grossly neglected totally disregarding his numerous previous
his duties by not attending to his work as lineman infractions and take into considerations only the
from Aug. 2, 1989 to September 19, 1989 without period covering August 2, 1989 to September 19,
notice to his superiors. 1989. As ruled by this Court in the cases of Mendoza
In a letter dated January 19, 1990, private v. National Labor Relation Commissions,8 and
respondent was notified of the investigation result National Service Corporation v. Leogardo, Jr.,9 it is
and consequent termination of his services the totality, not the compartmentalization, of such
effective January 19, 1990. company infractions that private respondents had
On March 7, 1990, private respondent filed a consistently committed which justified his penalty of
complaint for illegal dismissal against petitioner. dismissal.
Issue: Whether or not private respondents dismissal As correctly observed by the Labor Arbiter:
from the service was illegal? In the case at bar, it was established that
Ruling: complainant violated respondents Code on
NO, Article 283 of the Labor Code enumerates the Employee Discipline, not only once, but ten (10)
just causes for termination. Among such causes are times. On the first occasion, complainant was
the following: simply warned. On the second time, he was
Page 9 of 5
suspended for 5 days. With the hope of reforming or six-hour work-shift a day at five to six days a
the complainant, respondent generously imposed week.
penalties of suspension for his repeated The primary duty of private respondents who were
unauthorized absences and violations of sick leave assigned to PALs Air services Department and
policy which constitute violations of the Code. On ASD/CARGO was to load cargo to departing, and
the ninth time, complainant was already warned unload cargo from arriving PAL international flights
that the penalty of dismissal will be imposed for as well as flights of Cathay Pacific, Northwest
similar or equally serious violation (Annex 10). Airlines and Thai Airlines with which PAL had service
In total disregard of respondents warning, contract[s].
complainant, for the tenth time did not report for On certain occasions, PAL compelled private
work without prior authority from respondent; respondents to work overtime because of urgent
hence, unauthorized. Worse, in total disregard of his necessity. The contracts with private respondents
duties as lineman, he did not report for work from were extended twice, the last of which appears to
August 1, 1989 to September 19, 1989; thus, seriously have been for an indefinite period.
affected (sic) respondents operations as a public On February 3, 1994, private respondent Joselito
utility. This constitute[s] a violation of respondents Pascua, in his and on behalf of other 79 part-time
Code and gross neglect of duty and serious station attendants, filed with the Department of
misconduct under Article 283 of the labor Code.10 Labor and Employment a complaint for:
Habitual absenteeism should not and cannot be (1) Regularization
tolerated by petitioner herein which is a public utility (2) Underpayment of wages
company engaged in the business of distributing (3) Overtime pay
and selling electric energy within its franchise areas (4) Thirteenth month pay
and that the maintenance of Meralcos distribution (5) Service incentive leave pay
facilities (electric lines) by responding to customers (6) Full time of eight hours employment
complaints of power failure, interruptions, line (7) Recovery of benefits due to regular employees
trippings and other line troubles is of paramount (8) Night differential pay
importance to the consuming public. (9) Moral damages and
Hence, an employees habitual absenteeism (10) Attorneys fees,
without leave, which violated company rules and which was docketed as NLRC NCR Case No. 00-02-
regulation is sufficient to justify termination from the 00953-94.
service.11 During the pendency of the case, PAL President
Jose Antonio Garcia and PAL Chairman &
MANAGEMENT PREROGATIVE Corporate Executive Officer Carlos G. Dominguez
[G.R. No. 143258. August 15, 2003] converted the employment status of private
PHILIPPINE AIRLINES, INC., petitioner, vs. JOSELITO respondents from temporary part-time to regular
PASCUA, ROBERT ABION, IRENEO ACOSTA, GARY part-time.
NEPOMUCENO, JASON PALAD, CEFERINO de la On February 24, 1995, private respondents dropped
CRUZ, JOEL SALGADO, WILFREDO RIVERA, their money claim then pending before the Office
ALEXANDER ANORE, FERNANDO BACCAY, of Executive Labor Arbiter Guanio, thus leaving for
EDILBERTO FAUNE, REYMAR KALAW, GARY G. consideration their complaint for regularization -
MARASIGAN, RODOLFO ODO, JONATHAN RENGO, conversion of their employment status from part-
ARTHUR APOSTOL, EDUARDO BALICASAN, MATHIAS time to regular (working on an 8-hour shift).
GLEAN, ALINORMAN HARANGOTE, CRISANTO Finding private respondents remaining cause of
CASTILLO, REX MARION CUERPO, EDGARDO del action was rendered moot and academic by their
PRADO, RICARDO HERNANDEZ, PEDRO MERCADO supervening regularization and denying their prayer
JR., CESAR PAYOYO, RONALDO QUEROL, MAURELIO that their status as regular employees be given
SIERRA, MANUEL VILLELA, LOUISEN FELIPE, retroactive effect to six months after their stint as
LOBENEDICTO TIMBREZA, ANTONIO CABUG, ELISEO temporary contractual employees, the Executive
ESPIRITU, ARNEL BAUTISTA, ANTHONY ROBLES, DENNIS Labor Arbiter dismissed private respondents
ARANDIA, CHARLIE BALUBAL, RHODERIC BITAS, complaint.
ORLANDO CANDA, CHARLIE de la CRUZ, On appeal, the NLRC, finding for private
RIQUESENDO de la FUENTE, RENO DUQUE, respondents, declared them as regular employees
JONATHAN FEBRE, ALVIN RIBERTA, NATHANIEL of PAL with an eight-hour work-shift. The pertinent
MALABAS, JUANITO SERUMA, FREDERICH de ASIS, portions of the NLRC decision reads:
ROMMEL ESTRADA, SYDFREY EVARISTO, ERICSON Respondent admits that complainants have been
INTAL, FERDINAND GALANG, RUBEN PEROLINA, performing functions that are considered necessary
ROBERT McBURNEY, ENRIQUE SORIANO, ALVIN or desirable in the usual business of PAL. There is no
MANALAYSAY, NEMESIO MAALA, RAUL clear showing, however, that complainants
NEPOMUCENO, SAMUEL REYES, ERWIN MINA, employment had been fixed for a particular project
MANUEL REYES, REYNALDO ORAPA, TEODORICO or undertaking the completion or termination of
PADELIO, RANDY PIMENTEL, WILLIAM PATRIMONIO, which has been determined at the time of their
JOEL RAMOS, OLEGARIO REYES, RAUL OCULTO, engagement. Neither is there a clear showing that
ROGELIO OLQUINDO, and LARRY VILLAFLOR. the work or services which they performed, was
respondents. seasonal in nature and their employment for the
duration of the season. Complainants were simply
Facts: hired as part-time employees at the ASD and at the
In April, August, and September of 1992, PAL hired ASD/CARGO to do ramp services.
private respondents as station attendants on a four Complainants can therefore be considered as
casual employees for a definite period during the
Page 10 of 5
first year of their employment and, thereafter, as which he is employed, and his employment shall
regular employees of respondents by operation of continue while such activity actually exists.
law. As such, they should be entitled to the The NLRC decision now assailed is one based on
compensation and other benefits provided in the substantial evidence, which is that amount of
Collective Bargaining Agreement for regular relevant evidence that a reasonable mind might
employees from or day after one year [of] service. accept as adequate to justify a conclusion.[14] It
Having been paid less than what they should bears stressing that findings of fact of quasi-judicial
receive, complainants are therefore, entitled to the agencies like the NLRC which have acquired
differentials.[4] expertise in the specific matters entrusted to their
Petitioner promptly filed a motion for jurisdiction are accorded by this Court not only
reconsideration of the NLRC decision, which was respect but even finality if they are supported by
denied in an order dated October 12, 1998. substantial evidence.[15] Here we find no
Consequently, petitioner filed with the Court of compelling reason to go against the factual
Appeals a special civil action for certiorari to annul findings of the NLRC. The parties had ample
the NLRC decision. On January 26, 2000, the Court opportunity to present below the necessary
of Appeals dismissed the said petition and by evidence and arguments in furtherance of their
resolution issued on May 23, 2000, denied causes, and it is presumed that the quasi-judicial
petitioners motion for reconsideration. body rendered its decision taking into consideration
the evidence and arguments thus presented. Such
Issue: Whether or not the appellate court erred being the case, it is likewise presumed that the
when it upheld the decision of the NLRC to accord official duty of the NLRC to render its decision was
respondents regular full-time employment although regularly performed.[16] Petitioner has not shown
petitioner, in the exercise of its management any compelling justification to warrant reversal of
prerogative, requires only part-time services? the NLRC findings. Absent any showing of patent
error, or that the NLRC failed to consider a fact of
Ruling: No, According to petitioner, NLRC substance that if considered would warrant a
encroached upon this exclusive sphere of different result, we yield to the factual conclusions
managerial decision, when it ruled that of that quasi-judicial agency. More so, when as
respondents should be made regular full-time here, these NLRC conclusions are affirmed by the
employees instead of regular part-time employees, appellate court.
and the appellate court thereby erred in sustaining It is basic to the point of being elementary that
the NLRC. This contention does not quite ring true, nomenclatures assigned to a contract shall be
much less persuade us. It must be borne in mind disregarded if it is apparent that the attendant
that the exercise of management prerogative is not circumstances do not support their use or
absolute. While it may be conceded that designation. The same is true with greater force
management is in the best position to know its concerning contracts of employment, imbued as
operational needs, the exercise of management they are with public interest. Although respondents
prerogative cannot be utilized to circumvent the were initially hired as part-time employees for one
law and public policy on labor and social justice. year, thereafter the over-all circumstances with
That prerogative accorded management could respect to duties assigned to them, number of hours
not defeat the very purpose for which our labor they were permitted to work including over-time,
laws exist: to balance the conflicting interests of and the extension of employment beyond two
labor and management, not to tilt the scale in years can only lead to one conclusion: that they
favor of one over the other, but to guaranty that should be declared full-time employees. Thus, not
labor and management stand on equal footing without sufficient and substantial reasons, the claim
when bargaining in good faith with each other. By of management prerogative by petitioner ought to
its very nature, encompassing as it could be, be struck down for being contrary to law and
management prerogative must be exercised policy, fair play and good faith.
always with the principles of fair play at heart and In sum, we are in agreement with the Court of
justice in mind. Appeals that the NLRC did not commit grave abuse
Records show that respondents were first hired to of discretion simply because it overturned the labor
work for a period of one year. Notwithstanding the arbiters decision. Grave abuse of discretion is
fact that respondents perform duties that are committed when the judgment is rendered in a
usually necessary or desirable in the usual trade or capricious, whimsical, arbitrary or despotic manner.
business of petitioner, respondents were considered An abuse of discretion does not necessarily follow
temporary employees as their engagement was just because there is a reversal by the NLRC of the
fixed for a specific period. However, equally borne decision of the labor arbiter. Neither does variance
by the records, is the fact that respondents in the evidentiary assessment by the NLRC and by
employment was extended for more than two the labor arbiter warrant as a matter of course
years. Evidently, there was a continued and another full review of the facts. The NLRCs decision,
repeated necessity for their services, which puts to so long as it is not bereft of evidentiary support from
naught the contention that respondents, beyond the records, deserves respect from the Court.[17]
the one-year period, still continued to be temporary
part-time employees. Article 280 of the Labor
Code[13] provides that any employee who has G.R. No. 155651. July 28, 2005
rendered at least one year of service, whether such
service is continuous or broken, shall be considered COCA-COLA BOTTLERS PHILIPPINES, INC., SALES
a regular employee with respect to the activity in FORCE UNION-PTGWO-BALAIS, Petitioners, vs.

Page 11 of 5
COCA-COLA BOTTLERS, PHILIPPINES, INC., In these circumstances the CCBPI Executive
Respondents. Committee has decided that the CCBPI is not able
to pay bonuses to any staff in 1999.

FACTS: The Coca-Cola Bottlers Philippines, Inc. Sales After hearing and the submission of evidence and
Force Union-PTGWO is a legitimate labor position papers, the Arbitration Panel composed of
organization duly registered with the Department of Apron Mangabat and Noel Sanchez, as chairman
Labor and Employment, and is the sole and and member, respectively, denied petitioners
exclusive bargaining representative of all regular claim and declared that the P4,000.00 given as ex
route salesmen, regular relief route salesmen, gratia is not a bonus, while Arnel Dolendo, another
regular lead helpers, regular relief lead helpers, member dissented. The dispositive portion of the
regular route helpers, regular relief route helpers decision reads as follows:
and order-taker collectors who are assigned in
various sales offices specified in the parties On 30 May 2001, the Panel denied petitioners
collective bargaining agreement. On the other motion for reconsideration.
hand, the respondent company is a domestic
corporation duly organized and existing under the In dealing with the controversy, the Court of
laws of the Philippines and is engaged in the Appeals adopted a two-tiered approach. First, it
manufacture and distribution of its soft drink held that contrary to the view of the Panel, the
products. 4,000.00 "special ex gratia" payment is a Christmas
bonus, hence, petitioners members are entitled to
In January 1989, the UNION filed a Notice of Strike the additional 50% average commission for the last
with the National Conciliation and Mediation Board six months prior to the grant pursuant to the
raising certain issues for conciliation. As a result of Memorandum of Agreement entered into between
said dispute, the UNION staged a strike. petitioner and respondent Coca-Cola Bottlers
Philippines, Inc. This notwithstanding, the Court of
Subsequently, the Board succeeded in making the Appeals dismissed the petition on the ground that
parties agree to a voluntary settlement of the case petitioners motion for reconsideration dated 12
via a Memorandum of Agreement signed by them March 2001 of the Decision of the Panel that was
on February 9, 1989. Among others, the petitioner originally received on 20 February 2001 was filed out
and the respondent agreed, as follows: of time; hence, the said Decision already became
final and executory after ten (10) calendar days
1. Christmas Bonus- fifty (50%) percent of their from receipt of the copy of the Decision by the
average commission for the last six (6) months. parties pursuant to Article 262-A of the Labor Code.
However, in December 1999, the respondent
granted a fixed amount of P4,000.00 only, ISSUE: W/N the 4,000.00 "special ex gratia"
eliminating thereby the said 50% employees payment is a Christmas bonus, hence, petitioners
average commission for the last six months for members are entitled to the additional 50%
members of the union. Thus, claiming the same as average commission for the last six months prior to
violation of the MOA, the union submitted its the grant pursuant to the Memorandum of
grievance to the respondent. No settlement was Agreement entered into between petitioner and
reached, hence, the case was then referred to a respondent Coca-Cola Bottlers Philippines, Inc.
Panel of Voluntary Arbitrators.
HELD: YES. Rule VII, Section 1 of the "Procedural
The union hereby acknowledges that the granting Guidelines in the Conduct of Voluntary Arbitration
of a Christmas bonus is purely a Management Proceedings" provides the key. Therein, what
prerogative and as such, in determining the constitutes the voluntary arbitrators decision (and,
amount thereof the same is solely a discretion of by extension, that of the Panel of voluntary
Management. The parties however agree that arbitrators) is defined with precision, to wit:
henceforth whenever Management exercises this
prerogative, the same shall include the average Section 1. Decision Award. -- The final arbitral
commission for the last six (6) months prior to the disposition of issue/s submitted to voluntary
grant. arbitration is the Decision. The disposition may take
the form of a dismissal of a claim or grant of specific
Petitioner claimed that the MOA establishes the remedy, either by way of prohibition of particular
companys obligation to pay additionally 50% of acts or specific performance of particular acts. In
the average commission whenever it decides to the latter case the decision is called an Award.
grant a bonus and that the fixed amount of
P4,000.00 granted in December 1999, although In herein case, the Decision of the Panel was in the
denominated as "ex-gratia" was actually a form of a dismissal of petitioners complaint.
Christmas bonus. Naturally, this dismissal was contained in the main
decision and not in the dissenting opinion. Thus,
On the other hand, the respondent company under Section 6, Rule VII of the same guidelines
countered that in 1999 it suffered its worst financial implementing Article 262-A of the Labor Code, this
performance in its history; Four Thousand Pesos Decision, as a matter of course, would become
(P4,000.00) to all its permanent employees, . . final and executory after ten (10) calendar days
from receipt of copies of the decision by the parties
even without receipt of the dissenting opinion
unless, in the meantime, a motion for
Page 12 of 5
reconsideration5 or a petition for review to the
Court of Appeals under Rule 43 of the Rules of On August 18, 1997, Labor Arbiter (LA) Voltaire A.
Court6 is filed within the same 10-day period. As Balitaan rendered a decision in favor of the
correctly pointed out by the Court of Appeals, a petitioners declaring that they were regular
dissenting opinion is not binding on the parties as it employees. Finding that the termination of their
is a mere expression of the individual view of the employment was not based on any of the just
dissenting member from the conclusion held by the causes provided for in the Labor Code, the LA
majority of the Court, following our ruling in Garcia declared that they were allegedly illegally
v. Perez7 as reiterated in National Union of Workers dismissed.
in Hotels, Restaurants and Allied Industries v. NLRC.8
ISSUE: Whether or not the petitioners were regular
As we declared in Nacuray v. National Labor employees of the respondent GMC when their
Relations Commission12 -- employment was terminated.

. . . Nothing is more settled in law than that when a HELD: The SC held the petitioners were employees
judgment becomes final and executory it becomes with a fixed period, and, as such, were not regular
immutable and unalterable. The same may no employees. Article 280 of the Labor Code
longer be modified in any respect, even if the comprehends three kinds of employees: (a) regular
modification is meant to correct what is perceived employees or those whose work is necessary or
to be an erroneous conclusion of fact or law, and desirable to the usual business of the employer; (b)
whether made by the highest court of the land. The project employees or those whose employment has
reason is grounded on the fundamental been fixed for a specific project or undertaking the
considerations of public policy and sound practice completion or termination of which has been
that, at the risk of occasional error, the judgments determined at the time of the engagement of the
or orders of courts must be final at some definite employee or where the work or services to be
date fixed by law. performed is seasonal in nature and the
employment is for the duration of the season; and,
WHEREFORE, premises considered, the Court of (c) casual employees or those who are neither
Appeals Decision dated 22 May 2002 and its regular nor project employees.
Resolution dated 03 October 2002 are hereby
AFFIRMED. No costs. A regular employee is one who is engaged to
perform activities which are necessary and
desirable in the usual business or trade of the
ROSITA PANGILINAN - versus - GENERAL MILLING employer as against those which are undertaken
CORPORATION, for a specific project or are seasonal.[41] There are
two separate instances whereby it can be
FACTS: The respondent General Milling Corporation determined that an employment is regular: (1) if the
is a domestic corporation engaged in the particular activity performed by the employee is
production and sale of livestock and poultry.[2] It is, necessary or desirable in the usual business or trade
likewise, the distributor of dressed chicken to various of the employer; and, (2) if the employee has been
restaurants and establishments nationwide.[3] As performing the job for at least a year. Article 280 of
such, it employs hundreds of employees, some on a the Labor Code does not proscribe or prohibit an
regular basis and others on a casual basis, as employment contract with a fixed period. It does
emergency workers. not necessarily follow that where the duties of the
employee consist of activities usually necessary or
The petitioners[4] were employed by the desirable in the usual business of the employer, the
respondent on different dates as emergency parties are forbidden from agreeing on a period of
workers at its poultry plant in Cainta, Rizal, under time for the performance of such activities. There is
separate temporary/casual contracts of thus nothing essentially contradictory between a
employment for a period of five months.[5] Most of definite period of employment and the nature of
them worked as chicken dressers, while the others the employees duties.
served as packers or helpers.[6] Upon the expiration
of their respective contracts, their services were Stipulations in employment contracts providing for
terminated. They later filed separate complaints for term employment or fixed period employment are
illegal dismissal and non-payment of holiday pay, valid when the period were agreed upon knowingly
13th month pay, night-shift differential and service and voluntarily by the parties without force, duress
incentive leave pay against the respondent before or improper pressure, being brought to bear upon
the Arbitration Branch of the National Labor the employee and absent any other circumstances
Relations Commission. vitiating his consent, or where it satisfactorily
appears that the employer and employee dealt
The petitioners alleged that their work as chicken with each other on more or less equal terms with no
dressers was necessary and desirable in the usual moral dominance whatever being exercised by the
business of the respondent, and added that former over the latter. An examination of the
although they worked from 10:00 p.m. to 6:00 a.m., contracts entered into by the petitioners showed
they were not paid night-shift differential.[8] They that their employment was limited to a fixed period,
stressed that based on the nature of their work, they usually five or six months, and did not go beyond
were regular employees of the respondent; hence, such period. The records reveal that the stipulations
could not be dismissed from their employment in the employment contracts were knowingly and
unless for just cause and after due notice. voluntarily agreed to by the petitioners without
Page 13 of 5
force, duress or improper pressure, or any On November 23, 1993, the Labor Arbiter rendered
circumstances that vitiated their consent. Similarly, a decision, the dispositive portion of which reads:
nothing therein shows that these contracts were
used as a subterfuge by the respondent GMC to WHEREFORE, premises considered, judgment is
evade the provisions of Articles 279 and 280 of the hereby rendered declaring complainants illegally
Labor Code. dismissed, thus respondent is directed to pay
Complainants backwages computed from the time
The petitioners were hired as emergency workers they were illegally dismissed up to the actual
and assigned as chicken dressers, packers and reinstatement but subject to the three year
helpers at the Cainta Processing Plant. While the backwages rule, separation pay for one month for
petitioners employment as chicken dressers is every year of service since reinstatement is
necessary and desirable in the usual business of the evidently not feasible anymore, to pay
respondent, they were employed on a mere complainants 13th month pay, wage differentials
temporary basis, since their employment was and Ten Percent (10%) attorneys fees from the
limited to a fixed period. As such, they cannot be aggregate monetary award. However,
said to be regular employees, but are merely complainant Benedicto Faburada shall only be
contractual employees. Consequently, there was awarded what are due him in proportion to the
no illegal dismissal when the petitioners services nine and a half months that he had served the
were terminated by reason of the expiration of their respondent, he being a part-time employee.
contracts. Lack of notice of termination is of no
consequence, because when the contract All other claims are hereby dismissed for lack of
specifies the period of its duration, it terminates on merit.
the expiration of such period. A contract for
employment for a definite period terminates by its The computation of the foregoing awards is hereto
own term at the end of such period. attached and forms an integral part of this decision.

Petition is denied. That an employer-employee exists between the


parties is shown by the averments of private
respondents in their respective affidavits, carefully
[G.R. No. 121948. October 8, 2001] considered by respondent NLRC in affirming the
Labor Arbiter's decision, thus:
PERPETUAL HELP CREDIT COOPERATIVE, INC.,
petitioner, vs. BENEDICTO FABURADA, SISINITA Benedicto Faburada -Regular part-time Computer
VILLAR, IMELDA TAMAYO, HAROLD CATIPAY, and the programmer/ operator.
NATIONAL LABOR RELATIONS COMMISSION, Fourth Sisinita Vilar -Clerk.
Division, Cebu City, respondents. Imelda C. Tamayo - Clerk.
Harold D. Catipay - Clerk.
On January 3, 1990, Benedicto Faburada, Sisinita All of them were given a memorandum of
Vilar, Imelda Tamayo and Harold Catipay, private termination on January 2, 1990, effective
respondents, filed a complaint against the December 29, 1989.
Perpetual Help Credit Cooperative, Inc. (PHCCI),
petitioner, with the Arbitration Branch, Department Hence, this petition by the PHCCI.
of Labor and Employment (DOLE), Dumaguete City,
for illegal dismissal, premium pay on holidays and ISSUE: whether or not there is an employer-
rest days, separation pay, wage differential, moral employee relationship between the parties and
damages, and attorneys fees. that private respondents were illegally dismissed.

Forthwith, petitioner PHCCI filed a motion to dismiss


the complaint on the ground that there is no Article 280 of the Labor Code provides for three
employer-employee relationship between them as kinds of employees: (1) regular employees or those
private respondents are all members and co- who have been engaged to perform activities
owners of the cooperative. Furthermore, private which are usually necessary or desirable in the usual
respondents have not exhausted the remedies business or trade of the employer; (2) project
provided in the cooperative by-laws. employees or those whose employment has been
fixed for a specific project or undertaking, the
On September 3, 1990, petitioner filed a completion or termination of which has been
supplemental motion to dismiss alleging that Article determined at the time of the engagement of the
121 of R.A. No. 6939, otherwise known as the employee or where the work or service to be
Cooperative Development Authority Law which performed is seasonal in nature and the
took effect on March 26, 1990, requires conciliation employment is for the duration of the season; and
or mediation within the cooperative before a resort (3) casual employees or those who are neither
to judicial proceeding. regular nor project employees.[3] The employees
who are deemed regular are: (a) those who have
On the same date, the Labor Arbiter denied been engaged to perform activities which are
petitioner's motion to dismiss, holding that the case usually necessary or desirable in the usual trade or
is impressed with employer-employee relationship business of the employer; and (b) those casual
and that the law on cooperatives is subservient to employees who have rendered at least one (1)
the Labor Code. year of service, whether such service is continuous
or broken, with respect to the activity in which they
Page 14 of 5
are employed.[4] Undeniably, private respondents dismissed.[7] In this case, only one notice was
were rendering services necessary to the day-to- served upon private respondents by petitioner. It
day operations of petitioner PHCCI. This fact alone was in the form of a Memorandum signed by the
qualified them as regular employees. Manager of the Cooperative dated January 2, 1990
terminating their services effective December 29,
All of them, except Harold D. Catipay, worked with 1989. Clearly, petitioner failed to comply with the
petitioner for more than one (1) year: Benedicto twin requisites of a valid notice.
Faburada, for one and a half (1 1/2) years; Sisinita
Vilar, for two (2) years; and Imelda C. Tamayo, for We hold that private respondents have been
two (2) years and two (2) months. That Benedicto illegally dismissed.
Faburada worked only on a part-time basis, does
not mean that he is not a regular employee. Ones As illegally dismissed employees, private
regularity of employment is not determined by the respondents are therefore entitled to reinstatement
number of hours one works but by the nature and without loss of seniority rights and other privileges
by the length of time one has been in that and to full backwages, inclusive of allowances, plus
particular job.[5] Petitioner's contention that private other benefits or their monetary equivalent
respondents are mere volunteer workers, not computed from the time their compensation was
regular employees, must necessarily fail. Its witheld from them up to the time of their actual
invocation of San Jose City Electric Cooperative vs. reinstatement.[9] Since they were dismissed after
Ministry of Labor and Employment (173 SCRA 697, March 21, 1989, the effectivity date of R.A. 6715[10]
703 (1989 ) is misplaced. The issue in this case is they are granted full backwages, meaning, without
whether or not the employees-members of a deducting from their backwages the earnings
cooperative can organize themselves for purposes derived by them elsewhere during the period of
of collective bargaining, not whether or not the their illegal dismissal.[11] If reinstatement is no longer
members can be employees. Petitioner missed the feasible, as when the relationship between
point. petitioner and private respondents has become
strained, payment of their separation pay in lieu of
As regular employees or workers, private reinstatement is in order.[12]
respondents are entitled to security of tenure. Thus,
their services may be terminated only for a valid
cause, with observance of due process. [G.R. No. 149440. January 28, 2003]

The valid causes are categorized into two groups:


the just causes under Articles 282 of the Labor Code HACIENDA FATIMA and/or PATRICIO VILLEGAS,
and the authorized causes under Articles 283 and ALFONSO VILLEGAS and CRISTINE SEGURA,
284 of the same Code. The just causes are: (1) petitioners, vs. NATIONAL FEDERATION OF
serious misconduct or willful disobedience of lawful SUGARCANE WORKERS-FOOD AND GENERAL TRADE,
orders in connection with the employees work; (2) respondents.
gross or habitual neglect of duties; (3) fraud or willful
breach of trust; (4) commission of a crime or an Although the employers have shown that
offense against the person of the employer or his respondents performed work that was seasonal in
immediate family member or representative; and, nature, they failed to prove that the latter worked
analogous cases. The authorized causes are: (1) the only for the duration of one particular season. In
installation of labor-saving devices; (2) redundancy; fact, petitioners do not deny that these workers
(3) retrenchment to prevent losses; and (4) closing have served them for several years already. Hence,
or cessation of operations of the establishment or they are regular -- not seasonal -- employees.
undertaking, unless the closing is for the purpose of
circumventing the provisions of law. Article 284 FACTS: Contrary to the findings of the Labor Arbiter
provides that an employer would be authorized to that complainants [herein respondents] refused to
terminate the services of an employee found to be work and/or were choosy in the kind of jobs they
suffering from any disease if the employees wanted to perform, the records is replete with
continued employment is prohibited by law or is complainants persistence and dogged
prejudicial to his health or to the health of his fellow determination in going back to work.
employees[6]
Indeed, it would appear that respondents did not
Private respondents were dismissed not for any of look with favor workers having organized
the above causes. They were dismissed because themselves into a union. Thus, when complainant
petitioner considered them to be mere voluntary union was certified as the collective bargaining
workers, being its members, and as such work at its representative in the certification elections,
pleasure. Petitioner thus vehemently insists that their respondents under the pretext that the result was
dismissal is not against the law. on appeal, refused to sit down with the union for
the purpose of entering into a collective bargaining
Procedural due process requires that the employer agreement. Moreover, the workers including
serve the employees to be dismissed two (2) written complainants herein were not given work for more
notices before the termination of their employment than one month. In protest, complainants staged a
is effected: (a) the first, to apprise them of the strike which was however settled upon the signing
particular acts or omissions for which their dismissal of a Memorandum of Agreement which stipulated
is sought and (b) the second, to inform them of the among others that:
decision of the employer that they are being
Page 15 of 5
a) The parties will initially meet for CBA negotiations of fate now find themselves being accused of
on the 11th day of January 1991 and will endeavor refusing to work and being choosy in the kind of
to conclude the same within thirty (30) days. work they have to perform.[5] (Citations omitted)
b) The management will give priority to the women
workers who are members of the union in case work Ruling of the Court of Appeals: The CA affirmed
relative x x x or amount[ing] to gahit and [dipol] that while the work of respondents was seasonal in
arises. nature, they were considered to be merely on
c) Ariston Eruela Jr. will be given back his normal leave during the off-season and were therefore still
work load which is six (6) days in a week. employed by petitioners. Moreover, the workers
d) The management will provide fifteen (15) enjoyed security of tenure. Any infringement upon
wagons for the workers and that existing workforce this right was deemed by the CA to be tantamount
prior to the actual strike will be given priority. to illegal dismissal.
However, in case the said workforce would not be
enough, the management can hire additional ISSUE: Whether or not the respondents, admittedly
workers to supplement them. seasonal workers, were regular employees, contrary
e) The management will not anymore allow the to the clear provisions of Article 280 of the Labor
scabs, numbering about eighteen (18) workers[,] to Code, which categorically state that seasonal
work in the hacienda; and employees are not covered by the definition of
f) The union will immediately lift the picket upon regular employees under paragraph 1, nor covered
signing of this agreement. under paragraph 2 which refers exclusively to
casual employees who have served for at least one
However, alleging that complainants failed to load year.
the fifteen wagons, respondents reneged on its
commitment to sit down and bargain collectively. HELD: Article 280 of the Labor Code, as amended,
Instead, respondent employed all means including states:
the use of private armed guards to prevent the Art. 280. Regular and Casual Employment. - The
organizers from entering the premises. provisions of written agreement to the contrary
notwithstanding and regardless of the oral
Moreover, starting September 1991, respondents agreement of the parties, an employment shall be
did not any more give work assignments to the deemed to be regular where the employee has
complainants forcing the union to stage a strike on been engaged to perform activities which are
January 2, 1992. But due to the conciliation efforts usually necessary or desirable in the usual business
by the DOLE, another Memorandum of Agreement or trade of the employer, except where the
was signed by the complainants and respondents employment has been fixed for a specific project or
which provides: undertaking the completion or termination of which
has been determined at the time of the
Whereas the union staged a strike against engagement of the employee or where the work or
management on January 2, 1992 grounded on the services to be performed is seasonal in nature and
dismissal of the union officials and members; the employment is for the duration of the season.

Whereas parties to the present dispute agree to An employment shall be deemed to be casual if it is
settle the case amicably once and for all not covered by the preceding paragraph:
Provided, That, any employee who has rendered at
The meeting started at 10:00 A.M. A list of least one year of service, whether such service is
employees was submitted by Atty. Tayko based on continuous or broken, shall be considered a regular
who received their 13th month pay. The following employee with respect to the activity in which he is
are deemed not considered employees: employed and his employment shall continue while
1. Luisa Rombo; 2. Ramona Rombo; 3. Bobong such activity exist. (Italics supplied)
Abrega; 4. Boboy Silva
For respondents to be excluded from those
The name Orencio Rombo shall be verified in the classified as regular employees, it is not enough that
1990 payroll. they perform work or services that are seasonal in
nature. They must have also been employed only
The following employees shall be reinstated for the duration of one season. The evidence
immediately upon availability of work: proves the existence of the first, but not of the
second, condition. The fact that respondents -- with
1. Jose Dagle 7. Alejandro Tejares; 2. Rico Dagle 8. the exception of Luisa Rombo, Ramona Rombo,
Gaudioso Rombo; 3. Ricardo Dagle 9. Martin Alas- Bobong Abriga and Boboy Silva -- repeatedly
as Jr.; 4. Jesus Silva 10. Cresensio Abrega; 5. worked as sugarcane workers for petitioners for
Fernando Silva 11. Ariston Eruela Sr.; 6. Ernesto several years is not denied by the latter. Evidently,
Tejares 12. Ariston Eruela Jr. petitioners employed respondents for more than
one season. Therefore, the general rule of regular
When respondents again reneged on its employment is applicable.
commitment, complainants filed the present
complaint. In Abasolo v. National Labor Relations
Commission,[13] the Court issued this clarification:
But for all their persistence, the risk they had to [T]he test of whether or not an employee is a
undergo in conducting a strike in the face of regular employee has been laid down in De Leon v.
overwhelming odds, complainants in an ironic twist NLRC, in which this Court held:
Page 16 of 5
cause.[16] In the case at bar, petitioners failed to
The primary standard, therefore, of determining prove any such cause for the dismissal of
regular employment is the reasonable connection respondents who, as discussed above, are regular
between the particular activity performed by the employees.
employee in relation to the usual trade or business
of the employer. The test is whether the former is Unfair Labor Practice: The NLRC also found herein
usually necessary or desirable in the usual trade or petitioners guilty of unfair labor practice. It ruled as
business of the employer. The connection can be follows: Indeed, from respondents refusal to
determined by considering the nature of the work bargain, to their acts of economic inducements
performed and its relation to the scheme of the resulting in the promotion of those who withdrew
particular business or trade in its entirety. Also if the from the union, the use of armed guards to prevent
employee has been performing the job for at least the organizers to come in, and the dismissal of
a year, even if the performance is not continuous union officials and members, one cannot but
and merely intermittent, the law deems repeated conclude that respondents did not want a union in
and continuing need for its performance as their haciendaa clear interference in the right of
sufficient evidence of the necessity if not the workers to self-organization.[17]
indispensability of that activity to the business.
Hence, the employment is considered regular, but
only with respect to such activity and while such FIRST DIVISION
activity exists. [G.R. No. 157373. July 27, 2004]

xxxxxxxxx PENTAGON INTERNATIONAL SHIPPING, INC.,


x x x [T]he fact that [respondents] do not work petitioner, vs. WILLIAM B. ADELANTAR,
continuously for one whole year but only for the
duration of the x x x season does not detract from FACTS: On August 16, 1997, respondent William B.
considering them in regular employment since in a Adelantar was hired by Dubai Ports Authority of
litany of cases this Court has already settled that Jebel Ali under an employment contract (first
seasonal workers who are called to work from time contract) which provided for an unlimited period of
to time and are temporarily laid off during off- employment with a monthly salary of five thousand
season are not separated from service in said five hundred dirhams (Dhs 5,500).
period, but merely considered on leave until re-
employed.[14] On September 3, 1997, Adelantar and petitioner
Pentagon International Shipping, Inc. (Pentagon),
The CA did not err when it ruled that Mercado v. for and in behalf of Dubai Ports Authority of Jebel
NLRC[15] was not applicable to the case at bar. In Ali, entered into a Philippine Overseas Employment
the earlier case, the workers were required to Administration (POEA) standard employment
perform phases of agricultural work for a definite contract (second contract), this time providing for
period of time, after which their services would be a 12-month period with basic monthly salary of
available to any other farm owner. They were not US$380.00 and fixed overtime pay of US$152.00.
hired regularly and repeatedly for the same
phase/s of agricultural work, but on and off for any Upon completion of his probationary period on April
single phase thereof. On the other hand, herein 5, 1998, Adelantars basic salary was increased to
respondents, having performed the same tasks for five thousand eight hundred ninety dirhams (Dhs
petitioners every season for several years, are 5,890), while his overtime pay was increased to two
considered the latters regular employees for their thousand three hundred fifty-six dirhams (Dhs 2,356)
respective tasks. Petitioners eventual refusal to use effective April 1, 1998.
their services -- even if they were ready, able and
willing to perform their usual duties whenever these On June 11, 1998, however, the management
were available -- and hiring of other workers to barred Adelantar from entering the port due to a
perform the tasks originally assigned to respondents previous dispute with his superior. He was asked to
amounted to illegal dismissal of the latter. hand in his health and employment card. On the
The Court finds no reason to disturb the CAs same date, he received a letter from his employer,
dismissal of what petitioners claim was their valid stating that he was being terminated for assaulting
exercise of a management prerogative. The his superior officer, although he was promised
sudden changes in work assignments reeked of employment in another company.
bad faith. These changes were implemented
immediately after respondents had organized Adelantar was eventually repatriated after nine (9)
themselves into a union and started demanding months and seven (7) days of service. After almost
collective bargaining. Those who were union a year of waiting with no work forthcoming,
members were effectively deprived of their jobs. Adelantar filed a complaint for illegal dismissal with
Petitioners move actually amounted to unjustified money claim against Pentagon International
dismissal of respondents, in violation of the Labor Shipping, Inc. with the NLRC.
Code.
The Labor Arbiter found that the dismissal of
Where there is no showing of clear, valid and legal Adelantar was illegal. Adelantar appealed to the
cause for the termination of employment, the law NLRC arguing that the Labor Arbiter erred in
considers the matter a case of illegal dismissal and granting backwages of only three (3) months and in
the burden is on the employer to prove that the not granting attorneys fees, moral and exemplary
termination was for a valid and authorized damages and reinstatement.
Page 17 of 5
to resort to statutory construction where the rules
The NLRC affirmed the Labor Arbiters decision and and jurisprudence are clear.
held that under Section 10 of R.A. 8042, otherwise
known as the Migrant Workers and Overseas Besides, in Millares v. NLRC,[11] we held that:
Filipinos Act of 1995, an illegally dismissed contract . . . [I]t is clear that seafarers are considered
worker is entitled to the salaries corresponding to contractual employees. They can not be
the unexpired portion of his contract, or for three (3) considered as regular employees under Article 280
months for every year of the unexpired term, of the Labor Code. Their employment is governed
whichever is less. Thus, the NLRC awarded by the contracts they sign every time they are
backwages to Adelantar equivalent to three (3) rehired and their employment is terminated when
months of his basic salary, but exclusive of overtime the contract expires. Their employment is
pay.[4] contractually fixed for a certain period of time. They
fall under the exception of Article 280 whose
Aggrieved, Adelantar filed a petition for certiorari employment has been fixed for a specific project or
with the Court of Appeals. undertaking the completion or termination of which
On September 26, 2002, the Court of Appeals has been determined at the time of engagement
rendered judgment modifying the amounts of the employee or where the work or services to
awarded by the Labor Arbiter and the NLRC. be performed is seasonal in nature and the
employment is for the duration of the season.
ISSUE: whether or NOT the Court of Appeals
properly used as basis Article 279 of the Labor Code Therefore, Adelantar, a seafarer, is not a regular
in its award for backwages to Adelantar. employee as defined in Article 280 of the Labor
Code. Hence, he is not entitled to full backwages
HELD: YES. As early as the case of Coyoca v. and separation pay in lieu of reinstatement as
NLRC,[9] we held that Filipino seamen are provided in Article 279 of the Labor Code. As we
governed by the Rules and Regulations of the held in Millares, Adelantar is a contractual
POEA. The Standard Employment Contract employee whose rights and obligations are
governing the Employment of All Filipino Seamen on governed primarily by Rules and Regulations of the
Board Ocean-Going Vessels of the POEA, POEA and, more importantly, by R.A. 8042, or the
particularly in Part I, Sec. C specifically provides that Migrant Workers and Overseas Filipinos Act of 1995.
the contract of seamen shall be for a fixed period. We find, however, that the Court of Appeals
In no case should the contract of seamen be correctly awarded ten percent (10%) of the
longer than 12 months. It reads: monetary award in Adelantars favor as attorneys
Section C. Duration of Contract. fees, as he was forced to litigate and hence
incurred expenses to protect his rights and
The period of employment shall be for a fixed interest.[12]
period but in no case to exceed 12 months and
shall be stated in the Crew contract. Any extension
of the Contract period shall be subject to the EN BANC [G.R. No. 154472. June 30, 2005]
mutual consent of the parties.
ALEXANDER R. LOPEZ vs. METROPOLITAN
Under the circumstances, the Court of Appeals WATERWORKS AND SEWERAGE SYSTEM,
erred in resolving the issue of backwages based on
the first contract which provided for an unlimited Take not from the mouth of labor the bread it has
period of employment as this violated the explicit earned. The constitutional protection to labor, a
provision of the Rules and Regulations of the POEA. uniform feature of the last three Constitutions
While we recognize that Adelantar executed a including the present one, is outstanding in its
contract with Dubai Ports Authority of Ali Jebel and uniqueness and as a mandate for judicial activism.
might even have applied said contract in his
overseas station, this contract was not sanctioned
by the POEA. We agree with the NLRC when it By virtue of an Agreement,[3] petitioners were
observed thus: engaged by the Metropolitan Waterworks and
Sewerage System (MWSS) as collectors-contractors,
It should be stressed that whatever status of wherein the former agreed to collect from the
employment or increased benefits that the concessionaires of MWSS, charges, fees,
complainant may have gained while under the assessments of rents for water, sewer and/or
employ of Dubai Ports Authority, the undisputed plumbing services which the MWSS bills from time to
fact remains that prior to his deployment, he time.[4]
agreed to be hired under a 12-month POEA
contract, the duration of which is the basis for the In 1997, MWSS entered into a Concession
determination of the extent of the respondents Agreement with Manila Water Service, Inc. and
liability.[10] Benpress-Lyonnaise, wherein the collection of bills
was transferred to said private concessionaires,
The Court of Appeals erred when it adjudged the effectively terminating the contracts of service
first contract as the basis for Pentagons liability between petitioners and MWSS. Regular employees
instead of the second contract, which is in of the MWSS, except those who had retired or
conformity with the POEAs Standard Employment opted to remain with the latter, were absorbed by
Contract. As such, there would have been no need the concessionaires. Regular employees of the
MWSS were paid their retirement benefits, but not
Page 18 of 5
petitioners. Instead, they were refused said benefits, service[54] and falls under the jurisdiction of the
MWSS relying on a resolution[5] of the Civil Service Civil Service Commission.[55]
Commission (CSC) that contract-collectors of the CSC Memorandum Circular No. 38, Series of 1993,
MWSS are not its employees and therefore not categorically made the distinction between
entitled to the benefits due regular government contract of services/job orders and contractual
employees. and plantilla appointment, declaring that services
rendered under contracts of services and job orders
Petitioners filed a complaint with the CSC. In its are non-government services which do not have to
Resolution dated 1 July 1999,[6] the CSC denied be submitted to the CSC for approval. This was
their claims, stating that petitioners were engaged followed by CSC Memorandum Circular No. 4,
by MWSS through a contract of service, which Series of 1994, which allowed the crediting of
explicitly provides that a bill collector-contractor is services for purposes of retirement only for such
not an MWSS employee.[7] Relying on Part V of services supported by duly approved
CSC Memorandum Circular No. 38, Series of 1993, appointments. Subsequently, the CSC issued other
the CSC stated that contract services/job orders resolutions applying the above-mentioned circulars,
are not considered government services, which do stating that while some functions may have been
not have to be submitted to the CSC for approval, contracted out by a government agency, the
unlike contractual and plantilla appointments.[8] persons contracted are not entitled to the benefits
Moreover, it found that petitioners were unable to due to regular government employees.[56]
show that they have contractual appointments
duly attested by the CSC.[9] In addition, the CSC For purposes of determining the existence of
stated that petitioners, not being permanent employer-employee relationship, the Court has
employees of MWSS and not included in the list consistently adhered to the four-fold test, namely:
.submitted to the concessionaire, are not entitled to (1) whether the alleged employer has the power of
severance pay.[10] Petitioners claims for retirement selection and engagement of an employee; (2)
benefits and terminal leave pay were likewise whether he has control of the employee with
denied. respect to the means and methods by which work
is to be accomplished; (3) whether he has the
Petitioners sought reconsideration of the CSC power to dismiss; and (4) whether the employee
Resolution, which was however denied by the CSC. was paid wages.[57] Of the four, the control test is
the most important element.
Aggrieved, petitioners filed a petition for review
under Rule 43 of the Rules of Court with the Court of A review of the circumstances surrounding the case
Appeals. reveals that petitioners are employees of MWSS.
Despite the obvious attempt of MWSS to categorize
Affirming and generally reiterating the ruling of the petitioners as mere service providers, not
CSC, the Court of Appeals held that the Agreement employees, by entering into contracts for services,
entered into by petitioners and MWSS was clear its actuations show that they are its employees,
and unambiguous, and should be read and pure and simple. MWSS wielded its power of
interpreted according to its literal sense.[16] Hence, selection when it contracted with the individual
as per the terms of the agreement, petitioners were petitioners, undertaking separate contracts or
not MWSS employees. agreements. The same goes true for the power to
dismiss. Although termed as causes for termination
ISSUE: Whether or not the CSC erred in finding that of the Agreement, a review of the same shows that
petitioners are not contractual employees of the the grounds indicated therein can similarly be
government and, hence, are not entitled to grounds for termination of employment.
retirement and separation benefits?
In addition, the control test merely calls for the
HELD: YES. The Court has invariably affirmed that it existence of the right to control, and not the
will not hesitate to tilt the scales of justice to the exercise thereof. It is not essential for the employer
labor class for no less than the Constitution dictates to actually supervise the performance of duties of
that the State . . . shall protect the rights of workers the employee, it is enough that the former has a
and promote their welfare.[50] It is committed to right to wield the power.[68] While petitioners were
this policy and has always been quick to rise to contract-collectors of MWSS, they were under the
defense in the rights of labor, as in this case.[51] latters direction as to where and how to perform
their collection and were even subject to
Protection to labor, it has been said, extends to all disciplinary measures. Trainings were in fact
of laborlocal and overseas, organized and conducted to ensure that petitioners are
unorganized, in the public and private sectors.[52] conversant of the procedures of the MWSS.
Besides, there is no reason not to apply this principle
in favor of workers in the government. The Contrary to MWSS assertion that petitioners were
government, including government-owned and free to adopt (their) own method/strategy in the
controlled corporations, as employers, should set matter of collection,[69] the Agreement clearly
the example in upholding the rights and interests of provided that the procedure and/or manner of the
the working class. collection of bills to be followed shall be in
accordance with the provisions of the Manual of
The MWSS is a government owned and controlled Procedures.
corporation with its own charter, Republic Act No. Second, the work of the private respondents was
6234.[53] As such, it is covered by the civil directly related to the principal business or
Page 19 of 5
operation of the petitioner. Being in the business of were under the direct control and supervision of the
providing water to the consumers in the East Zone, concessionaire, much like the arrangement
the collection of the charges therefor by private between herein petitioners and MWSS. Third, they
respondents for the petitioner can only be performed the same vital function of collection in
categorized as clearly related to, and in the pursuit both cases. Fourth, they worked exclusively for their
of the latters business. employers. Hence, the bill collectors in the Manila
Water case were declared employees of Manila
Lastly, ACGI did not carry on an independent Water despite the existence of a sham labor
business or undertake the performance of its service contractor. In the present case, petitioners were
contract according to its own manner and method, directly and individually hired by MWSS, the latter
free from the control and supervision of its principal, not resoting to the intermediary labor contractor
petitioner. Prior to private respondents alleged artifice, but a mere a scrap of paper impudently
employment with ACGI, they were already working declaring the bill collectors to be not employees of
for petitioner, subject to its rules and regulations in MWSS. With greater reason, therefore, should the
regard to the manner and method of performing actuality of the employer-employee relationship
their tasks. This form of control and supervision never between MWSS and petitioners be recognized.
changed although they were already under the
seeming employ of ACGI. These are indications that Petitioners are indeed regular employees of the
ACGI was not left alone in the supervision and MWSS. The primary standard of determining regular
control of its alleged employees. Consequently, it employment is the reasonable connection
can be concluded that ACGI was not an between the particular activity performed by the
independent contractor since it did not carry a employee in relation to the usual business or trade
distinct business free from the control and of the employer. The connection can be
supervision of petitioner.[79] determined by considering the nature of the work
performed and its relation to the scheme of the
Even under the four-fold test, the bill collectors particular business or trade in its entirety. Likewise,
proved to be employees of Manila Water. Thus, the the repeated and continuing need for the
Court held that: performance of the job has been deemed
sufficient evidence of the necessity, if not
Even the four-fold test will show that petitioner is the indispensability of the activity to the business.[84]
employer of private respondents. The elements to Some of the petitioners had rendered more than
determine the existence of an employment two decades of service to the MWSS. The
relationship are: (a) the selection and engagement continuous and repeated rehiring of these bill
of the employee; (b) the payment of wages; (c) collectors indicate the necessity and desirability of
the power of dismissal; and (d) the employers their services, as well as the importance of the role
power to control the employees conduct. The most of bill collectors in the MWSS.
important element is the employers control of the
employees conduct, not only as to the result of the PHIL. FEDERATION OF CREDIT COOPERATIVES, INC.
work to be done, but also as to the means and (PECCI) and FR. BENEDICTO JAYOMA, petitioners, vs.
methods to accomplish it. NATIONAL LABOR RELATIONS COMMISSION (First
Division) and VICTORIA ABRIL, respondents.
We agree with the Labor Arbiter that in the three
stages of private respondents services with the FACTS: Victoria Abril was employed by PFCCI in
petitioner, i.e., (1) from August 1, 1997 to August 31, different capacities from 1982 to 1988, when she
1997; (2) from September 1, 1997 to November 30, went on leave until she gave birth. When she went
1997; and (3) from December 1, 1997 to February 8, back in 1989, after 8 months, another employee
1999, the latter exercised control and supervision had been permanently appointed to her former
over the formers conduct. position of office secretary. She accepted a
position of Regional Field Officer. The contract
In fine, the Court found that the so-called reads:
independent contractor did not have substantial "That the employer hires the employee on
capitalization or investment in the form of tools, contractual basis to the position of Regional Field
equipment, machineries, work premises and other Officer of Region 4 under FCCI/WOCCU/Aid Project
material to qualify as an independent contractor. No. 8175 and to do the function as stipulated in the
Moreover, respondents therein reported daily to the job description assigned to him (her): on
Manila Water branch office and dealt with the probationary status effective February 17/90 for a
consumers through receipts and I.D.s. issued by the period not to exceed six (6) months from said
latter. Likewise, their work was directly related to effectivity, subject to renewal of this contract
and in the pursuit of Manila Waters principal should the employee's performance be
business. More importantly, the Court noted that satisfactory."
ACGI did not carry a distinct business free from the - Said period having elapsed, respondent was
control and supervision of Manila Water. allowed to work until PFCCI presented to her
another employment contract for a period of one
The similarity between this case and the instant year commencing on January 2, 1991 until
petition cannot be denied. For one, the December 31, 1991, after which period, her
respondents in said case are petitioners in this employment was terminated.
case.[81] Second, the work set-up was essentially - LA dismissed her complaint for illegal dismissal
the same. While the bill collectors were individually against PFCCI.
hired, or eventually engaged through ACGI, they
Page 20 of 5
- NLRC set aside LAs decision and ordered her the deduction of their earnings from other sources
reinstated to her last position held (RFO) or to an during the pendency of the appeal.
equivalent position, with full backwages from Jan 1,
1992 until she is reinstated. Mar 22 1995: filed in this court petition for certiorari.
The Supreme Court however sent this case to the
ISSUE: Whether or not Abril was a probationary Court of Appeals (CA) pursuant to St Martins
employee. Funeral vs NLRC. In this case, the Court of Appeals
approves NLRCs decision and came up with the
RULING: No. Abril is a regular employee. decision that respondents were illegally dismissed.
It is an elementary rule in the law on labor relations The petitioners a filed motion for reconsideration
that a probationary employee who is engaged to however it was denied.
work beyond the probationary period of six months,
as provided under Art. 281 of the Labor Code, as Hence this petition, Petitioners contend: Appellate
amended, or for any length of time set forth by the Court committed a serious error when it unilaterally
employer, shall be considered a regular employee. extended the 6-month probationary employment
contracts of the respondents by awarding them full
Article 281 of the Labor Code, as amended, allows backwages, or in lieu of their reinstatement, when it
the employer to secure the services of an ordered payment of their separation pay
employee on a probationary basis which allows computed from the time of their dismissal up to the
him to terminate the latter for just cause or upon finality of its Decision
failure to qualify in accordance with reasonable
standards set forth by the employer at the time of ISSUE: Whether or not probationary employees are
his engagement. As defined in the case of entitled to security of tenure.
International Catholic Migration v. NLRC, a
probationary employee is one who is on trial by an RULING: Yes, probationary employees do not enjoy
employer during which the employer determines permanent status. However, they are entitled to the
whether or not he is qualified for permanent constitutional protection of security of
employment. A probationary employment is made tenure. Under the law, Probationary employees
to afford the employer an opportunity to observe employment may only be terminated for just cause
the fitness of a probationer while at work, and to or when they fail to qualify as regular employees in
ascertain whether he will become a proper and accordance with reasonable standards made
efficient employee. known to them by their employer at the time of
engagement, and after due process.

CEBU MARINE BEACH RESORT, OFELIA PELAEZ AND Instead of reinstatement, they were awarded
TSUYOSHI SASAKI, petitioners, vs. NATIONAL LABOR separation pay equivalent to at least one month
RELATIONS COMMISSION (FOURTH DIVISION), RIC pay, or one month pay for every year of service,
RODRIGO RODRIGUEZ, MANULITO VILLEGAS and whichever is higher; and their full backwages, other
LORNA G. IGOT, respondents. privileges and benefits, or their monetary equivalent
FACTS: Cebu Marine Beach Resort owned by Victor during the period of their dismissal up to their
Dualan, started operations with the recruitment of supposed actual reinstatement
its employees, including Ric Rodrigo Rodriguez,
Manulita Villegas and Lorna G. Igot. Since it wanted
to cater to Japanese guests, employees in the ORIENT EXPRESS PLACEMENT PHILIPPINES, PETITIONER,
VS. NATIONAL LABOR RELATIONS COMMISSION,
resort had to undergo Japanese customs training
PHILIPPINE OVERSEAS EMPLOYMENT
under Tsuyoshi Sasaki, also a petitioner ADMINISTRATION AND ANTONIO F. FLORES,
May 24 1990: Sasaki threw brooms, floor maps, iron RESPONDENTS.
trays, fire hoses and other things to the respondents
while scolding them. Sasaki also screamed to go FACTS: ANTONIO F. FLORES was hired as crane
home and never come back respondents filed at operator by ORIENT EXPRESS PLACEMENT
Regional Arbitration Branch at Cebu City a PHILIPPINES for 1 year, subject to a 3-month
complaint for illegal dismissal and other monetary probationary period. However, after 1 month and 5
claims. days in Saudi Arabia, Flores was repatriated to the
Thereafter, Labor Arbiter dismissed the case but Philippines. Consequently, he filed a complaint with
ordered them to go back to work the POEA for having been terminated from work for
On appeal, NLRC reversed Labor Arbiters decision no valid reason. ORIENT EXPRESS and NADRICO
which ruled that Respondents were illegally countered that Flores was terminated for poor job
dismissed performance.
And Ordered full back wages om May 24, 1990 up
to their actual reinstatement, separation pay and On 7/14/1992, POEA rendered a decision in
attorney fees. favor of complainant. It was observed that neither
ORIENT EXPRESS nor NADRICO pointed out the
On February 28, 1995: the NLRC issued a Resolution reasonable standards of work required of Flores by
declaring that the back wages shall correspond which his incompetency was adjudged; much less
only to the period from May 24, 1990 (the date of did they specify how the latter failed to live up to
their dismissal) until March 23, 1993 (when they were such reasonable standards. Hence, his dismissal was
ordered reinstated by the Labor Arbiter), subject to unwarranted.

Page 21 of 5
On appeal, NLRC affirmed the POEA A PRIME SECURITY SERVICES, INC., petitioner, vs.
decision. In addition, it ruled that the designation of NATIONAL LABOR RELATIONS COMMISSION
Flores as floorman instead of crane operator for (SECOND DIVISION), HON. ARBITER VALENTIN
which he was hired violated his employment GUANIO, and OTHELLO MORENO, respondents.
contract. The NLRC concluded that since Flores FACTS: Private respondent Othello Moreno worked
never worked as crane operator, his foreign as a security guard for a year with Sugarland
employer could not have observed and assessed Security Services Inc, a sister company of petitioner.
his performance as such and then come up with a He was rehired as a security guard on January 30,
performance evaluation sheet, especially 1988 by the petitioner and assigned to the same
considering his consistent claim that he was made post at the U.S. Embassy Building; that he was
to work as floorman instead. Subsequent motion for among those absorbed by the petitioner when it
reconsideration filed by ORIENT EXPRESS and took over the security contracts of its sister
NADRICO was denied. Hence, this petition. company with the U.S. Embassy.
Private respondent Othello Moreno filed a
ISSUE: Whether or not the dismissal of FLORES is valid complaint against petitioner A Prime Security for
illegal dismissal, illegal deduction and
RULING: Under Art. 281 of the Labor Code, the underpayment of wages. Moreno alleged that he
services of an employee hired on a probationary was forced by petitioner to sign new probationary
basis may be terminated when he fails to qualify as contracts of employment for 6 months and further
a regular employee in accordance with reasonable alleged that on August 1, 1988, his employment was
standards made known by the employer to the terminated. Moreno also claimed that during his
employee at the time of his engagement. However, employment, the amount of P20.00 per month was
the Court cannot sustain his dismissal on this ground deducted from his salary allegedly for withholding
because petitioner failed to specify the reasonable tax, and the salary he was receiving was only
standards by which private respondent's alleged P2,187.00 a month, which was way below the
poor performance was evaluated, much less to P2,410.17 stipulated in the PADPAO memorandum
prove that such standards were made known to him of agreement.
at the time of his recruitment in Manila. Both private
respondent's Agency-Worker Agreement with On the other hand, petitioner, A Prime Security,
ORIENT EXPRESS and NADRICO never mentioned alleged that the private respondent was hired on
that: January 30, 1988, on a probationary basis, and he
He must first take and pass a Crane signed an authority to deduct from his salary any
Operators' License Examination in Saudi reimbursement for any loss or damage caused to
Arabia before he would be allowed to even properties of the client; that he was given a copy of
touch a crane petitioners rules and regulations which provide that
He would be assigned as floorman pending sleeping on post is punishable by warning,
release of the results of the examination or in suspension and dismissal and he was caught
the event that he failed; sleeping on post on March 17, 1988, for which he
He would be subjected to a performance was sent a memorandum giving him a last warning;
evaluation by his superior 1 month after his that on March 25, 1988, he figured in a quarrel with
hiring to determine whether the company another security guard, which resulted in a near
was amenable to continuing with his shootout; that at the end of his probationary
employment. employment, he was given a psychological test
and on the basis of the foregoing, petitioner told
Hence, respondent Flores could not be faulted him that his probationary employment had come
for precisely harboring the impression that he was to an end as he did not pass the company
hired as crane operator for a definite period of 1 standard and therefore, he could not be hired as a
year to commence upon his arrival at the work-site regular employee.
and to terminate at the end of 1 year. No other
condition was laid out except that he was to be on Labor Arbiter handed down a decision in favor of
probation for 3 months. No standard whatsoever by complainant. The respondent was ordered to
which such probationary period could be hurdled reinstate the complainant to his former position and
was specified and made known to him. accord to him the status of a regular employee,
and to refund to the complainant the deduction it
Due process dictates that an employee be had made from his salary in the amount of P20.00
apprised beforehand of the condition of his per month.
employment and of the terms of advancement
therein. Precisely, implicit in Art. 281 of the Code is NLRC affirmed the decision with a slight
the requirement that reasonable standards be modification: the refund of the deductions made
previously made known by the employer to the by respondent from complainants salaries in the
probationary employee at the time of his amount of P20.00 per month was vacated and set
engagement, such an essential requirement was aside.
not met by petitioner, even assuming that Flores'
alleged unsatisfactory performance was true. ISSUE: Whether or not private respondents
Besides, unsatisfactory performance is not one of employment with A Prime Security Services, Inc.
the just causes for dismissal under the Labor Code. was just a continuation of his employment with
Sugarland Security Services, Inc

Page 22 of 5
company's history, corporate philosophy,
RULING: YES. On the issue as to whether the private organizational structure, and company rules and
respondent is a probationary or regular employee, regulations, including the company standards for
the Court holds that the latter became a regular regularization, code of conduct and company-
employee upon completion of his six-month period provided benefits. Paras started reporting for work
of probation. Private respondent started working on on May 27, 1996. He was assigned at the paint
January 30, 1988 and completed the said period of ovens, air make-up and conveyors. As part of the
probation on July 27, 1988. Thus, at the time private MMPC's policy, Paras was evaluated by his
respondent was dismissed on August 1, 1988, he immediate supervisors after six (6) months, and
was already a regular employee with a security of received an average rating. Later, his supervisor
tenure. He could only be dismissed for a just and informed Paras that based on his performance
authorized cause. rating, he would be regularized.
There is no basis for subjecting private respondent
to a new probationary or temporary employment However, the Department and Division Managers,
on January 30, 1988, considering that he was reviewed the performance evaluation made on
already a regular employee when he was Paras. They unanimously agreed, along with Paras'
absorbed by A' Prime from Sugarland, its sister immediate supervisors, that the performance of
company. Paras was unsatisfactory. As a consequence, Paras
The Court cannot sanction the practice of some was not considered for regularization. On
companies which, shortly after a worker has November 26, 1996, he received a Notice of
become a regular employee, effects the transfer of Termination dated November 25, 1996, informing
the same employee to another entity whose him that his services were terminated effective the
owners are the same, or identical, in order to said date since he failed to meet the required
deprive subject employee of the benefits and company standards for regularization.
protection he is entitled to under the law.
The allegations of the private respondent that ISSUE:Whether or not Paras was already a regular
Sugarland is a sister company of A Prime and that employee on November 26, 1996.
the latter absorbed the security contracts and
security guards of Sugarland with the U.S. Embassy RULING: Yes, Paras was already a regular
were neither denied nor controverted by the employee.
petitioner before the Labor Arbiter.
Petitioners failure to deny that Sugarland is its sister An employer, in the exercise of its management
company and that petitioner absorbed Sugarlands prerogative, may hire an employee on a
security contract and security personnel assumes probationary basis in order to determine his fitness
overriding significance over the resignation to perform work. Under Article 281 of the Labor
theorized upon, evincing petitioners design to Code, the employer must inform the employee of
ignore or violate labor laws through the use of the the standards for which his employment may be
veil of corporate personality. considered for regularization. Such probationary
period, unless covered by an apprenticeship
Mitsubishi Motors Corp vs NLRC agreement, shall not exceed six (6) months from
the date the employee started working. The
FACTS: employees services may be terminated for just
cause or for his failure to qualify as a regular
Mitsubishi Motors Philippines Corporation (MMPC) is employee based on reasonable standards made
a domestic corporation engaged in the assembly known to him.
and distribution of Mitsubishi motor vehicles.
Chrysler Philippines Labor Union (CPLU) is a Respondent Paras was employed as a
legitimate labor organization and the duly certified management trainee on a probationary basis.
bargaining agent of the hourly-paid regular rank During the orientation conducted on May 15, 1996,
and file employees of MMPC. Nelson Paras was a he was apprised of the standards upon which his
member of CPLU while wife, Cecille Paras, was the regularization would be based. He reported for
President of the Chrysler Philippines Salaried work on May 27, 1996. As per the company's policy,
Employees Union (CPSU). the probationary period was from three (3) months
to a maximum of six (6) months.
Nelson Paras was first employed by MMPC as a
shuttle bus driver. He resigned and went to Saudi Applying Article 13 of the Civil Code, the
Arabia to work as a diesel mechanic and heavy probationary period of six (6) months consists of one
equipment operator in from 1982 to 1993. When he hundred eighty (180) days. This is in conformity with
returned to the Philippines, he was re-hired as a paragraph one, Article 13 of the Civil Code, which
welder-fabricator at MMPC tooling shop from provides that the months which are not designated
October 3,1994 to October 31, 1994. On October by their names shall be understood as consisting of
29, 1994, his contract was renewed from November thirty (30) days each. The number of months in the
1, 1994 up to March 3, 1995. probationary period, six (6), should then be
multiplied by the number of days within a month,
Sometime in May of 1996, Paras was re-hired on a thirty (30); hence, the period of one hundred eighty
probationary basis as a manufacturing trainee at (180) days.
the Plant Engineering Maintenance Department.
He and the new and re-hired employees were As clearly provided for in the last paragraph of
given an orientation on May 15, 1996 respecting the Article 13, in computing a period, the first day shall
Page 23 of 5
be excluded and the last day included. Thus, the In the instant case, petitioner cannot successfully
one hundred eighty (180) days commenced on say that he was never informed by private
May 27, 1996, and ended on November 23, 1996. respondent of the standards that he must satisfy in
The termination letter dated November 25, 1996 was order to be converted into regular status. This rans
served on respondent Paras only at 3:00 a.m. of (sic) counter to the agreement between the parties
November 26, 1996.He was, by then, already a that after five months of service the petitioners
regular employee of the petitioner under Article 281 performance would be evaluated. It is only but
of the Labor Code natural that the evaluation should be made vis--vis
the performance standards for the job. Private
respondent Trifona Mamaradlo speaks of such
RADIN C. ALCIRA, petitioner, vs. NATIONAL LABOR standard in her affidavit referring to the fact that
RELATIONS COMMISSION, MIDDLEBY PHILIPPINES petitioner did not perform well in his assigned work
CORPORATION/FRANK THOMAS, XAVIER G. PEA and and his attitude was below par compared to the
TRIFONA F. MAMARADLO, respondents. companys standard required of him.

FACTS: Petitioner Radin C. Alcira was hired by EXTENSION OF CONTRACT


respondent Middleby as engineering support MARIWASA MANUFACTURING v LEOGARDO
services supervisor on a probationary period for six (Narvasa, 1989)
months.
QUICK FACTS: Dequila, a probationary utility worker
Despite the indication of probationary period in the of Mariwasa, agreed to have his probationary
appointment paper, the dates indicated in the period extended for another 3 months after the first
copies in the possession of the petitioner and the 6 months, so that he may have another chance to
respondent, were different, May 20, 1996 and May improve his performance and qualify as a regular
27, 1996, respectively. worker. After the extension, he was terminated.

On November 20, 1996, unhappy with petitioners FACTS:


performance, respondent Middeby terminated the
formers services. Joaquin A. Dequila (or Dequilla) was hired on
probation by Mariwasa Manufacturing, Inc. as a
But according to the petitioner he is already a general utility worker on January 10, 1979. After 6
regular employee effective November 16, 1996, months, he was informed that his work was
using Article 13 of the Civil Code that one month is unsatisfactory and had failed to meet the required
composed of 30 days, six months total 180 days. standards. To give him another chance, and with
Dequilas written consent, Mariwasa extended
Dequilas probationary period for another three
Hence, using May 20, 1996 as the reference point, it
months: from July 10 to October 9, 1979. Dequilas
was already considered a dismissal since it was
performance, however, did not improve and
made after the lapse of his probationary
Mariwasa terminated his employment at the end of
employment.
the extended period.
ISSUE: WHETHER OR NOT PROBATIONARY
Dequila filed a complaint for illegal dismissal against
EMPLOYMENT IS EMPLOYMENT FOR A DEFINITE
Mariwasa and its VP for Administration, Angel T.
PERIOD
Dazo, and violation of Presidential Decrees Nos. 928
and 1389.
RULING: Section 6 (d) of Rule 1 of the Implementing
Rules of Book VI of the Labor Code (Department DIRECTOR OF MINISTRY OF LABOR: Complaint is
Order No. 10, Series of 1997) provides that: dismissed. Termination is justified. Thus, Dequila
appeals to the Minister of Labor.
(d) In all cases of probationary employment, the
employer shall make known to the employee the MINISTER OF LABOR: Deputy Minister Vicente
standards under which he will qualify as a regular Leogardo, Jr. held that Dequila was already a
employee at the time of his engagement. Where no regular employee at the time of his dismissal, thus,
standards are made known to the employee at that he was illegally dismissed. (Initial order:
time, he shall be deemed a regular employee. Reinstatement with full backwages. Later amended
to direct payment of Dequila's backwages from the
We hold that respondent Middleby substantially date of his dismissal to December 20, 1982 only.)
notified petitioner of the standards to qualify as a
regular employee when it apprised him, at the start ISSUE: WON employer and employee may, by
of his employment, that it would evaluate his agreement, extend the probationary period of
supervisory skills after five months. employment beyond the six months prescribed in
Art. 282 of the Labor Code?
Conversely, an employer is deemed to substantially
comply with the rule on notification of standards if RULING: YES, agreements stipulating longer
he apprises the employee that he will be subjected probationary periods may constitute lawful
to a performance evaluation on a particular date exceptions to the statutory prescription limiting such
after his hiring. We agree with the labor arbiter periods to six months.
when he ruled that:

Page 24 of 5
The SC in its decision in Buiser vs. Leogardo, Jr.
(1984) said that Generally, the probationary period
of employment is limited to six (6) months. The
exception to this general rule is when the parties to
an employment contract may agree otherwise,
such as when the same is established by company
policy or when the same is required by the nature
of work to be performed by the employee. In the
latter case, there is recognition of the exercise of
managerial prerogatives in requiring a longer
period of probationary employment, such as in the
present case where the probationary period was
set for eighteen (18) months, i.e. from May, 1980 to
October, 1981 inclusive, especially where the
employee must learn a particular kind of work such
as selling, or when the job requires certain ABSORBED EMPLOYEES
qualifications, skills experience or training.
CEBU STEVEDORING vs. REGIONAL DIRECTOR
In this case, the extension given to Dequila could Cebu Stevedoring Co., Inc. vs. The Honorable
not have been pre-arranged to avoid the legal Regional Director/Minister of Labor Arsenio Gelig
consequences of a probationary period and Maria Luz Quijano
satisfactorily completed. In fact, it was ex gratia, December 8, 1988
an act of liberality on the part of his employer Regalado, J.
affording him a second chance to make good
after having initially failed to prove his worth as an FACTS
employee. Such an act cannot now unjustly be
turned against said employer's account to compel Arsenio Gelig and Maria Luz Quijano were former
it to keep on its payroll one who could not perform employees of the Cebu Customs Arrastre Service
according to its work standards. (CCAS).

By voluntarily agreeing to an extension of the May 2, 1977 Pursuant to Customs Administrative


probationary period, Dequila in effect waived any Order No. 21-77 of Acting Commissioner of Customs
benefit attaching to the completion of said period and Acting Secretary of Finance, the CCAS was
if he still failed to make the grade during the period abolished for the reason that the objectives for
of extension. By reasonably extending the period of which it was created had already been attained.
probation, the questioned agreement actually As a consequence, all the employees of CCAS,
improved the probationary employee's prospects including Gelig and Quijano, were given their
of demonstrating his fitness for regular employment. termination and/or separation pay by the Bureau of
Customs, Cebu City, computed up to April 30, 1977.
Petition granted. Order of Deputy Minister Leogardo
reversed. Case for illegal dismissal is dismissed. May 3, 1977 all the employees of CCAS, including
Gelig and Quijano, were absorbed by Cebu
Stevedoring Co., Inc. (CSCI), with the same
positions that they held in CCAS.

October 17, 1977 Gelig and Quijano were


dismissed by CSCI without prior clearance,
allegedly for redundancy.

Gelig and Quijano filed a complaint for


reinstatement with backwages.

Regional Office of Ministry of Labor:

Gelig and Quijano were employed by CCAS and


their functions were carried over when they were
absorbed by CSCI. There was no need to employ
them as probationary because they were already
well-trained in their functions. A probationary period
of employment means that an employee is hired for
training for a certain period in order to determine
whether they qualify for the position or not. Here,
Gelig and Quijano cannot be considered
probationary.

Minister of Labor affirmed. CSCI elevated the case


to the Office of the President which, through
Presidential Executive Assistant Jacobo Clave,
issued a resolution dismissing the appeal as there
Page 25 of 5
was no law expressly recognizing the parties right time this supposed program was mentioned
to appeal to the Office of the Pres and there was was when CSCI was trying to justify the
no exceptionally meritorious cause for the exercise dismissal of Gelig and Quijano before the
of the constitutional power of review of the labor arbiter.
President/Prime Minister.
The constitutional duty of the State to protect the
CSCIs position: right of laborers to security of tenure demands than
1. Upon abolition of CCAS, all its employees an employer may be permitted to terminate the
were given separation pay. Thus, when the services of an employee only under conditions
employees were absorbed by CSCI when it allowed by and with due process of law.
took over the arrastre operations, they were
all employed as casuals. DISPOSITIVE
2. As casuals, Gelig and Quijano can be Termination of Gelig and Quijano was unjust and
terminated within the 6-month period illegal. CSCI is ordered to reinstate Gelig and
without need of clearance from Ministry of Quijano to their fromer positions at the time of
Labor and neither is the employer obligated dismissal, or if such reinstatement is not possible, to
to pay them termination pay. Gelig and substantially equivalent positions, without loss of
Quijano were terminated on October 18, seniority rights and other privileges appertaining
1977 or within 5 months from employment. thereto and to pay them 3 years backwards, from
3. The positions occupied by Gelig and Octber 18, 1977 without qualification or deduction.
Quijano with CCAS were identical with the If reinstatement is not possible due to
positions already filled up and with the same supervenience of events which prevent the same,
functions being discharged in the main CSCI is ordered to pay them, more as a vindication
office of CSCI. Thus, they may be of a right and less as indemnification of a loss,
terminated for redundancy and financial separation pay equivalent to 1 months salary
losses incurred by CSCI justify the based on their monthly salaries as of October 17,
retrenchment of employees. 1977.

ISSUE: Whether the termination of Gelig and


Quijano (and 52 others) was valid (NO)

REASONING

Gelig and Quijano could not be considered


probationary employees because they were
already well-trained in their respective functions.
Gelig had been a clerk for CCAS for more than 10
years, while Quijano had slightly less than 10 years
of service.

Findings of quasi-judicial agencies (pertaining to


Ministry of Labor here) which have acquired
expertise because their jurisdiction is confined to
specific matters are generally accorded not only
respect but, at times, even finality where such
findings are supported by substantial evidence.

It is true that Article 283 of the Labor Code provides


that an employer may also terminate the
employment of any employee due to the
installation of labor-serving devices, redundancy,
retrenchment to prevent losses or the closing or
cessation of operation of the establishment or
undertaking.

BUT:
1. Records failed to establish that the positions
occupied by Gelig and Quijano with CCAS
are identical with those presently existing in
CSCIs office.
2. CSCI kept them in its employ for almost 6
months without raising issue of redundancy.
3. CSCIs submission that it is suffering financial
losses is untenable since it absorbed and
employed for almost 6 months, without any
intimation of supposed financial distress, the
majority of the former employees of CCAS. It
never advised Gelig and Quijano of a
company retrenchment program. The first
Page 26 of 5
RULES ON PRIVATE SCHOOL TEACHERS

[G.R. No. 121962. April 30, 1999]

ESPERANZA C. ESCORPIZO, and UNIVERSITY OF


BAGUIO FACULTY EDUCATION WORKERS
UNION, petitioners, vs. UNIVERSITY OF BAGUIO and
VIRGILIO C. BAUTISTA and NATIONAL LABOR
RELATIONS COMMISSION, respondents.

FACTS
Esperanza Escorpizo was a high school teacher in
the University of Baguio contracted to be a
probationary teacher from 1989-1991. The terms of
her employment stipulate that in order for her to be
regularized, she should get a satisfactory rating and
she should pass the board examination for
teachers. After the lapse of her probationary
period, UB was not supposed to re-sign her because
she failed the board exam but upon her pleading
she was re-signed for one year but still a
probationary. She took the board exam again but
she failed during the same school year. So when UB
was considering the list of teachers for next year,
Escopizo was no longer considered. Meanwhile,
Escorpizo again took the exam this time she passed.
But UB no longer took her in to teach. Her labor
union assisted her in suing UB averring that in the
CBA, passing the board is not a requisite to be
regularized.
ISSUE: Whether or not Escorpizo should be reinstated
as a teacher.
HELD: No. Escorpizo was not illegally dismissed. UB
was well within its right to require its teachers to pass
the board before teaching, The Department of
Education also rolled out an order requiring that
teachers should pass the board before teaching.
This is to ensure the quality of education in the
country. As between the CBA and the DECS order,
what should prevail is the requirements so provided
by the government.

Page 27 of 5
RULES ON PRIVATE SCHOOL TEACHERS On 08 February 1993, respondent de la Pea
wrote petitioner Sis. Rosalinda Bayla, O.S.A., princip
al of High School, stating that he would like to apply
[G. R. No. 127241. September 28, 2001] for reinstatement as a faculty member for SY 1993-
LA CONSOLACION COLLEGE, SR. ROSALINDA BAYLA, 1994. [9]
SR. CELIA BAYONA, REODITA MABAYAG, JUDITH
In a letter dated 11 March 1993, the academic
VERDADERO and JOSE BAYOGUING, petitioners,
team composed of petitioners Erodita P. Madayag,
vs. NATIONAL LABOR RELATIONS
Verdadero and Bayoguing informed respondent de
COMMISSION and JOSE DE LA PEA, III, respondents.
la Pea of his unsatisfactory performance and
FACTS
advised him that the school would no longer hire
La Consolacion College (LCC) initially him for the incoming school year.
employed Jose de la Pea III as a CAT Commandant
and YCAP Coordinator for school year 1975-1976.
His employment as YCAP coordinator lasted until 29
September 1979, after which he resigned. He
severed all ties with LCC when he left in 1980.Prior to On 9 June 1993, respondent de la Pea filed
his resignation and despite demands by LCC for with Regional Arbitration Branch No. VI, Bacolod
him to submit a syllabi in YDT I, II, III, and CAT I City a complaint against LCC and/or Rosalinda
containing course objectives, subject matter, Bayla, Sr. Celia Bayona, Erodita Mabayag, Judith
content, concepts, skills, activities and evaluation Verdadero and Jose Bayoguing, for illegal dismissal,
not later than 12 November 1979, respondent de la moral damages and exemplary damages. After
Pea failed to comply. submission of position papers, on 11 November
1994, Labor Arbiter Reynaldo J. Gumaltico rendered
After his employment with LCC, respondent de a decision dismissing the complaint, holding that at
la Pea sought and found employment in other the time respondent de la Pea was dismissed, he
establishments. had not attained regular status. The Labor Arbiter
also found respondent de la Pea guilty of serious
However, on 2 December 1991, LCC received
misconduct and gross disobedience which were
an application from respondent de la Pea. The
just causes for termination of service.
applicant requested that he be considered for the
positions of CAT Commander and YDT Instructor, On appeal to the NLRC, on 31 January 1996,
positions he held for eleven (11) years prior to his the NLRC rendered a resolution reversing the
resignation from LCC. decision of the labor arbiter. The NLRC held that
respondent de la Pea attained regular status at the
In June 1992, LCC appointed respondent de la
time he was dismissed and that LCC failed to prove
Pea as a classroom teacher in physical education
the existence of just cause to warrant his
and health, a position he never held during his
dismissal.] On 4 March 1996, LCC filed a motion for
previous employment with LCC.
reconsideration of the NLRC decision; however, on
The written contract of employment between 19 August 1996, the NLRC denied the motion.
LCC and respondent de la Pea expressly provided
that the employment was for one (1) academic
year, that is, from June 1992 to March
1993. Respondent de la Pea accepted such
condition.[6] ISSUE

On 14 July 1992, petitioner Jose B. Bayoguing, WHETHER OR NOT the NLRC committed palpable
Jr., a member of the academic team tasked to error amounting to grave abuse of discretion in
evaluate the performance of the schools teachers, ruling that respondent Jose de la Pea was a regular
reminded respondent de la Pea in writing to or permanent employee of La Consolacion College
comply with the requirements and standard in a position in which he had not undergone the
operating procedure of the school, namely; timely three (3) year probationary period provided in the
submission of lesson plans, class records and other manual of regulations for private schools.
papers, attendance at regular monthly meetings, RULING
and informing the school of absences. Respondent
de la Pea ignored the reminder without any valid The Supreme Court reverse the NLRC decision
reason, and continued to defy these requirements having been issued in grave abuse of discretion.
and procedures. In the case at bar, there is a written contract
On 27 November 1992, respondent de la Pea defining the period of employment of respondent
called an emergency meeting of faculty members. de la Pea.
In said meeting, respondent de la Pea berated Clearly, the employment was not permanent
petitioner Bayoguing, shouted invectives, ridiculed but for a specified duration of one school year.
and threatened Bayoguing with bodily harm. No
untoward incident ensued as petitioner Bayoguing In resolving the issue of whether or not
kept his composure. During the same faculty respondent de la Pea was permanent employee of
meeting, respondent de la Pea was physically petitioner, it is the Manual of Regulations for Private
restrained by his fellow teachers whenever he Schools, not the Labor Code, which is
would charge the person of petitioner Bayoguing. applicable. This was settled in University of Sto.
Tomas v. NLRC, where we ruled that for a private
school teacher to acquire permanent status in
Page 28 of 5
employment the following requisites must
concur: (1) the teacher is a full-time teacher; (2) the
teacher must have rendered three (3) consecutive
years of service; and (3) such service must have
been satisfactory

Page 29 of 5
CHIANG KAI SHEK COLLEGE, and CHIEN YIN 1993-1994. In fact, they should not have required
SHAO, petitioners, vs. HON. COURT OF APPEALS; her to re-apply to teach. In accordance with the
HON. NATIONAL LABOR RELATIONS COMMISSION; written statement of policies dated 12 March 1993,
HON. COMMISSIONER VICTORIANO R. CALAYLAY, only probationary teachers are required by the
HON. PRESIDING COMMISSIONER RAUL T. AQUINO, petitioners to re-apply in March. Failure of
and HON. COMMISSIONER ANGELITA A. GACUTAN; probationary teachers to re-apply in March is an
and MS. DIANA P. BELO, respondents. indication of their lack of interest to teach again at
G.R. No. 152988 August 24, 2004 the school.

Facts: Private respondent Belo was employed as a Petitioners invocation of the third policy that of
permanent teacher by petitioner CKSC for 15 years. giving teaching assignments to probationary
Belo had to take a leave of absence for the S.Y. teachers in April to justify their refusal to provide
1992 1993 due to personal reasons. Petitioner Ms. Belo a teaching load is, therefore, a lame
informed her that they could not guarantee her a excuse that rings of untruth and
teaching load when she would return and that only dishonesty. Patently clear is the illegal manner by
teachers in service may enjoy the privilege and which the petitioners eased out Ms. Belo from the
benefits provided by the school. When she signified teaching corps.
her intention to return to teach for the S.Y. 1993
1994, petitioner reasoned that it already hired non
permanent teachers to take her load. The Labor
Arbiter reasoned that she was not dismissed but
there was simply no available teaching load for her.
The NLRC reversed the LAs decision and ordered
her reinstatement with full back wages. The Court of
Appeals declared that Belo was constructively
dismissed; the dismissal, illegal, for being violative of
her security of tenure.

Issue: Whether private respondent was


constructively dismissed and therefore entitled to
reinstatement and back wages

Held: It must be noted at the outset that Ms. Belo


had been a full-time teacher in petitioner CKSC
continuously for fifteen years or since 1977 until she
took a leave of absence for the school year 1992-
1993. Under the Manual of Regulations for Private
Schools, for a private school teacher to acquire a
permanent status of employment and, therefore,
be entitled to a security of tenure, the following SECURITY OF TENURE
requisites must concur: (a) the teacher is a full-time
teacher; (b) the teacher must have rendered three LLOSA-TAN V SILAHIS INTERNATIONAL HOTEL
consecutive years of service; and (c) such service
must have been satisfactory.
181 SCRA 738 February 5, 1990
Since Ms. Belo has measured up to these standards,
she therefore enjoys security of tenure. The
fundamental guarantees of security of tenure and
due process dictate that no worker shall be NATURE
dismissed except for just and authorized cause
provided by law and after due notice and hearing. Petition for certiorari seeking to set aside the
decision and resolutions of the NLRC
Case law defines constructive dismissal as a
cessation from work because continued
employment is rendered impossible, unreasonable,
or unlikely; when there is a demotion in rank or a FACTS
diminution in pay or both; or when a clear
discrimination, insensibility, or disdain by an - The complainant was a front office cashier of
employer becomes unbearable to the employee. Silahis International Hotel since November 2, 1976
until her questioned dismissal on October 30, 1982.
It, therefore, blows our mind why the petitioners
would require Ms. Belo, a permanent teacher since
- Since 1977, the Silahis International Hotel, had a
1977 with a satisfactory service record, to signify her
standing corporate policy (Corporate Policy No.
intention to teach in March 1993. Plainly, the
014), which orders all cashiers of SMC and its
petitioners violated their avowed policies. Since Ms.
affiliates to refuse the cashing of personal checks of
Belo was not retiring, resigning or filing another
employees and officials, endorsement by any
leave of absence after the school year 1992-1993,
executive of the Sulo Management Company, or
the petitioners should have considered her as
Philippine Village Hotel or Silahis International Hotel
consenting to teach for the incoming school year
Page 30 of 5
or Sulo Hotel notwithstanding, because based on - Admittedly, the encashment of the checks in
experience, a number of these checks question is a violation of Policy No. 014 of said hotel.
unfortunately bounce to the detriment of SMC and But as found by the Labor Arbiter, it was established
its affiliates. that: (a) complainant was not motivated by bad
faith; (b) Policy No. 014 is not strictly or consistently
- On August 22, 1982, while petitioner was on duty, enforced but has been relaxed repeatedly to meet
she was approached by Mr. Gayondato, the business exigencies; and (c) complainant's
general cashier of Puerto Azul Beach Resorta encashment of the checks in question was not only
sister company of Silahis International Hotel and with the knowledge but with clearance from her
nephew of the Executive Vice President, to encash superiors who are more knowledgeable as to the
two (2) US dollar checks with a combined value of circumstances under which the enforcement of the
US$1,200.00 or P10,389.60. same may be relaxed.

- Although petitioner politely explained the - Moreover, it cannot be said that complainant was
existence of Policy No. 014 prohibiting such precipitate or that she has acted in utter disregard
transactions, Gayondato persisted and assured that of consequences. On the contrary, she refused to
the presentation of aforesaid checks to the front encash subject checks despite the request of Mr.
office cashier was upon instructions of the Executive Gayondato, the general cashier of Puerto Azul, but
Vice President. was persuaded only upon the assurances of the
latter that such was the wish of the Executive Vice
- Petitioner, eventually encashed the aforesaid President and that said encashment was necessary
checks, notwithstanding Corporate Policy No. 014. to meet certain disbursements in Puerto Azul. In
addition, she informed personally Mr. Samuel Grulla,
Assistant Manager of the Silahis International Hotel,
- Thereafter, the said checks bounced.
of said encashment, who also told her that such is
"alright".
- On October 1, 1982, respondent Vanessa
Suatengco issued a memorandum to the petitioner
- Finally, against the background of her previous
requiring her to explain in writing why she should not
experience when she refused to encash a similar
be terminated for encashing the two (2) personal
check for Mr. Katte, the Food and Beverage
checks without proper authorization.
Manager of Silahis International Hotel, and that she
was reprimanded by the management of the Silahis
- Despite petitioner's explanation, her services were
International Hotel for her refusal, as well as
terminated effective October 30, 1982.
threatened with suspension or dismissal from her
job, coupled with the advice of Mr. Nestor
- Petitioner filed a complaint against respondents Famatigan, Jr., Silahis International Hotel
for illegal dismissal. Comptroller, to use her discretion in handling similar
requests in the future, it is not at all surprising that
- Labor Arbiter Virginia G. Son rendered a decision she opted to take subject course of action.
in favor of petitioner.
- It is well settled that dismissal based on loss of trust
- Hotel appealed the decision of the LA to the and confidence arising from alleged misconduct of
NLRC, and the NLRC rendered a decision setting employee, is not to be used as a shield to dismiss an
aside the decision of the Labor Arbiter and employee arbitrarily. Although the power to dismiss
dismissing the complaint for illegal dismissal for lack is a normal prerogative of the employer, the same is
of merit not without limitations. The right of the employer
must not be exercised arbitrarily and without just
- Petitioners 2 MFRs having been denied, recourse cause. Otherwise, the constitutional guarantee of
was made to the SC security of tenure of the workers would be rendered
nugatory. While dismissing or laying off of an
employee is a management's prerogative, it must
nevertheless be done without abuse of discretion.
ISSUE Furthermore, the right of employer to freely select or
discharge his employees is regulated by the State,
because the preservation of the lives of the citizens
WON the acts of petitioner constitute gross
is a basic duty of the State, more vital than the
negligence resulting in a valid ground for the
preservation of the corporate profit. In addition,
termination of her employment
security of tenure is a right of paramount value
guaranteed by the Constitution and should not be
denied on mere speculation. Protection for labor
and social justice provisions of the Constitution and
HELD the labor laws and rules and regulations are
interpreted in favor of the exercise of labor rights.
NO
Disposition The assailed decision of the NLRC is
- Gross negligence has been defined as the want of DISMISSED, and SET ASIDE and private respondent
any or slight care or the utter disregard of Silahis International Hotel is ordered to reinstate
consequences. petitioner Anita Llosa-Tan to her former position or

Page 31 of 5
similar position without loss of seniority rights with full accomplished. The last element, the so-called
backwages beginning October 30, 1982 for a "control test", is the most important element.
period of three (3) years therefrom.
Sonzas services to co-host its television and radio
programs are because of his peculiar talents, skills
and celebrity status. Independent contractors often
present themselves to possess unique skills, expertise
or talent to distinguish them from ordinary
employees. The specific selection and hiring of
SONZA, because of his unique skills, talent and
celebrity status not possessed by ordinary
employees, is a circumstance indicative, but not
++++++++++++++++++++++++++++++=++++++++++ conclusive, of an independent contractual
+++++ SIR relationship. All the talent fees and benefits paid to
CRUDA+++++++++++++++++++++++++++++++++++ SONZA were the result of negotiations that led to
++++++++++ the Agreement. For violation of any provision of the
Agreement, either party may terminate their
relationship. Applying the control test to the present
case, we find that SONZA is not an employee but an
independent contractor.
JOSE SONZA vs. ABS-CBN BROADCASTING
CORPORATION The control test is the most important test our courts
G.R. No. 138051 apply in distinguishing an employee from an
June 10, 2004 independent contractor. This test is based on the
extent of control the hirer exercises over a worker.
Facts: In May 1994, ABS-CBN signed an agreement The greater the supervision and control the hirer
with the Mel and Jay Management and exercises, the more likely the worker is deemed an
Development Corporation (MJMDC). ABS-CBN was employee. The converse holds true as well the less
represented by its corporate officers while MJMDC control the hirer exercises, the more likely the worker
was represented by Sonza, as President and is considered an independent contractor. To
general manager, and Tiangco as its EVP and perform his work, SONZA only needed his skills and
treasurer. Referred to in the agreement as agent, talent. How SONZA delivered his lines, appeared on
MJMDC agreed to provide Sonzas services television, and sounded on radio were outside ABS-
exclusively to ABS-CBN as talent for radio and CBNs control. ABS-CBN did not instruct SONZA how
television. ABS-CBN agreed to pay Sonza a monthly to perform his job. ABS-CBN merely reserved the
talent fee of P310, 000 for the first year and P317, right to modify the program format and airtime
000 for the second and third year. schedule "for more effective programming." ABS-
CBNs sole concern was the quality of the shows
On April 1996, Sonza wrote a letter to ABS-CBN and their standing in the ratings.
where he irrevocably resigned in view of the recent
events concerning his program and career. After Clearly, ABS-CBN did not exercise control over the
the said letter, Sonza filed with the Department of means and methods of performance of Sonzas
Labor and Employment a complaint alleging that work. A radio broadcast specialist who works under
ABS-CBN did not pay his salaries, separation pay, minimal supervision is an independent contractor.
service incentive pay,13th month pay, signing Sonzas work as television and radio program host
bonus, travel allowance and amounts under the required special skills and talent, which SONZA
Employees Stock Option Plan (ESOP). ABS-CBN admittedly possesses.
contended that no employee-employer
relationship existed between the parties. However, ABS-CBN claims that there exists a prevailing
ABS-CBN continued to remit Sonzas monthly talent practice in the broadcast and entertainment
fees but opened another account for the same industries to treat talents like Sonza as independent
purpose. contractors. The right of labor to security of tenure
as guaranteed in the Constitution arises only if there
The Labor Arbiter dismissed the complaint and is an employer-employee relationship under labor
found that there is no employee-employer laws. Individuals with special skills, expertise or talent
relationship. NLRC affirmed the decision of the enjoy the freedom to offer their services as
Labor Arbiter. CA also affirmed the decision of independent contractors. The right to life and
NLRC. livelihood guarantees this freedom to contract as
independent contractors. The right of labor to
Issue: Whether or not there was employer- security of tenure cannot operate to deprive an
employee relationship between the parties. individual, possessed with special skills, expertise
and talent, of his right to contract as an
Ruling: NO. Case law has consistently held that the independent contractor.
elements of an employee-employer relationship are
selection and engagement of the employee, the
payment of wages, the power of dismissal and the
G.R. No. 80383 September 26, 1988
employers power to control the employee on the
REV. FR. EMMANUEL LABAJO, personally and in his
means and methods by which the work is
capacity as Director of San Andres High School,
Maramag, Bukidnon, and, SAN ANDRES HIGH
Page 32 of 5
SCHOOL OF MARAMAG, respondents were not under probation at the time
INCORPORATED, petitioners, of their dismissal; even assuming that private
vs. respondents were then merely probationary
PUREZA V. ALEJANDRO, ZENAIDA S. DAHILAN, employees of and under contract with petitioner
JOSEPHINE A. CHAN, HERNANI C. MIAGUE, OPHELIA High School, nevertheless, they could only be
M. MIAGUE, ROLANDO T. AMAR and The HON. dismissed for cause and only after having been
NATIONAL LABOR RELATIONS accorded due process.
COMMISSION, respondents.
NLRCs RULING: In a Resolution dated 8 May 1987,
FACTS:The six (6) private respondents had all been public respondent National Labor Relations
contracted by petitioners to work as classroom Commission, affirmed on appeal the decision of the
teachers at the San Andres High School, a private Labor Arbiter. A Motion for Reconsideration filed by
learning institution situated in Maramag, Bukidnon. petitioners was denied by the Commissions.
Private respondents Pureza V. Alejandro and Hence, this petition.
Rolando T. Amar were assigned to handle petitioner
High School's regular day classes; private ISSUE: whether or not the private respondents were
respondents Zenaida S. Dahilan, Josephine A. illegally dismissed by petitioners
Chan, Hernani C. Miague and Ophelia M. Miague
upon the other hand, were assigned to handle the RULING: NO. The Supreme Court cited the Biboso vs.
school's special evening classes. Victorias Milling Company Inc., case which also
On 3 June 1985, private respondents filed a involved the separation of private school teachers,
Complaint (docketed as NRLRC RAB X Case No. 5- probationary employees who had been covered
0410-85) with Regional Arbitration Branch No. 10 similarly by corresponding contracts of
(Cagayan de Oro City) of the then Ministry of Labor employment. The Court, speaking through then Mr.
and Employment, alleging that they had each Justice Fernando, stated in that case:
received on 29 March 1985 from petitioner Fr.
Emmanuel Labajo, Director of the San Andres High 2. This is by no means to assert that the security of
School, a letter dated March 13, 1985 stating that tenure protection of the Constitution does not
they are terminated effective March 31, 1985. apply to probationary employees. The Labor Code
has wisely, provided for such a case thus: 'The
Private respondents alleged that their dismissal by termination of employment of probationary
petitioner High School was without justifiable cause employees and those employed with a fixed period
and in violation of their rights to due process and shall be subject to such regulations as the Secretary
security of tenure. It was also alleged that of Labor may prescribe to prevent the
petitioners had failed to pay private respondents circumvention of the right of the employees to be
the full amounts corresponding to certain secured in their employment as provided herein.
employment benefits (i.e., daily wages, basic pay,
service incentive leave, sick leave, and cost of In view of all the foregoing, we hold that none of
living allowance) granted to the latter under labor the six (6) private respondents in this case, at the
laws. time of their separation, had achieved permanent
status in their employment as teachers at the San
Petitioners admitted that private respondents had Andres High School. As probationary and
not been paid in full the employment benefits contractual employees, private respondents
enumerated by the latter in their complaint. enjoyed security of tenure, but only to a limited
Petitioners alleged, however, that private extent i.e., they remained secure in their
respondents, prior to their acceptance of teaching employment during the period of time their
jobs at the San Andres High School, "were already respective contracts of employment remained in
made aware that the school could not give them effect. That temporary security of tenure, however,
everything due them under existing laws" and, ended the moment their employment contracts
hence, were estopped from claiming such benefits. expired on 31 March 1985 and petitioners declined
Petitioners, further, denied having dismissed illegally to renew the same for the next succeeding school
any of the six (6) private respondents and, in turn, year. Consequently, as petitioners were not under
alleged that each of the latter were, at the time of obligation to renew those contracts of
their dismissal, merely probationary employees of employment, the separation of private respondents
the San Andres High School whose services thereat in this case cannot be said to have been without
were terminated for just cause i.e., upon expiration justifiable cause, much less illegal.
on 31 March 1985 of their respective contracts of
employment with petitioner High School and before SKILLWORD MANAGEMENT AND MARKETING
any of them had achieved regular or permanent CORPORATION V NLRC (MANUEL)
status in their jobs. 186 SCRA 465 June 13, 1990

LABOR ARBITERS RULING: The Labor Arbiter held NATURE


that dismissal of the six (6) private respondents in this Petition for certiorari
case "was violative of the Constitution which
guarantees security of tenure of employment and a FACTS: On June 24, 1983, Francisco Manuel was
provision of Batas Pambansa Blg. 130 which requires deployed to Saudi Arabia to work as driver by
notice and investigation before outright termination petitioner Skillworld Management and Marketing, a
from the service." The Labor Arbiter also held that, duly licensed recruitment agency operated by
contrary to the claim of petitioners, private petitioners-spouses Serafin and Alicia Ramos. Upon
Page 33 of 5
his arrival in Jeddah, Manuel signed a 2-year with the Shary Limousine Branch in Jeddah. It is for
employment contract with his foreign employer, this reason that after being accosted twice at
petitioner Shary Limousine for a monthly basic salary checkpoints by Saudi police, who informed
of $300. 2 months later, Manuel was repatriated to complainant and his co-drivers that the alleged
the Philippines. Upon his arrival in the Philippines, temporary licenses were not valid, they brought the
Manuel confronted the Ramoses who promised to matter first to their Lebanese superior and then to
deploy him to other projects. the Philippine Embassy.

- After the lapse of more than one year without - Further, records show that Manuel reported for
being deployed to other projects of petitioners, work regularly and even rendered regular overtime
Manuel filed a complaint with the POEA against services; that he did not even attempt to join a
petitioners for illegal dismissal. He alleged that while strike or any other form of mass action while working
he was employed as driver of Shary Limousine in its in Jeddah, because he knew that the laws in
branch at Jeddah he was stopped, and his driver's Jeddah are very strict and being a foreigner he did
license sought for inspection, by Saudi Arabian not have the courage to join much less lead a strike
police. He showed the police two documents given which is prohibited there; that he and his co-workers
to him by his employer, Shary Limousine who made merely inquired from the Philippine Embassy why
him believe that these pertained to a driver's they were allowed to drive without licenses; and
temporary license. However, Manuel was informed that their action prompted the Philippine Embassy
that the documents were not valid for a drivers to write their employer, which is perfectly in order as
license. Together with eleven other drivers, they it was designed to protect them in foreign soil.
brought the matter before their superiors. Three
days after bringing the matter to his superior, MANAGERIAL EMPLOYEE
respondent was ordered to pack his things. He was INTERORIENT MARITIME ENTERPRISES INC V NLRC
taken to Riyadh and from there, repatriated to the (TAYONG)
Philippines. Upon respondent's arrival in the 235 SCRA 268 August 11, 1994
Philippines, he requested the Ministry of Foreign
Affairs for a translation of what purported to be his NATURE PETITION for review of a decision of the
driver's license. When translated it was only a National Labor Relations Commission
certification of employment with Shary Limousine in FACTS Captain Rizalino Tayong, a licensed Master
its branch at Jeddah. Mariner with experience in commanding ocean-
going vessels, was employed on 1989 by petitioners
- Petitioners alleged that Manuels dismissal was for for 1 yr as stated in his employment contract. He
a valid and just cause. Petitioners alleged that assumed command of petitioners vessel at the port
Manuel was dismissed because of disobedience, of Hongkong. His instructions were to replenish
absenteeism, refusal to work and banding together bunker and diesel fuel, to sail to South Africa and
to engage in concerted activities against the there to load 120,000 metric tons of coal. However,
employer. while in HK and unwarding cargo, he received a
weather report that a storm would hit HK, so
- POEA rendered judgment in favor of Manuel, precautionary measures were taken to secure the
directing petitioners to pay him $6,900.00 or its peso safety of the vessel and its crew, considering that
equivalent. Upon appeal, the NLRC affirmed said the vessels turbo-charger was leaking and the
decision. vessel was 14 yrs old. He also followed-up the
requisition by the former captain for supplies of
- According to petitioners, because of the oxygen and acetylene necessary for the welding-
probationary status of the employment of Manuel, repair of the turbo-charger and the economizer.
he may be dismissed at any time. Furthermore, this
agreement was contained in paragraph four (4) of -The vessel then sailed from HK for Singapore.
the employment contract signed by Manuel. Captain Tayong reported a water leak from M.E.
Turbo Chapter No. 2 Exhaust gas casing so he was
ISSUE WON Manuel was illegally dismissed instructed to black off the cooling water and
maintain reduced RPM unless authorized by the
HELD YES There is no dispute that as a probationary owners. However, the vessel stopped in mid-ocean
employee, Manuel had but a limited tenure. for 6 hrs and 45 minutes due to a leaking
Although on probationary basis, however, he still economizer. He was instructed to shut down the
enjoys the constitutional protection on security of economizer and use the auxiliary boiler instead.
tenure. During his tenure of employment therefore,
or before his contract expires, he cannot be - The Chief Engineer reminded Captain Tayong that
removed except for cause as provided for by law. the oxygen and acetylene supplies had not been
delivered. He then informed the shipowner that the
- The alleged causes for which private respondent departure of the vessel for South Africa may be
was dismissed (disobedience, absenteeism, refusal affected because of the delay in the delivery of the
to work, etc.) were not established. Respondent supplies. The shipowner advised Captain Tayong to
NLRC found that the purported temporary licenses contact its technical director who would provide a
to drive issued to Manuel and his co-drivers by their solution for the supply of said oxygen and
employer-the Shary Rent a Car/Limousine, turned acetylene. The technical director recommended to
out to be mere certifications to the effect that they Captain Tayong that by shutting off the water to
are Filipino citizens who are holders of given the turbo charger and using the auxiliary boiler,
passport numbers and that they were sent to work there should be no further problem. Captain
Page 34 of 5
Tayong agreed to the recommendation of the - NLRCs conclusion was supported by substantial
technical director, but communicated his evidence: The official report of the technical
reservations regarding proceeding to South Africa director, which stated that a disruption in the
without the requested supplies. So the shipowner normal functioning of the vessels turbo charger
advised him to wait for the supplies. and economizer had prevented the full or regular
- Finally, the vessel arrived at South Africa. However, operation of the vessel and that he was the one
Captain Tayong was instructed to turn-over his post who recommended the reduction of RPM during
to the new captain, and was repatriated to the the voyage to South Africa instead of waiting in
Philippines after serving petitioners for around 2 wks. Singapore for the supplies that would permit
He was not informed of the charges against him, shipboard repair of the malfunctioning machinery
and was just sent a letter after arriving in the and equipment, supported NLRCs conclusion that
Philippines. He therefore instituted a complaint for Captain Tayong did not arbitrarily and maliciously
illegal dismissal before the POEA, claiming his delay the voyage to South Africa.
unpaid salary for the unexpired portion of the
written employment contract, plus attorneys fees. - Captain Tayong's decision (arrived at after
consultation with the vessel's Chief Engineer) to wait
- POEA: dismissed complaint, there was valid cause seven (7) hours in Singapore for the delivery on
for his untimely repatriation (the company alleged board the Oceanic Mindoro of the requisitioned
that due to Captain Tayongs refusal to sail supplies needed for the welding-repair, on board
immediately to South Africa, the vessel was placed the ship, of the turbo-charger and the economizer
off-hire by the charterers, and the charterers equipment of the vessel, did not constitute merely
refused to pay the charter hire or compensation arbitrary, capricious or grossly insubordinate
corresponding to 12 hours, amounting to US behavior on his part. In the view of the NLRC, that
$15,500.00.They fired Captain Tayong for lost of decision of Captain Tayong did not constitute a
confidence; POEA believed that the Captains legal basis for the summary dismissal of Captain
concern for the oxygen and acetylene was not Tayong and for termination of his contract with
legitimate as these supplies were not necessary or petitioners prior to the expiration of the term
indispensable for running the vessel.) thereof.
Obiter
- NLRC: reversed and set aside POEA decision
because Captain Tayong had not been afforded - The captain of a vessel is a confidential and
an opportunity to be heard and that no substantial managerial employee within the meaning of the
evidenced was adduced to establish the basis for above doctrine. A master or captain, for purposes
petitioners loss of trust or confidence. Captain had of maritime commerce, is one who has command
acted in accordance with his duties to maintain the of a vessel. A captain commonly performs three (3)
seaworthiness of the vessel and to insure the safety distinct roles: (1) he is a general agent of the
of the ship and crew. shipowner; (2) he is also commander and technical
director of the vessel; and (3) he is a representative
ISSUE WON Captain Tayong was arbitrarily dismissed of the country under whose flag he navigates. Of
and without cause as reasonably established in an these roles, by far the most important is the role
appropriate investigation (whether or not Captain performed by the captain as commander of the
Tayong had reasonable grounds to believe that the vessel; for such role (which, to our mind, is
safety of the vessel and the crew under his analogous to that of "Chief Executive Officer" [CEO]
command or the possibility of substantial delay at of a present-day corporate enterprise) has to do
sea required him to wait for the delivery of the with the operation and preservation of the vessel
supplies needed for the repair of the turbo-charger during its voyage and the protection of the
and the economizer before embarking on the long passengers (if any) and crew and cargo. In his role
voyage from Singapore to South Africa) as general agent of the shipowner, the captain has
authority to sign bills of lading, carry goods aboard
HELD YES. Ratio It is well settled in this jurisdiction and deal with the freight earned, agree upon rates
that confidential and managerial employees and decide whether to take cargo. The ship
cannot be arbitrarily dismissed at any time, and captain, as agent of the shipowner, has legal
without cause as reasonably established in an authority to enter into contracts with respect to the
appropriate investigation. Such employees, too, are vessel and the trading of the vessel, subject to
entitled to security of tenure, fair standards of applicable limitations established by statute,
employment and the protection of labor laws. contract or instructions and regulations of the
shipowner. To the captain is committed the
Reasoning: Captain Tayong was denied any governance, care and management of the vessel.
opportunity to defend himself. Petitioners curtly Clearly, the captain is vested with both
dismissed him from his command and summarily management and fiduciary functions.
ordered his repatriation to the Philippines without
informing him of the charge or charges against him, - Indeed, if the ship captain is convinced, as a
and much less giving him a chance to refute any reasonably prudent and competent mariner acting
such charge. In fact, it was only 2 months after his in good faith that the shipowner's or ship agent's
repatriation that Captain Tayong received a instructions (insisted upon by radio or telefax from
telegram dated 24 October 1989 from Inter-Orient their officers thousand of miles away) will result, in
requiring him to explain why he delayed sailing to the very specific circumstances facing him, in
South Africa. imposing unacceptable risks of loss or serious
danger to ship or crew, he cannot casually seek
Page 35 of 5
absolution from his responsibility, if a marine
casualty occurs, in such instructions. 23 NLRC: reversed the LA and ruled that the
circumstances of the case showed that they were
- Compagnie de Commerce v. Hamburg: xxx only project employees of Viva.
where by the force of circumstances, a man has
the duty cast upon him of taking some action for ISSUES:
another, and under that obligation adopts a course - WON Maraguinot and Enero are
which, to the judgment of a wise and prudent man, employees of Viva.
is apparently the best for the interest of the persons
for whom he acts in a given emergency, it may - WON they were illegally dismissed.
properly be said of the course so taken that it was
in a mercantile sense necessary to take it." HELD:
- They were regular employees.
- ON management prerogative: that prerogative is
- They were illegally dismissed.
nevertheless not to be exercised, in the case at bar,
at the cost of loss of Captain Tayong's rights under RATIO:
his contract with petitioner's and under Philippine ISSUE 1
law. - Viva claims that the producers were job
Disposition petitioners having failed to show grave contractors.
abuse of discretion amounting to loss or excess of
jurisdiction on the part of the NLRC in rendering its o However, under Section 8 of Rule
assailed decision, the Petition for Certiorari is hereby VIII, Book III of the Omnibus Rules
DISMISSED, for lack of merit. Costs against Implementing the Labor Code,
petitioners. to be considered a job
contractor, such associate
MARAGUINOT VS. NLRC producers must have tools,
Davide, Jr., J. | January 22, 1998 equipment, machinery, work
premises and other materials
FACTS:Alejandro Maraguinot (Maraguinot) alleges necessary to make motion
that he was employed by Viva Films (Viva) as part picture. The associate producers
of the filming crew. He was later designated as had none of these, and that in
Assistant Electrician and then later promoted to fact, the movie making
Electrician. equipment is owned by Viva.

Paulino Enero (Enero) likewise claims that Viva hired o Given that, these producers can
him as a member of the shooting crew. be considered only as labor-only
contractors. As such is prohibited,
Maraguinot and Eneros tasks consisted of loading, the law considers the person or
unloading and arranging movie equipment in the entity engaged in the same a
shooting area. mere agent or intermediary of
the direct employer.
They later asked the company that their salaries be
- BUT EVEN GIVEN THAT, these producers
adjusted in accordance with the minimum wage
cannot be considered as job
law. In response, the company said that they would
contractors, much less labor-only
grant the adjustment provided they signed a blank
contractors as they did not supply,
employment contract. When they refused, they
recruit nor hire the workers. In this case, it
were forced to go on leave. Upon his return, the
was Viva who recruited the crew
company refused to take Enero back. As regards
members from an available groups of
Maraguinot, he was dropped from the company
freelance workers which include the
payroll, but was later returned. When again he
complainants.
refused to sign the blank contract, his services were
terminated. - The relationship between Viva and its
producers seem to be that of agency as
Maraguinot and Enero then sued for illegal the latter makes movies on behalf of
dismissal. Viva whose business is that of making
movies.
VIVA CLAIMS that they contract persons called
producers/assistant producers to make movies - The existence of an employer-employee
and contend that Maraguinot and Enero are relationship between Maraguinot &
project employees of these producers who act as Enero and Viva is further supported by
independent contractors. Hence there is no the following:
employer-employee relationship between them. In
addition, Viva claims that Maraguinot was hired for o The four elements under 4-fold
the movie Mahirap Maging Pogi, while Enero was test are present.
hired for the movie Sigaw ng Puso.
o CONTROL: Viva has a Supervising
LABOR ARBITER: ruled in favor of Maraguinot and Producer that monitors the
Enero and held that they were employees of Viva progress of the producers. Viva,
and as such were illegally dismissed by the latter. in effect, controls the outcome
Page 36 of 5
of the film and the means (completion of the project) is not one of
through which it is produced. the valid causes for termination under
Article 282 of the Labor Code.
o SELECTION AND SUPERVISION:
Viva issued appointment slips Audion Electric vs NLRC
with their corporate name as the 308 SCRA 340
heading.
Hiring Extend Period
o SALARIES: It was likewise Viva
who paid the employees FACTSFrom the position paper and affidavit
salaries. corroborated by oral testimony, it appears that
complainant was employed by respondent Audion
ISSUE 2
Electric Company on June 30, 1976 as fabricator
- While Maraguinot and Enero were
and continuously rendered service assigned in
possibly initially hired as project
different offices or projects as helper electrician,
employees, they had attained the status
stockman and timekeeper. He as rendered thirteen
of regular employees.
(13) years of continuous, loyal and dedicated
- A project employee or a member of a service with a clean record.
work pool may acquire the status of a
regular employee when the ff. concur: On August 3, complainant was surprised to receive
a letter informing him that he will be considered
o There is a continuous rehiring of terminated after the turnover of materials, including
project employees even after respondents, tools and equipment not later than
cessation of the project. August 15, 1989.

o The tasks performed are vital, Complainant claims that he was dismissed without
necessary and indispensable to justifiable cause and due process and that his
the usual business or trade of the dismissed was done in bad faith which renders the
employer. dismissal illegal. For this reason, he claims that he is
entitled to reinstatement with full backwages. He
- The length of time during which the also claims that he is entitled to moral and
employee was continuously rehired is exemplary damages. He includes payment of his
not controlling, but merely serves as a overtime pay, project allowance, minimum wage
badge of regular employment. increase adjustment, proportionate 13th month pay
and attorney's fees.
- In this case, Enero was employed for 2
years and engaged in at least 18
ISSUE Whether the extended hiring of an employee
projects; while Maragunot was
after the termination of the project makes said
employed for 3 years and worked on at
employee a regular and no longer a project
least 23 projects.
employee.
- Citing Lao vs. NLRC, the could held that
a work pool may exist although the RULING We have held that where the employment
workers in the pool do not receive of project employees is extended long after the
salaries and are free to seek other supposed project has been finished, the employees
employment during temporary breaks in are removed from the scope of project employees
the business, provided that the workers and considered regular employees.
shall be available when called to report Private respondent had presented substantial
for a project. Although primarily evidence to support his position, while petitioner
applicable to regular seasonal workers, merely presented an unverified position paper
this setup can likewise be applied to merely stating therein that private respondent has
project workers in so far as the effect of no cause to complain since the employment
temporary cessation of work is contract signed by private respondent with
concerned. petitioner was co-terminus with the project.
Notably, petitioner failed to present such
- Once a project or work pool employee employment contract for a specific project signed
has been (a) continuously, as opposed by private respondent that would show that his
to intermittently, rehired by the same employment with the petitioner was for the duration
employer for the same tasks or nature of of a particular project. Moreover, notwithstanding
tasks; (b) these tasks are vital, necessary, petitioner's claim in its reply that in taking interest in
and indispensable to the usual business the welfare of its workers, petitioner would strive to
or trade of the employer, then the provide them with more continuous work by
employee must be deemed a regular successively employing its workers, in this case,
employee, pursuant to Article 280 of the private respondent, petitioner failed to present any
Labor Code and jurisprudence. report of termination.

- As Maraguinot and Enero have already As stated earlier, the rule in our jurisdiction is that
gained the status of regular employees, findings of facts of the NLRC affirming those of the
their dismissal was unwarranted since Labor Arbiter are entitled to great weight and will
the cause invoked for their dismissal not be disturbed if they are supported by
Page 37 of 5
substantial evidence. Substantial evidence is an of the employer. The test is whether the former is
amount of relevant evidence which a reasonable usually necessary or desirable in the usual business
mind might accept as adequate to justify a or trade of the employer. The connection can be
conclusion. We find no grave abuse of discretion determined by considering the nature of work
committed by NLRC in finding that private performed and its relation to the scheme of the
respondent was not a project employee. particular business or trade in its entirety. Also, if the
employee has been performing the job for at least
WHEREFORE, the challenged resolutions of the a year, even if the performance is not continuous
respondent NLRC are hereby AFFIRMED with the and merely intermittent, the law deems repeated
MODIFICATION that the awards of moral and and continuing need for its performance as
exemplary damages and attorney's fees are sufficient evidence of the necessity if not
DELETED. indispensability of that activity to the business.
Hence, the employment is considered regular, but
only with respect to such activity and while such
activity exists.

UNIVERSAL ROBINA CORPORATION vs CATAPANG It is obvious that the said five-month contract of
[G.R. No. 164736 October 14, 2005] employment was used by petitioners as a
convenient subterfuge to prevent private
FACTS:Petitioner Universal Robina Corporation is a respondents from becoming regular employees.
corporation duly organized and existing under the Such contractual arrangement should be struck
Philippine laws, while petitioner Randy Gregorio is down or disregarded as contrary to public policy or
the manager of the petitioner companys duck farm morals. To uphold the same would, in effect, permit
in Calauan, Laguna. The individual respondents petitioners to avoid hiring permanent or regular
were hired by the petitioner company on various employees by simply hiring them on a temporary or
dates from 1991 to 1993 to work at its duck farm casual basis, thereby violating the employees
in Barangay Sto. Tomas, Calauan, Laguna. The security of tenure in their jobs. Petitioners act of
respondents were hired under an employment repeatedly and continuously hiring private
contract which provided for a five-month period. respondents in a span of 3 to 5 years to do the
After the expiration of the said employment same kind of work negates their contention that
contracts, the petitioner company would renew private respondents were hired for a specific
them and re-employ the respondents. This practice project or undertaking only.
continued until sometime in 1996, when the
petitioners informed the respondents that they were
no longer renewing their employment contracts. [G.R. No. 114734. March 31, 2000]
When their contract was no longer renewed in
1996, respondents filed complaints for illegal
VIVIAN Y. IMBUIDO, petitioner, vs. NATIONAL LABOR
dismissal, reinstatement, backwages, damages and
RELATIONS COMMISSION, INTERNATIONAL
attorneys fees against URC.
INFORMATION SERVICES, INC. and GABRIEL
Labor Arbiter ruled that Respondents are regular
LIBRANDO,respondents.
employees and were illegally dismissed. LA ordered
for their reinstatement. Several Writs of Execution
were issued by the LA enforcing the immediate Facts:Petitioner was employed as a data encoder
reinstatement of the illegally dismissed respondents. by private respondent International Information
URC was not able to fully comply with the order Services, Inc., a domestic corporation engaged in
because the section to which some of the the business of data encoding and keypunching,
respondents were assigned was abolished. from August 26, 1988 until October 18, 1991 when
Furthermore, NLRC affirmed the decision of the her services were terminated. From August 26, 1988
Labor Arbiter. Upon appeal, the CA ruled that until October 18, 1991, petitioner entered into
Respondents are regular employees after rendering thirteen (13) separate employment contracts with
more than one year of continuous service. The 5- private respondent, each contract lasting only for a
month contract is contrary to public policy for period of three (3) months.
being used as a subterfuge to prevent the
respondents from becoming regular employees. In September 1991, petitioner and twelve (12) other
URC should have included the respondents in their employees of private respondent allegedly agreed
payroll even if reinstatement cannot be to the filing of a petition for certification election
accommodated. URC filed a motion for involving the rank-and-file employees of private
reconsideration of the CA decision. CA denied the respondent.[3] Thus, on October 8, 1991, Lakas
motion for being filed 2 days late. Manggagawa sa Pilipinas (LAKAS) filed a petition
ISSUE: Whether or not respondents are regular for certification election with the Bureau of Labor
employees of petitioner corporation. Relations (BLR), docketed as NCR-OD-M-9110-128.[4]

HELD: The SC held that the CA, the NLRC and the Subsequently, on October 18, 1991, petitioner
Labor Arbiter correctly categorized the respondents received a termination letter from Edna Kasilag,
as regular employees of the petitioner company. Administrative Officer of private respondent,
The primary standard of determining regular allegedly "due to low volume of work."[5]Thus, on
employment is the reasonable connection May 25, 1992, petitioner filed a complaint for illegal
between the particular activity performed by the dismissal with prayer for service incentive leave pay
employee in relation to the usual trade or business
Page 38 of 5
and 13th month differential pay, with the National Issue: WON Vivian is a project employee.
Labor Relations Commission
Held: Yes. We agree with the findings of the NLRC
Petitioner alleged that her employment was that petitioner is a project employee. The principal
terminated not due to the alleged low volume of test for determining whether an employee is a
work but because she "signed a petition for project employee or a regular employee is whether
certification election among the rank and file the project employee was assigned to carry out a
employees of respondents," thus charging private specific project or undertaking, the duration and
respondent with committing unfair labor practices scope of which were specified at the time the
employee was engaged for that project. A project
On the other hand, private respondent, in its employee is one whose employment has been
position paper filed on July 16, 1992, maintained fixed for a specific project or undertaking, the
that it had valid reasons to terminate petitioners completion or termination of which has been
employment and disclaimed any knowledge of the determined at the time of the engagement of the
existence or formation of a union among its rank- employee or where the work or service to be
and-file employees at the time petitioners services performed is seasonal in nature and the
were terminated.[8] Private respondent stressed that employment is for the duration of the season. In the
its business "relies heavily on companies availing instant case, petitioner was engaged to perform
of its services. Its retention by client companies with activities which were usually necessary or desirable
particular emphasis on data encoding is on a in the usual business or trade of the employer, as
project to project basis,"[9] usually lasting for a admittedly, petitioner worked as a data encoder
period of "two (2) to five (5) months." Private for private respondent, a corporation engaged in
respondent further argued that petitioners the business of data encoding and keypunching,
employment was for a "specific project with a and her employment was fixed for a specific
specified period of engagement. project or undertaking the completion or
termination of which had been determined at the
In his decision, the labor arbiter found petitioner to time of her engagement, as may be observed from
be a regular employee, ruling that even if herein the series of employment contracts between
complainant [petitioner herein] had been petitioner and private respondent, all of which
obstensively hired for a fixed period or for a specific contained a designation of the specific job
undertaking, she should be considered as a regular contract and a specific period of employment.
employee of the respondents in conformity with the
provisions laid down under Article 280 of the Labor However, even as we concur with the NLRC's
Code,"[14] after finding that "[i]t is crystal clear that findings that petitioner is a project employee, we
herein complainant [petitioner herein] performed a have reached a different conclusion. In the recent
job which are (sic) usually necessary or desirable in case of Maraguinot, Jr. vs. NLRC, we held that "[a]
the usual business of respondent [s]."[15] The labor project employee or a member of a work pool may
arbiter further denounced "the purpose behind acquire the status of a regular employee when the
the series of contracts which respondents required following concur:
complainant to execute as a condition of
employment was to evade the true intent and spirit 1) There is a continuous rehiring of project
of the labor laws for the employees even after [the] cessation of a
workingmen."[16] Furthermore, the labor arbiter project; and
concluded that petitioner was illegally dismissed
because the alleged reason for her termination, 2) The tasks performed by the alleged
that is, low volume of work, is "not among the just "project employee" are vital, necessary and
causes for termination recognized by indispensable to the usual business or trade
law,"[17] hence, he ordered her immediate of the employer.
reinstatement without loss of seniority rights and with
full backwages. With regard to the service incentive The evidence on record reveals that petitioner was
leave pay, the labor arbiter decided "to grant the employed by private respondent as a data
same for failure of the respondents to fully encoder, performing activities which are usually
controvert said claims."[18] Lastly, the labor arbiter necessary or desirable in the usual business or trade
rejected petitioners claim for 13th month pay of her employer, continuously for a period of more
"since complainant [petitioner herein] failed to than three (3) years, from August 26, 1988 to
fully substantiate and argued (sic) the same October 18, 1991 36 and contracted for a total of
thirteen (13) successive projects. We have
On appeal, the NLRC reversed the decision of the previously ruled that "[h]owever, the length of time
labor arbiter. The NLRC held that the during which the employee was continuously re-
complainant [petitioner herein], while hired as a hired is not controlling, but merely serves as a
regular worker, is statutorily guaranteed, in her badge of regular employment." 37Based on the
tenurial security, only up to the time the specific foregoing, we conclude that petitioner has
project for which she was hired is attained the status of a regular employee of private
completed."[23] Hence, the NLRC concluded that respondent.
"[w]ith the specific project "at RCBC 014" admittedly
completed, the complainant [petitioner herein] has The Court's ruling here is meant precisely to give life
therefore no valid basis in charging illegal dismissal to the constitutional policy of strengthening the
for her concomittant (sic) dislocation labor sector, but, we stress, not at the expense of
Page 39 of 5
management. Lest it be misunderstood, this ruling In G.R. No. 65689, Rogelio Diamante, Manuel
does not mean that simply because an employee is Pacres, Macario Saputalo, Rolando Cervales and
a project or work pool employee even outside the Dionisio Cervales were assigned to the construction
construction industry, he is deemed, ipso jure, a of the LCT Catarman, Project No. 7511. After three
regular employee. All that we hold today is that months of work, the project was completed on July
once a project or work pool employee has 26, 1979. The five workers were served a termination
been: (1) continuously, as opposed to intermittently, notice. The termination was reported to the Ministry
re-hired by the same employer for the same tasks or of Labor on August 3, 1979. They filed a complaint
nature of tasks; and (2) these tasks are vital, for illegal dismissal.
necessary, and indispensable to the usual business
or trade of the employer, then the employee must The National Labor Relations Commission affirmed
be deemed a regular employee, pursuant to Article the decision of the Labor Arbiter ordering the
280 of the Labor Code and jurisprudence. To rule reinstatement of the five complainants with
otherwise would allow circumvention of labor laws backwages from July 27, 1979.
in industries not falling within the ambit of Policy
Instruction No. Policy Department Order No. 19, In G.R. No. 66119, respondents Danilo de la Cruz, et
hence allowing the prevention of acquisition of al., 17 in all, were assigned to work in Project No.
tenurial security by project or work pool employees 7901 for the construction of a tanker ordered by
who have already gained the status of regular Mobil Oil Philippines, Inc. There were 55 workers in
employees by the employer's conduct. 39 (emphasis that project. The tanker was launched on January
supplied) 31, 1980. Upon the yard manager's
recommendation, the personnel manager of
Being a regular employee, petitioner is entitled to Sandoval Shipyards terminated the services of the
security of tenure and could only be dismissed for a welders, helpers and construction workers effective
just or authorized cause, as provided in Article 279 February 4, 1980. The termination was duly reported
of the Labor Code. The alleged causes of to the Ministry of Labor and Employment.
petitioner's dismissal (low volume of work and
belatedly, completion of project) are not valid Three days later, or on February 7, twenty-seven out
causes for dismissal under Articles 282 and 283 of of the 55 workers were hired for a new project. The
the Labor Code. Thus, petitioner is entitled to 27 included four of the 17 respondents who filed a
reinstatement without loss of seniority rights and complaint for illegal dismissal on February 6.
other privileges, and to her full backwages, inclusive
of allowances, and to her other benefits or their
After hearing, the Director of the Ministry's Capital
monetary equivalent computed from the time her
Region ordered the reinstatement of the
compensation was withheld from her up to the time
complainants. The Deputy Minister of Labor
of her actual reinstatement. However, complying
affirmed that order.
with the principles of "suspension of work" and "no
work, no pay" between the end of one project and
the start of a new one, in computing petitioner's
backwages, the amounts corresponding to what
could have been earned during the periods from Issue: WON Respondents are a project employee.
the date petitioner was dismissed until her
reinstatement when private respondent was not Held: We hold that private respondents were
undertaking any project, should be deducted. project employees whose work was coterminous
with the project for which they were hired. Project
employees, as distinguished from regular or non-
G.R. No. L-65689 May 31, 1985
project employees, are mentioned in section 281 of
the Labor Code as those "where the employment
SANDOVAL SHIPYARDS, INC., petitioner, vs. has been fixed for a specific project or undertaking
NATIONAL LABOR RELATIONS COMMISSION, the completion or termination of which has been
ROGELIO DIAMANTE, MANUEL PACRES, ROLANDO determined at the time of the engagement of the
CERVALES, DIONISIO CERVALES and MACARIO employee."
SAPUTALO, respondents.
Policy Instructions No. 20 of the Secretary of Labor,
G.R. No. L-66119 May 31, 1985 which was issued to stabilize employer-employee
relations in the construction industry, provides:
Facts: These cases are about the dismissal of
alleged project workers. Sandoval Shipyards, Inc. Project employees are those
has been engaged in the building and repair of employed in connection with a
vessels. It contends that each vessel is a separate particular construction project. Non-
project and that the employment of the workers is project (regular) employees are
terminated with the completion of each project. those employed by a construction
company without reference to any
The workers contend otherwise. They claim to be particular project.
regular workers and that the termination of one
project does not mean the end of their Project employees are not entitled
employment since they can be assigned to to termination pay if they are
unfinished projects. terminated as a result of the

Page 40 of 5
completion of the project or any right to self-organization, (c) nonregularization of
phase thereof in which they are contractual employees, (d) illegal termination of
employed, regardless of the number employees, (e) nonpayment of wage/benefit
of projects in which they have been differentials, and (f) nonrecognition of NSCEASPFL
employed by a particular as the sole bargaining representative of the
construction company. Moreover, company. The petitioner-workers continually lost in
the company is not required to the case with respect to regularization (they were
obtain clearance from the Secretary awarded wage increases) from the Labor Minister
of Labor in connection with such to the NLRC despite numerous MRs. The initial
termination. proceedings (1986) simply ordered NSC to submit
their complete records for the tribunals to be able
It is significant to note that the corporation does not to make a definite ruling, in the meantime making a
construct vessels for sale or otherwise which will provisional ruling that the petitioners were project
demand continuous productions of ships and will employees. The correctness of this order was
need permanent or regular workers. It merely affirmed by the SC in an earlier case and
accepts contracts for ship-building or for repair of remanded it back to the NLRC. This decision by the
vessels from third parties and, only, on occasion NLRC (1992) affirmed their earlier resolution that the
when it has work contract of this nature that it hires petitioners were merely project employees. In the
workers to do the job which, needless to say, lasts dispositive, it said that the individual rulings as to
only for less than a Year or longer. which petitioner is contractual and which is regular,
will be the subject of future proceedings. A break-
The completion of their work or away group from the former group of petitioners
project automatically terminates their employment, MR this with the NLRC twice before raising the
in which case, the employer is, under the law, only present case to the SC for grave abuse of
obliged to render a report on the termination of the discretion. Note that if declared to be regular, the
employment. (139-140, Rollo of G. R. No. 65689). petitioners would be reinstated.

Issue: WON Petitioners are project employees.

[G.R. No. 117043. January 14, 1998] Held: Yes.SC relies on the factual findings of labor
administrative tribunals like the NLRC which have
acquired expertise because their jurisdiction is
confined to specific matters. In its Decision and
FELIX VILLA, vs. NATIONAL LABOR RELATIONS Resolutions, the NLRC adamantly held that
COMMISSION, Fifth Division and NATIONAL petitioners were contractual project employees
STEEL CORPORATION, respondents. who are not entitled to regularization under Art. 280.
280 conceives of three kinds of employees: regular
Facts: Respondent National Steel Corporation employees or those who have been engaged to
(NSC), one of the biggest modern steel mills in perform activities which are usually necessary or
Southeast Asia, produces hot rolled products, cold desirable in the usual business or trade of the
rolled products, tinplates and billets. These products employer; project employees or those whose
are in turn transformed by downstream industries employment has been fixed for a specific project or
into truss assemblies, farm implements, pipe undertaking the completion or termination of which
structures, shipbuilding and repairing materials, has been determined at the time of the
automotive structures and machine parts, GI roof engagement of the employee or where the work or
sheets or galvanized iron, drums, nails, fasteners and services to be performed is seasonal in nature and
wires.[1] the employment is for the duration of the season,
and casual employees or those who are neither
National Steel Corporation (NSC) in regular nor project employees.
Cagayan de Oro, in the first part of an expansion In the earlier case with the same parties,
program, constructed a Billet Steelmaking Plant. the Court distinguished between two types of
The plan was to first use scrap metal by projects: a project could refer to a particular job or
shipbreaking decommissioned ships before undertaking that is within the regular or usual
converting to the plant to a Direct Reduced Iron business of the employer company, but which is
plant. A billet is a semi-finished steel product later distinct and separate, and identifiable as such, from
processed into steel bars and wire rods. the other undertakings of the company.
Shipbreaking means cutting up/salvaging ships and (construction company that may undertake two or
reprocessing the steel. However, after four or five more projects at the same time in different
ships, the project was phased out because of the places) a project may refer to a particular job or
scarcity of vessels/ships for salvaging, the higher undertaking that is not within the regular business of
costs of operation, and the unsuitability of raw the corporation. Such a job or undertaking must
materials. It was not a viable undertaking. The also be identifiably separate and distinct from the
petitioners are the former employees of the plant, ordinary or regular business operations of the
being masons, carpenters, laborers, electricians, employer. The job or undertaking also begins and
and painters. Even before the plant was phased ends at determined or determinable times. (instant
out, there were already many labour problems. case; expansion program being different
Notably, there was a strike complaining of (a) wage component projects a distinct undertaking
discrimination, (b) interference with the employees identifiable from the ordinary business and activity
of NSC)
Page 41 of 5
The Supreme Court disposed of the including the penalty imposed for late premium
regularization issue by reasoning that though: The remittances.
records show that the petitioners were utilized in
operations other than billet making or other On 01 February 1995, the SSC issued
components of the expansion porgram, such as its Order11 which ruled in favor of private
shipbreaking. And while it is true that they respondents. The SSC, relying on NLRC Case No.
performed other activities which were necessary or RAB-III-8-2373-85,12 declared private respondents to
desirable in the usual business of the NSC And that be petitioners regular employees.13 It ordered
the duration of their employment was for a period petitioner to pay the SSS the unpaid SS/EC and
of more than one year. These factors did not make Medicare contributions plus penalty for the delayed
them regular because: They remain project remittance thereof, without prejudice to any other
employees regardless of the number of projects in penalties which may have accrued.14 The SSC
which they have worked. Length of service is not denied the Motion for Reconsideration15 of
the controlling determinant of the employment petitioner for lack of merit.16
tenure of a project employee. In the case of
Mercado, Sr. v. NLRC, it was ruled that the proviso in Petitioner elevated the matter to the Court of
the second paragraph of Article 280, providing that Appeals via a Petition for Review.17 He claimed that
an employee who has served for at least one year, private respondents were project employees,
shall be considered a regular employees, relates whose periods of employment were terminated
only to casual employees and not to project upon completion of the project. Thus, he claimed,
employees. no employer-employee relation existed between
the parties.18 There being no employer-employee
relationship, private respondents are not entitled to
G.R. No. 125837 October 6, 2004 coverage under the Social Security Act.19 In
addition, petitioner claimed that private
REYNALDO CANO CHUA, doing business under the respondents length of service did not change their
name & style PRIME MOVER CONSTRUCTION status from project to regular employees. Petitioner
DEVELOPMENT, petitioner, vs. COURT OF APPEALS, also claimed that the case has prescribed. The
SOCIAL SECURITY COMMISSION, SOCIAL SECURITY Court of Appeals ruled in favor of the private
SYSTEM, ANDRES PAGUIO, PABLO CANALE, RUEL respondents.
PANGAN, AURELIO PAGUIO, ROLANDO TRINIDAD,
ROMEO TAPANG and CARLOS The Court of Appeals, citing Article 280 of the Labor
MALIWAT, respondents. Code,26 declared that private respondents were all
regular employees of the petitioner in relation to
Facts: On 20 August 1985, private respondents certain activities since they all worked either as
Andres Paguio, Pablo Canale, Ruel Pangan, Aurelio masons, carpenters and fine graders in petitioners
Paguio, Rolando Trinidad, Romeo Tapang and various construction projects for at least one year,
and that their work was necessary and desirable to
Carlos Maliwat (hereinafter referred to as
petitioners business which involved the
respondents) filed a Petition4 with the SSC for SSS
construction of roads and bridges. The Court of
coverage and contributions against petitioner Appeals rejected the claim of prescription, stating
Reynaldo Chua, owner of Prime Mover that the filing of private respondents claims was
Construction Development, claiming that they were well within the twenty (20)-year period provided by
all regular employees of the petitioner in his the Social Security Act. Petitioner filed a Motion for
construction business.5 Reconsideration,32 claiming that the Court of
Appeals overlooked (1) the doctrine that length of
Private respondents claimed that they were service of a project employee is not the controlling
assigned by petitioner in his various construction test of employment tenure, and (2) petitioners
projects continuously in the following capacity, failure to place private respondents under SSS
since the period indicated, and with the coverage was in good faith. The motion was
corresponding basic salaries denied for lack of merit. Hence, this petition.

Private respondents alleged that petitioner Issue: WON private respondents are project
dismissed all of them without justifiable grounds, emplyees.
and without notice to them and to the then Ministry
of Labor and Employment. They further alleged that Held: No.There is no dispute that private
petitioner did not report them to the SSS for respondents were employees of petitioner.
compulsory coverage in flagrant violation of the Petitioner himself admitted that they worked in his
Social Security Act. construction projects,38 although the period of their
employment was allegedly co-terminus with their
Meanwhile, the SSS filed a Petition in phase of work.39 Even without such admission from
Intervention10 alleging that it has an interest in the petitioner, the existence of an employer-employee
petition filed by private respondents as it is charged relationship between the parties can easily be
with the implementation and enforcement of the determined by the application of the "control
provisions of the Social Security Act. The SSS stated test,"40 the elements of which are enumerated
that it is the mandatory obligation of every above. It is clear that private respondents are
employer to report its employees to the SSS for employees of petitioner, the latter having control
coverage and to remit the required contribution, over the results of the work done, as well as the

Page 42 of 5
means and methods by which the same were employed.49 Moreover, it is peculiar that petitioner
accomplished. Suffice it to say that regardless of did not show proof that he submitted reports of
the nature of their employment, whether it is regular termination after the completion of his construction
or project, private respondents are subject of the projects, considering that he alleges that private
compulsory coverage under the SSS Law, their respondents were hired and rehired for various
employment not falling under the exceptions projects or phases of work therein.
provided by the law.41 This rule is in accord with the
Courts ruling in Luzon Stevedoring Corp. v. SSS42 to
the effect that all employees, regardless of tenure,
would qualify for compulsory membership in the [G.R. No. 116352. March 13, 1997]
SSS, except those classes of employees
contemplated in Section 8(j) of the Social Security
Act.43
J. & D.O. AGUILAR CORPORATION, petitioner, vs.
This Court also finds no reason to deviate from the NATIONAL LABOR RELATIONS COMMISSION and
finding of the Court of Appeals regarding the ROMEO ACEDILLO, respondents.
nature of employment of private respondents.
Despite the insistence of petitioner that they were
project employees, the facts show that as masons,
carpenters and fine graders in petitioners various Facts: This petition for certiorari is questioning the
construction projects, they performed work which decision of respondent National Labor Relations
was usually necessary and desirable to petitioners Commission dated March 30, 1994, as well as its
business which involves construction of roads and resolution of June 20, 1994, denying for lack of merit
bridges. In Violeta v. NLRC,44 this Court ruled that to petitioner's motion for reconsideration of said
be exempted from the presumption of regularity of decision.
employment, the agreement between a project
employee and his employer must strictly conform to Private respondent Romeo Acedillo began
the requirements and conditions under Article 280 working for petitioner in February 1989 as a helper-
of the Labor Code. It is not enough that an electrician. On January 16, 1992, he received a
employee is hired for a specific project or phase of letter from petitioner informing him of his severance
work. There must also be a determination of, or a from the company allegedly due to lack of
clear agreement on, the completion or termination available projects and excess in the number of
of the project at the time the employee was workers needed. He decided to file a case for
engaged if the objectives of Article 280 are to be illegal dismissal before the NLRC after learning that
achieved.45 This second requirement was not met in new workers were being hired by petitioner while his
this case. request to return to work was being ignored. In
reply, petitioner maintained that its need for workers
Moreover, while it may be true that private varied, depending on contracts procured in the
respondents were initially hired for specific projects course of its business of contracting refrigeration
or undertakings, the repeated re-hiring and and other related works. It contended that its
continuing need for their services over a long span workers are hired on a contractual or project basis,
of timethe shortest being two years and the and their employment is deemed terminated upon
longest being eighthave undeniably made them completion of the project for which they were
regular employees.46 This Court has held that an hired. Finally, petitioner argued that Acedillo was
employment ceases to be co-terminus with specific not a regular employee because his employment
projects when the employee is continuously rehired was for a definite period and apparently made only
due to the demands of the employers business to augment the regular work force.
and re-engaged for many more projects without
On June 17, 1993, Labor Arbiter Arthur L.
interruption.47 The Court likewise takes note of the
Amansec rendered judgment declaring Acedillo's
fact that, as cited by the SSC, even the National
dismissal to be illegal, finding him to be a member
Labor Relations Commission in a labor case
of the regular work pool, and ordering petitioner to
involving the same parties, found that private
pay him a total of P71,906.00 representing
respondents were regular employees of the
backwages, 13th month pay, separation pay in lieu
petitioner.48
of reinstatement, service incentive leave pay and
underpayment of wages.
Another cogent factor militates against the
allegations of the petitioner. In the proceedings On appeal, the NLRC affirmed Labor Arbiter
before the SSC and the Court of Appeals, petitioner Amansec's decision.
was unable to show that private respondents were
appraised of the project nature of their Issue: WON private respondent is a regular
employment, the specific projects themselves or employee.
any phase thereof undertaken by petitioner and for
which private respondents were hired. He failed to Held: Yes. While respondent (herein petitioner)
show any document such as private respondents strongly maintains that complainant (Acedillo) was
employment contracts and employment records not a regular worker, however, the nature of his job
that would indicate the dates of hiring and as a helper and the length of service that he had
termination in relation to the particular construction been with respondent are clear proof(s) that he
project or phases in which they were was a regular employee. For what determines

Page 43 of 5
whether a certain employment is regular or casual Acedillo is not a regular employee. As we held in
is not the will and word of the employer, to which the case of Philippine National Construction
the desperate worker often accedes, much less the Corporation v. NLRC,[5]
procedure of hiring the employee or the manner of
"Members of a work pool from which a
praying (sic) his salary. It is the nature of the
construction company draws its project employees,
activities performed in relation to the particular
if considered employees of the construction
business or trade (of the employer) considering all
company while in the work pool, are non-project
circumstances, and in some cases the length of
employees or employees for an indefinite period. If
time of its performance and its continued
they are employed in a particular project, the
existence.[1] x x x Suffice it to state that respondent's
completion of the project or any phase thereof will
contention to show that he (Acedillo) was (a)
not mean severance of (the) employer-employee
regular employee will not prosper because in the
relationship.
absence of any proof that he was hired on a
project or contractual basis, the valid presumption WHEREFORE, in view of the foregoing, the
is that the employment is regular. instant petition for certiorari is DISMISSED. Costs
against petitioner.
Likewise, respondent's allegation that since
complainant was not a regular employee, he was SO ORDERED.
not entitled to any of his monetary claims must
fail. Respondent must be reminded that payment of Workpool Employees: MARAGUINOT vs. NLRC (DEL
these benefits in accordance with law does not
ROSARIO, VIVA FILMS)
depend on whether the worker is a regular, casual
284 SCRA 539
or project worker. It is bound to comply with the
DAVIDE; January 22, 1998
basic requirement of (labor standards) law on
NATURE: Special civil action for certiorari seeking to
payment of (wages and other benefits) and proof
annul the decision of NLRC and its Resolution
of payment or non-payment thereof rests on the
respondent. It must be so because where the issue
FACTS: Petitioner Alejandro Maraguinot, Jr.
is the payment of labor standards benefits, burden
maintains that he was employed by private
of proof is on the employer, not on the employees
respondents as part of the filming crew. About 4
because the latter are neither required nor months later, he was designated Asst. Electrician.
expected to keep records of payment or non-
He was then promoted to the rank of Electrician.
payment of benefits granted to them by law."
Petitioner Paulino Enero claims that private
respondents employed him as a member of the
Petitioner is to be reminded that a project shooting crew.
employee is one whose "employment has been Petitioners tasks consisted of loading, unloading
fixed for a specific project or undertaking, the and arranging movie equipment in the shooting
completion or termination of which has been area as instructed by the cameraman, returning
determined at the time of the engagement of the the equipment to Viva Films warehouse, assisting in
employee or where the work or services to be the fixing of the lighting system, and performing
performed is seasonal in nature and the other tasks that the cameraman and/or director
employment is for the duration of the season." [2] The may assign.
records reveal that petitioner did not specify the
duration and scope of the undertaking at the time Petitioners requested that private respondents
Acedillo's services were contracted. Petitioner adjust their salary in accordance with the minimum
could have easily presented an employment wage law. Petitioners were informed that Mr. Vic
contract showing that he was engaged only for a del Rosario would agree to increase their salary only
specific project, but it failed to do so. It is not even if they signed a blank employment contract. As
clear if Acedillo ever signed an employment petitioners refused to sign, private respondents
contract with petitioner. Neither is there any proof forced Enero to go on leave then refused to take
that the duration of his assignment was made clear him back when he reported for work. Meanwhile,
to him other than the self-serving assertion of Maraguinot was dropped from the company
petitioner that the same can be inferred from the payroll but was returned and again asked to sign a
tasks he was made to perform. blank employment contract, and when he still
What is clear is that Acedillo's work as a helper- refused, private respondents terminated his
electrician was an activity "necessary or desirable in services. Petitioners thus sued for illegal dismissal
the usual business or trade"[3] of petitioner, since before the Labor Arbiter.
refrigeration requires considerable electrical work.
This necessity is further bolstered by the fact that Private respondents claim that Viva Films is primarily
petitioner would hire him anew after the engaged in the distribution and exhibition of
completion of each project, a practice which movies, but not in the business of making movies; in
persisted throughout the duration of his tenure. the same vein, private respondent Vic del Rosario is
merely an executive producer, i.e., the financier
The petitioner admits that it maintains two sets who invests a certain sum of money for the
of workers, viz., those who are permanently production of movies distributed and exhibited by
employed and get paid regardless of the VIVA; that they contract persons called
availability of work and those who are hired on a producers -- also referred to as associate
project basis.[4] This practice of keeping a work pool producers-- to produce or make movies for
further renders untenable petitioner's position that private respondents; and that petitioners are

Page 44 of 5
project employees of the associate producers who,
in turn, act as independent contractors. As such, 5) Respondents also alleged that
there is no employer-employee relationship complainants were not prohibited from
between petitioners and private respondents; that working with other movie companies.
it was the associate producer of a film who hired
The NLRC, in reversing the Labor Arbiter, then
Maraguinot.and he was released upon payment of
concluded that these circumstances, taken
his last salary, as his services were no longer
together, indicated that complainants (herein
needed; that Enero was hired for a movie, went on
petitioners) were project employees.
vacation and by the time he reported back to work
the move had been completed.
Petitioners Claim To support their claim that they
were regular (and not project) employees of
The Labor Arbiter found that: complainants are the
private respondents, petitioners cited their
employees of the respondents. The producer
performance of activities that were necessary or
cannot be considered as an independent
desirable in the usual trade or business of private
contractor but should be considered only as a
respondents and added that their work was
labor-only contractor and as such, acts as a mere
continuous, i.e., after one project was completed
agent of the real employer, the herein respondents.
they were assigned to another project.
Also, it is an admitted fact that the complainants
received their salaries from the respondents. It is
Respondents Private respondents reiterate their
very clear also that complainants are doing
version of the facts and stress that their evidence
activities which are necessary and essential to the
supports the view that petitioners are project
business of the respondents, that of movie-making.
employees; point to petitioners irregular work load
Complainant Maraguinot worked as an electrician
and work schedule; emphasize the NLRCs finding
while complainant Enero worked as a crew
that petitioners never controverted the allegation
[member]. Hence, the complainants were illegally
that they were not prohibited from working with
dismissed.
other movie companies; and ask that the facts be
viewed in the context of the peculiar characteristics
Private respondents appealed to the NLRC. In its
of the movie industry.
decision, it said that:
1) Complainants were hired for specific movie
The Office of the Solicitor General (OSG) is
projects and their employment was co- convinced that this petition is improper since
terminus with each movie project the petitioners raise questions of fact; and submits that
completion/termination of which are pre- petitioners reliance on Article 280 of the Labor
determined, such fact being made known Code to support their contention that they should
to complainants at the time of their be deemed regular employees is misplaced, as
said section merely distinguishes between two
engagement.
types of employees, i.e., regular employees and
casual employees, for purposes of determining the
2) Each shooting unit works on one movie right of an employee to certain benefits. The OSG
project at a time. And the work of the likewise rejects petitioners contention that since
shooting units, which work independently they were hired not for one project, but for a series
from each other, are not continuous in of projects, they should be deemed regular
nature but depends on the availability of employees. In closing, the OSG disagrees with
movie projects. petitioners claim that the NLRCs classification of
the movie producers as independent contractors
had no basis in fact and in law, since, on the
3) Further shown by respondents is the irregular
contrary, the NLRC took pains in explaining its
work schedule of complainants on a daily basis for its decision.
basis. Maraguinot was supposed to report
on 05 August 1991 but reported only on 30
August 1991, or a gap of 25 days. ISSUE: Whether Or Not petitioners were illegally
dismissed
Complainant Enero worked on 10
September 1991 and his next scheduled HELD: YES. A project employee or a member of a
working day was 28 September 1991, a gap work pool may acquire the status of a regular
of 18 days. employee when the following concur: 1) There is a
continuous rehiring of project employees even after
4) The extremely irregular working days and cessation of a project; and 2) The tasks performed
hours of complainants work explain the by the alleged project employee are vital,
lump sum payment for complainants necessary and indispensable to the usual business
services for each movie project. Hence, or trade of the employer. However, the length of
complainants were paid a standard weekly time during which the employee was continuously
salary regardless of the number of working re-hired is not controlling, but merely serves as a
days and hours they logged in. Otherwise, if badge of regular employment.
the principle of no work no pay was strictly
applied, complainants earnings for certain In the instant case, the evidence on record shows
weeks would be very negligible. that petitioner Enero was employed for a total of

Page 45 of 5
two (2) years and engaged in at least eighteen (18) Nature of Work: A.M. ORETA & CO INC vs. NLRC
projects, while petitioner Maraguinot was (GRULLA)
employed for some three (3) years and worked on 176 SCRA 218
at least twenty-three (23) projects. Moreover, as MEDIALDEA; August 10, 1989
petitioners tasks involved, among other chores, the NATURE: Petition for certiorari
loading, unloading and arranging of movie
equipment in the shooting area as instructed by the FACTS: Private respondent Grulla was engaged by
cameramen, returning the equipment to the Viva Engineering Construction and Industrial
Films warehouse, and assisting in the fixing of the Development Company (ENDECO) through A.M.
lighting system, it may not be gainsaid that these Oreta and Co., Inc. as a carpenter in its project in
tasks were vital, necessary and indispensable to the Jeddah, Saudi Arabia.
usual business or trade of the employer. As regards
the underscored phrase, it has been held that this is The contract of employment, which was entered
ascertained by considering the nature of the work into on June 11, 1980 was for a period of 12 months.
performed and its relation to the scheme of the Respondent Grulla left the Philippines for Jeddah,
particular business or trade in its entirety. Saudi Arabia on August 5, 1980.
It may not be ignored, however, that private
respondents expressly admitted that petitioners On August 15, 1980, Grulla met an accident which
were part of a work pool; and, while petitioners fractured his lumbar vertebrae while working at the
were initially hired possibly as project employees, jobsite. He was rushed to the New Jeddah Clinic
they had attained the status of regular employees and was confined there for 12 days.
in view of VIVAs conduct.
On August 27, 1980, Grulla was discharged from the
At this time, we wish to allay any fears that this hospital and was told that he could resume his
decision unduly burdens an employer by imposing normal duties after undergoing physical therapy for
a duty to re-hire a project employee even after two weeks.
completion of the project for which he was hired.
The import of this decision is not to impose a positive On September 18, 1980, respondent Grulla reported
and sweeping obligation upon the employer to re- back to his Project Manager and presented to the
hire project employees. What this decision merely latter a medical certificate declaring the former
accomplishes is a judicial recognition of the already physically fit for work. Since then, he stated
employment status of a project or work pool working again until he received a notice of
employee in accordance with what is fait termination of his employment on October 9, 1980.
accompli, i.e., the continuous re-hiring by the
employer of project or work pool employees who Grulla filed a complaint for illegal dismissal,
perform tasks necessary or desirable to the recovery of medical benefits, unpaid wages for the
employers usual business or trade. Let it not be unexpired ten (10) months of his contract and the
said that this decision coddles labor, for as Lao1 sum of P1,000.00 as reimbursement of medical
has ruled, project or work pool employees who expenses against A.M. Oreta and Company, Inc.
have gained the status of regular employees are and ENDECO with the POEA.
subject to the no work-no pay principle.
The petitioner A.M. Oreta and Company, Inc. and
The Courts ruling here is meant precisely to give life ENDECO filed their answer and alleged that the
to the constitutional policy of strengthening the contract of employment entered into between
labor sector, but, we stress, not at the expense of petitioners and Grulla provides, as one of the
management. Lest it be misunderstood, this ruling grounds for termination of employment, violation of
does not mean that simply because an employee is the rules and regulations promulgated by the
a project or work pool employee even outside the contractor; and that Grulla was dismissed because
construction industry, he is deemed, ipso jure, a he has not performed his duties satisfactorily within
regular employee. All that we hold today is that the probationary period of three months.
once a project or work pool employee has been:
(1) continuously, as opposed to intermittently, re- POEA held that complainant's dismissal was illegal
hired by the same employer for the same tasks or and warrants the award of his wages for the
nature of tasks; and (2) these tasks are vital, unexpired portion of the contract.
necessary and indispensable to the usual business Petitioner appealed from the adverse decision to
or trade of the employer, then the employee must the respondent Commission.
be deemed a regular employee, pursuant to Article
280 of the Labor Code and jurisprudence. Respondent Commission dismissed the appeal for
lack of merit and affirmed in toto the decision of
Disposition: Instant petition is GRANTED. the POEA..

Casual, Art. 280, 2nd par.; Omnibus rules, Book VI, ISSUES:1. Whether Or Not the employment of
Rule 1, Sec. 5 (b) respondent Grulla was illegally terminated by the
petitioner
1 A work pool may exist although the workers in the pool do not receive salaries and are free to seek 2. Whether Or Not Grulla is entitled to salaries
other employment during temporary breaks in the business, provided that the worker shall be available
when called to report for a project. Although primarily applicable to regular seasonal workers, this set-up corresponding to the unexpired portion of his
can likewise be applied to project workers insofar as the effect of temporary cessation of work is
concerned. This is beneficial to both the employer and employee for it prevents the unjust situation of
employment contract.
coddling labor at the expense of capital and at the same time enables the workers to attain the status of
regular employees.

Page 46 of 5
HELD1. YES. Article 280 (formerly Article 281) of the Nowhere in the employment contract executed
Labor Code, as amended, provides: between petitioner company and respondent
"Article 280. Regular and Casual Grulla is there a stipulation that the latter shall
Employment. - The provisions of written undergo a probationary period for three months
agreement to the contrary notwithstanding before he can quality as a regular employee. There
and regardless of the oral agreements of is also no evidence on record showing that the
the parties, an employment shall be Grulla had been apprised of his probationary status
deemed to be regular where the employee and the requirements which he should comply in
has been engaged to perform activities order to be a regular employee. In the absence of
which are usually necessary or desirable in these requisites, there is justification in concluding
the usual business or trade of the employer, that respondent Grulla was a regular employee at
except where the employment has been the time he was dismissed by petitioner.
fixed for a specific project or undertaking
the completion or termination of which has As such, he is entitled to security of tenure during his
been determined at the time of the period of employment and his services cannot be
engagement of the employment or where terminated except for just and authorized causes
the work or service to be performed is enumerated under the Labor Code and under the
seasonal in nature and the employment is emloyment contract.
far the duration of the season.
Granting, in gratia argumenti, that respondent is a
"An employment shall be deemed to be probationary employee, he cannot, likewise, be
casual if it is not covered by the preceding removed except for cause during the period of
paragraph: Provided, that any employee probation. Although a probationary or temporary
who has rendered at least one year of employee has limited tenure, he still enjoys security
service, whether such service is continuous of tenure. During his tenure of employment or
or broken, shall be considered a regular before his contract expires, he cannot be removed
employee with respect to the activity in except for cause as provided for by law.
which he is employed and his employment The alleged ground of unsatisfactory performance
shall continue while such actually exists." relied upon by petitioner for dismissing respondent
Grulla is not one of the just causes for dismissal
Policy Instructions No. 12 of the then Minister of provided in the Labor Code. Neither is it included
Labor (now Secretary of Labor and Employment) among the grounds for termination of employment
which provides: under Article VII of the contract of employment
"PD 850 has defined the concept of regular executed by petitioner company and respondent
and casual employment. What determines Grulla.
regularity or casualness is not the
employment contract, written or otherwise, Grulla was not, in any manner, notified of the
but the nature of the job. If the job is usually charges against him before he was outrightly
necessary or desirable to the main business dismissed. Neither was any hearing or investigation
of the employer, then employment is conducted by the company to give the
regular. . . ." respondent a chance to be heard concerning the
A perusal of the employment contract reveals that alleged unsatisfactory performance of his work.
although the period of employment of respondent
Grulla is 12 months, the contract period is 2. YES. The dismissal of Grulla violated the security of
renewable subject to future agreement of the tenure under the contract of employment which
parties. It is clear from the employment contract specifically provides that the contract term shall be
that the respondent Grulla was hired by the for a period of twelve (12) calendar months.
company as a regular employee and not just a Consequently, the respondent Grulla should be
mere probationary employee. paid his salary for the unexpired portion of his
On the matter of probationary employment, the contract of employment which is ten (10) months.
law in point is Article 281 (formerly Article 252) of the
Labor Code which provides in part: Disposition: Petition was dismissed
"Art. 281. Probationary Employment. - . . . .
The services of an employee who has been One Year Service: KIMBERLY INDEPENDENT LABOR
engaged on a probationary basis may be UNION V DRILON
terminated for a just cause or when he fails 185 SCRA 190
to qualify as a regular employee in REGALADO; May 9, 1990
accordance with reasonable standards FACTS: Kimberly-Clark Philippines, Inc. (KIMBERLY)
made known by the employer to the executed a three-year collective bargaining
employee at the time of his engagement. agreement (CBA) with United Kimberly-Clark
An employee who is allowed to work after a Employees Union-Philippine Transport and General
probationary period shall be considered Workers' Organization (UKCEUPTGWO) which
regular employee." expired on June 30, 1986.

The law is clear to the effect that in all cases Within the 60-day freedom period prior to the
involving employees engaged on probationary' expiration of and during the negotiations for the
basis, the employer shall make known to the renewal of the aforementioned CBA, some
employee at the time he is hired, the standards by members of the bargaining unit formed another
which he will qualify as a regular employee. union called "Kimberly Independent Labor Union for
Page 47 of 5
Solidarity, Activism and Nationalism-Organized desirable in the usual business or trade of
Labor Association in Line Industries and Agriculture the employer; and
(KILUSAN-OLALIA) (2) those who have rendered at least one year
of service, whether continuous or broken,
April 21, 1986, KILUSAN-OLALIA filed a petition for with respect to the activity in which they are
certification election. KIMBERLY and UKCEU-PTGWO employed
did not object to the holding of a certification
election but objected to the inclusion of the so- The individual petitioners herein who have been
called contractual workers whose employment with adjudged to be regular employees (by law) fall
KIMBERLY was coursed through an independent under the second category. These are the
contractor, Rank Manpower Company (RANK, for mechanics, electricians, machinists, machine shop
short), as among the qualified voters. helpers, warehouse helpers, painters, carpenters,
On June 2, 1986, Med-Arbiter Bonifacio I. pipefitters and masons. It is not disputed that these
Marasigan, who was handling the certification workers have been in the employ of KIMBERLY for
election case issued an order declaring the more than one year at the time of the filing of the
following as eligible to vote in the certification petition for certification election by KILUSAN-
election, thus: OLALIA.
1) regular rank-and-file laborers/employees of
the respondent company; While the actual regularization of these employees
2) casuals who have worked at least six (6) entails the mechanical act of issuing regular
months; 3) Contractual employees who are appointment papers and compliance with such
allegedly in the employ of an independent other operating procedures as may be adopted by
contractor and who have also worked for at the employer, it is more in keeping with the intent
least six (6) months and spirit of the law to rule that the status of regular
employment attaches to the casual worker on the
During the pre-election conference, 64 casual day immediately after the end of his first year of
workers were challenged by KIMBERLY and UKCEU- service
PTGWO on the ground that they are not employees The law is explicit. As long as the employee has
of KIMBERLY but of RANK. It was agreed by all the rendered at least one year of service, he becomes
parties that the 64 voters shall be allowed to cast a regular employee with respect to the activity in
their votes but that their ballots shall be segregated which he is employed. The law does not provide
and subject to challenge proceedings. the qualification that the employee must first be
issued a regular appointment or must first be
After the elections, UKCEU-PTGWO won over formally declared as such before he can acquire a
KILUSAN-OLALIA by 20 votes. This count considered regular status. Obviously, where the law does not
the votes of the 64 employees as separate. distinguish, no distinction should be drawn.
In a case regarding the status of the 64 employees
in relation to the certification election, it was held On the basis of the foregoing circumstances, and
by med-arbiter Sanchez that: as a consequence of their status as regular
2)The other casual employees not employees, those workers not perforce janitorial
performing janitorial and yard maintenance and yard maintenance service were performance
services were deemed labor-only entitled to the payment of salary differential, cost of
contractuals and since labor-only living allowance, 13th month pay, and such other
contracting is prohibited, such employees benefits extended to regular employees under the
were held to have attained the status of CBA, from the day immediately following their first
regular employees, the regularization being year of service in the company.
effective as of the date of the decision;
3. UKCEU-PTGWO, having garnered more These regular employees are likewise entitled to
votes than KILUSAN-OLALIA, was certified as vote in the certification election held in July 1, 1986.
the exclusive bargaining representative of Consequently, the votes cast by those employees
KlMBERLY's employees; not performing janitorial and yard maintenance
Since the members were only considered regular at service, which forms part of the 64 challenged
the time of the decision, their votes were not re- votes, should be opened, counted and considered
considered as regards the election. Winning union for the purpose of determining the certified
and company executed a CBA. KIMBERLY-OLALIA bargaining representative.
filed for a TRO on the CBA and included the
question of the status of the 64 members in INTEGRATED CONTRACTOR vs. NLRC (SOLON)
question. 464 SCRA 265
QUISUMBING; August 9, 2005
ISSUE: Whether Or Not the 64 employees were NATURE: Appeal from a decision of the CA affirming
regular employees at the time of the certification the NLRCs findings which declared respondent
election Solon a regular employee of the petitioner and
awarded him with 13th month pay, service incentive
HELD: YES. Article 280 of the Labor Code provides leave pay, reinstatement to his former position with
for two kinds of regular employees: full backwages from the time his salary was
(1) those who are engaged to perform withheld until his reinstatement.
activities which are usually necessary or

Page 48 of 5
FACTS: Petitioner is a plumbing contractor. Its renewed on a month to month basis until
business depends on the number and frequency of terminated by either party. The pertinent provisions
the projects it is able to contract with its clients. of the contract are:
1. The cooperative agrees and undertakes
Respondent Solon worked for petitioner several to perform and/or provide for the company,
months at a time from 1994 to 1998. on a non-exclusive basis for a period of one
year the following services for the Bacolod
On Feb. 1998, while Solon was about to log out from Shrimp Processing Plant:
work, he was informed that it was his last day of A. Messengerial/Janitorial
work as he had been terminated. He went back to B. Shrimp Harvesting/Receiving
petitioners office to sign a clearance so he could C. Sanitation/Washing/Cold Storage
claim his 13th month pay and tax refunds. However,
he refused to sign when he read the clearance 4. There is no employer-employee
indicating that he had resigned. He then filed a relationship between the company and the
complaint for illegal dismissal without due cause cooperative, or the cooperative and any of
and due process. its members, or the company and any
members of the cooperative. The
The Labor Arbiter ruled that Solon was a regular cooperative is an association of self-
employee and could only be removed for cause. employed members, an independent
NLRC affirmed with only a modification as to the contractor, and an entrepreneur. It is
computation of 13th month pay. CA also affirmed. subject to the control and direction of the
company only as to the result to be
ISSUE: Whether Or Not respondent is a regular accomplished by the work or services herein
employee specified, and not as to the work herein
contracted. The cooperative and its
HELD: YES. The test to determine whether members recognize that it is taking a
employment is regular or not is the reasonable business risk in accepting a fixed service fee
connection between the particular activity to provide the services contracted for and
performed by the employee in relation to the usual its realization of profit or loss from its
business or trade of the employer. Also, if the undertaking, in relation to all its other
employee has been performing the job for at least undertakings, will depend on how efficiently
one year, even if the performance is not continuous it deploys and fields its members and how
or merely intermittent, the law deems the repeated they perform the work and manage its
and continuing need for its performance as operations.
sufficient evidence of the necessity, if not
indispensability of that activity to the business. (De Pursuant to the contract, Sunflower engaged
Leon v NLRC) private respondents to, as they did, render services
While length of time may not be the controlling test at SMCs Bacolod Shrimp Processing Plant at Sta.
for project employment, it is vital in determining if Fe, Bacolod City. The contract was deemed
the employee was hired for a specific undertaking renewed by the parties every month after its
or tasked to perform functions vital, necessary and expiration on January 1, 1994 and private
indispensable to the usual business or trade of the respondents continued to perform their tasks until
employer. Here, private respondent had been a September 11, 1995. In July 1995, private
project employee several times over. His respondents filed a complaint before the NLRC,
employment ceased to be coterminous with Regional Arbitration Branch No. VI, Bacolod City,
specific projects when he was repeatedly re-hired praying to be declared as regular employees of
due to the demands of petitioners business. SMC, with claims for recovery of all benefits and
privileges enjoyed by SMC rank and file employees.
Disposition: Assailed Decision dated October 30, Private respondents subsequently filed on
2001 and the Resolution dated February 28, 2002 of September 25, 1995 an Amended Complaint to
CA are AFFIRMED with MODIFICATION. The include illegal dismissal as additional cause of
petitioner id hereby ORDERED to (1) reinstate the action following SMCs closure of its Bacolod Shrimp
respondent with no loss of seniority rights and other Processing Plant on September 15, 1995which
privileges; and (2) pay respondent his backwages, resulted in the termination of their services. SMC
13th month pay for the year 1998 and Service filed a Motion for Leave to File Attached Third Party
Incentive Leave Pay computed from the date of his Complaint dated November 27, 1995 to implead
illegal dismissal up to the date of his actual Sunflower as Third Party Defendant which was, by
reinstatement Order of December 11, 1995, granted by Labor
Arbiter Ray Alan T. Drilon. In the meantime, on
SAN MIGUEL CORP vs. ABALLA September 30, 1996, SMC filed before the Regional
461 SCRA 392 Office at Iloilo City of the Department of Labor and
CARPIO-MORALES; June 28 2005 Employment (DOLE) a Notice of Closure of its
NATURE: Special Civil Action in the Supreme Court. aquaculture operations effective on even date,
Certiorari citing serious business losses. By Decision of
September 23, 1997, Labor Arbiter Drilon dismissed
FACTS: Petitioner San Miguel Corporation (SMC) private respondents complaint for lack of merit.
and Sunflower Multi-Purpose Cooperative
(Sunflower), entered into a one-year Contract of Private respondents appealed to the NLRC. By
Services commencing on January 1, 1993, to be Decision of December 29, 1998, the NLRC dismissed
Page 49 of 5
the appeal for lack of merit, it finding that third being subject to the control of the employer,
party respondent Sunflower was an independent except only as to the results of the work. As for
contractor in light of its observation that [i]n all the those of private respondents who were engaged in
activities of private respondents, they were under janitorial and messengerial tasks, they fall under the
the actual direction, control and supervision of third second category and are thus entitled to
party respondent Sunflower, as well as the payment differential pay and benefits extended to other
of wages, and power of dismissal. By Decision of SMC regular employees from the day immediately
February 7, 2001, the appellate court reversed the following their first year of service.
NLRC decision and accordingly found for private
respondents. Justifying its reversal of the findings of In legitimate labor contracting, the law creates an
the labor arbiter and the NLRC, the appellate court employer-employee relationship for a limited
reasoned:Although the terms of the non-exclusive purpose, i.e., to ensure that the employees are paid
contract of service between SMC and [Sunflower] their wages. The principal employer becomes
showed a clear intent to abstain from establishing jointly and severally liable with the job contractor,
an employer-employee relationship between SMC only for the payment of the employees wages
and [Sunflower] or the latters members, the extent whenever the contractor fails to pay the
to which the parties successfully realized this intent same. Other than that, the principal employer is
in the light of the applicable law is the controlling not responsible for any claim made by the
factor in determining the real and actual employees.[50]
relationship between or among the parties.There
being a finding of labor-only contracting, liability In labor-only contracting, the statute creates an
must be shouldered either by SMC or [Sunflower] or employer-employee relationship for a
shared by both (See Tabas vs. California comprehensive purpose: to prevent a
Manufacturing, Inc., supra, p. 502). SMC however circumvention of labor laws. The contractor is
should be held solely liable for [Sunflower] became considered merely an agent of the principal
non-existent with the closure of the aquaculture employer and the latter is responsible to the
business of SMC. employees of the labor-only contractor as if such
employees had been directly employed by the
ISSUEs:1. Whether Or Not the respondents are principal employer.[51]
employees of San Miguel Corporation. The Contract of Services between SMC and
2. Whether Or Not the retrenchment was valid and Sunflower shows that the parties clearly disavowed
consequently, whether the respondents are entitled the existence of an employer-employee
to relief. relationship between SMC and private
respondents. The language of a contract is not,
HELD:1. YES. Since private respondents who were however, determinative of the parties relationship;
engaged in shrimp processing performed tasks rather it is the totality of the facts and surrounding
usually necessary or desirable in the aquaculture circumstances of the case.[52] A party cannot
business of SMC, they should be deemed regular dictate, by the mere expedient of a unilateral
employees of the latter and as such are entitled to declaration in a contract, the character of its
all the benefits and rights appurtenant to regular business, i.e., whether as labor-only contractor or
employment. They should thus be awarded job contractor, it being crucial that its character be
differential pay corresponding to the difference measured in terms of and determined by the
between the wages and benefits given them and criteria set by statute
those accorded SMCs other regular employees.
Respecting the private respondents who were 2. San Miguel Corporation has thus proven
tasked with janitorial and messengerial duties, this substantial business reverses justifying retrenchment
Court quotes with approval the appellate courts of its employees.
ruling thereon:
In the case at bar, company losses were duly
Those performing janitorial and messengerial established by financial documents audited by
services however acquired regular status only after Joaquin Cunanan & Co. showing that the
rendering one-year service pursuant to Article 280 aquaculture operations of SMCs Agribusiness
of the Labor Code. Although janitorial and Division accumulated losses amounting to
messengerial services are considered directly P145,848,172.00 in 1992 resulting in the closure of its
related to the aquaculture business of SMC, they Calatrava Aquaculture Center in Negros
are deemed unnecessary in the conduct of its Occidental, P11,393,071.00 in 1993 and
principal business; hence, the distinctionThe law of P80,325,608.00 in 1994 which led to the closure of its
course provides for two kinds of regular employees, San Fernando Shrimp Processing Plant in
namely: (1) those who are engaged to perform Pampanga and the Bacolod Shrimp Processing
activities which are usually necessary or desirable in Plant in 1995. For termination due to retrenchment
the usual business or trade of the employer; and (2) to be valid, however, the law requires that written
those who have rendered at least one year of notices of the intended retrenchment be served by
service, whether continuous or broken, with respect the employer on the worker and on the DOLE at
to the activity in which they are employed. least one (1) month before the actual date of the
retrenchment in order to give employees some time
The test to determine the existence of independent to prepare for the eventual loss of their jobs, as well
contractorship is whether one claiming to be an as to give DOLE the opportunity to ascertain the
independent contractor has contracted to do the verity of the alleged cause of termination. Private
work according to his own methods and without respondents, however, were merely verbally
Page 50 of 5
informed on September 10, 1995 by SMC Prawn The employment contract of 1971 was executed
Manager Ponciano Capay that effective the when the Labor Code of the Philippines had not yet
following day or on September 11, 1995, they were been promulgated, which came into effect some 3
no longer to report for work as SMC would be years after the perfection of the contract.
closing its operations. Where the dismissal is based
on an authorized cause under Article 283 of the ISSUE: Whether Or Not the provisions of the Labor
Labor Code but the employer failed to comply with Code regarding probationary/regular employees
the notice requirement, the sanction should be stiff have anathematized fixed period employment or
as the dismissal process was initiated by the employment for a term.
employers exercise of his management
prerogative, as opposed to a dismissal based on a HELD: NO. Before the Labor Code, there was no
just cause under Article 282 with the same doubt about the validity of term employment. It
procedural infirmity where the sanction to be was impliedly but clearly recognized by the
imposed upon the employer should be tempered Termination Pay law, RA 1052.
as the dismissal process was, in effect, initiated by
an act imputable to the employee. In light of the The Civil Code, which has always recognized, and
factual circumstances of the case at bar, the Court continues to recognize, the validity and propriety of
awards P50,000.00 to each private respondent as contracts and obligations with a fixed or definite
nominal damages.The grant of separation pay as period, and imposes no restraints on the freedom of
an incidence of termination of employment due to the parties to fix the duration of a contract,
retrenchment to prevent losses is a statutory whatever its object, be it specie, goods or services,
obligation on the part of the employer and a except the general admonition against stipulations
demandable right on the part of the contrary to law, morals, good customs, public order
employee. Private respondents should thus be or public policy. Under the Civil Code, therefore,
awarded separation pay equivalent to at least one and as a general proposition, fixed-term
(1) month pay or to at least one-half month pay for employment contracts are not limited, as they are
every year of service, whichever is higher, as under the present Labor Code, to those by nature
mandated by Article 283 of the Labor Code or the seasonal or for specific projects with pre-
separation pay awarded by SMC to other regular determined dates of completion; they also include
SMC employees that were terminated as a result of those to which the parties by free choice have
the retrenchment, depending on which is most assigned a specific date of termination.
beneficial to private respondents.Considering that (theres a long, long history about the changes of
private respondents were not illegally dismissed, provisions in the labor code, showing how fixed
however, no backwages need be awarded. It is period employment became less and less
well settled that backwages may be granted only acceptable...)
when there is a finding of illegal dismissal.[80] The
appellate court thus erred in awarding backwages Where from the circumstances it is apparent that
to private respondents. What was involved in that periods have been imposed to preclude acquisition
case was one of illegal dismissal. of tenurial security by the employee, they should be
struck down or disregarded as contrary to public
Fixed-Term: BRENT SCHOOL vs. ZAMORA policy, morals, etc. But where no such intent to
181 SCRA 702 circumvent the law is shown, where the reason for
NARVASA; February 5, 1990 the law does not exist, e.g., where it is indeed the
NATURE: Appeal from judgment employee himself who insists upon a period or
where the nature of the engagement is such that,
FACTS: Alegre was athletic director at Brent, at a without being seasonal or for a specific project, a
yearly compensation of P20,000. Her contract fixed definite date of termination is a sine qua non,
a specific term of 5 years for its existence, from July, would an agreement fixing a period be essentially
1971, to July, 1976. Subsequent subsidiary evil or illicit, therefore anathema? Would such an
agreements in March 1973, August 1973, and Sept. agreement come within the scope of Article 2802
1974 reiterated the same terms and conditions, which admittedly was enacted "to prevent the
including the expiry date, as those contained in the circumvention of the right of the employee to be
original contract of July, 1971. secured in (his) employment?"
Three months before the expiration of the stipulated
period, in April 1976, Alegre was given a copy of Article 280 of the Labor Code, under a narrow and
the report filed by Brent with the Dep. Of Labor literal interpretation would appear to restrict,
advising of the termination of his services, effective without reasonable distinctions, the right of an
July 16, 1976. The stated ground for termination was employee to freely stipulate with his employer the
completion of contract, expiration of the definite duration of his engagement, it logically follows that
period of employment. such a literal interpretation should be eschewed or
avoided. The law must be given a reasonable
Alegre protested the announced termination of his interpretation, to preclude absurdity in its
employment. He argued that although his contract
2 Art. 280. Regular and casual employment. The provisions of written agreement to the contrary notwithstanding
did stipulate that the same would terminate on July and regardless of the oral agreement of the parties, an employment shall be deemed to be regular where the
17, 1976, since his services were necessary and employee has been engaged to perform activities which are usually necessary or desirable in the usual business or
trade of the employer, except where the employment has been fixed for a specific project or undertaking the
desirable in the usual business of his employer, and completion or termination of which has been determined at the time of the engagement of the employee or where
the work or service to be performed is seasonal in nature and the employment is for the duration of the season.
his employment had lasted for five years, he had An employment shall be deemed to be casual if it is not covered by the preceding paragraph: Provided, That any
employee who has rendered at least one year of service, whether such service is continuous or broken, shall be
acquired the status of regular employee and could considered a regular employee with respect to the activity in which he is employed and his employment shall
not be removed except for valid cause. continue while such activity exists.

Page 51 of 5
application. Outlawing the whole concept of term
employment and subverting to boot the principle of FERDINAND PALOMARES and TEODULO
freedom of contract to remedy the evil of MUTIA, petitioners, vs. NATIONAL LABOR RELATIONS
employer's using it as a means to prevent their COMMISSION, (5TH DIVISION) and NATIONAL STEEL
employees from obtaining security of tenure is like CORPORATION, respondents.
cutting off the nose to spite the face or, more
relevantly, curing a headache by lopping off the Facts: Petitioners Ferdinand Palomares and Teodulo
head. Mutia were hired by respondent National Steel
Corporation (NSC) by virtue of contracts of
Familiar examples of employment contracts which employment for its Five Year Expansion Program or
may be neither for seasonal work nor for specific FYEP, Phase I and II-A, for varying lengths of time.
projects, but to which a fixed term is essential:
overseas employment contracts to which, the Petitioners, along with other employees, filed a
concept of regular employment will all that it consolidated petition for regularization, wage
implies does not appear ever to have been differential, CBA coverage and other benefits.[2] In
applied, Article 280 of the Labor Code not his decision dated April 29, 1992, Labor Arbiter
withstanding; appointments to the positions of Nicodemus G. Palangan ordered the dismissal of
dean, assistant dean, college secretary, principal, the complaint with respect to 26 complainants but
and other administrative offices in educational ruled in favor of petitioners. Palomares, Mutia and
institutions, which are by practice or tradition four other complainants were adjudged as regular
rotated among the faculty members, and where employees of respondent corporation. The
fixed terms are a necessity, without which no dispositive portion of his decision reads:
reasonable rotation would be possible. Despite the
provisions of Article 280, Policy, Instructions No. 8 of WHEREFORE, premises considered, the petition for
the Minister of Labor implicitly recognize that regularization as well as the monetary benefits of
certain company officials may be elected for what the above-named complainants are hereby
would amount to fixed periods, at the expiration of ordered DISMISSED for lack of merit except six
which they would have to stand down, in providing complainants stated below.
that these officials," . . . may lose their jobs as
president, executive vice-president or vice- For the complainants who were terminated during
president, etc. because the stockholders or the the pendency of these cases the respondent is
board of directors for one reason or another did not hereby ordered to pay them separation pay
re-elect them." equivalent to one month salary for those who have
rendered one or two years of service and three
Since the entire purpose behind the development months salary for those who have served the
of legislation culminating in the present Article 280 company for at least 5 years.
of the Labor Code clearly appears to have been to
prevent circumvention of the employee's right to For complainants Edgardo Pongase, Aquiles Colita,
be secure in his tenure, the clause in said article Lolinio Solatorio, Ferdinand Palomares, Teodulo
indiscriminately and completely ruling out all written Mutia, and Rodolfo Leopoldo, this office consider
or oral agreements conflicting with the concept of (sic) them as regular employees for reason that the
regular employment as defined therein should be activities they performed are regular, and
construed to refer to the substantive evil that the necessary in the usual trade or course of business of
Code itself has singled out: agreements entered the company.
into precisely to circumvent security of tenure. It
should have no application to instances where a Respondent is likewise ordered to pay these regular
fixed period of employment was agreed upon employees their salary differential to be computed
knowingly and voluntarily by the parties, without three years back from the filing of these complaints.
any force, duress or improper pressure being
brought to bear upon the employee and absent On appeal, the NLRC reversed the findings of the
any other circumstances vitiating his consent, or Labor Arbiter in a decision dated November 23,
where it satisfactorily appears that the employer 1994. Respondent Commission held that petitioners
and employee dealt with each other on more or were project employees and that their assumption
less equal terms with no moral dominance of regular jobs were mainly due to peakloads or the
whatever being exercised by the former over the absence of regular employees during the latters
latter. Unless thus limited in its purview, the law temporary leave.[4] After their motion for
would be made to apply to purposes other than reconsideration was denied on March 30,
those explicitly stated by its framers; it thus becomes 1995,[5] petitioners filed this petition. The Court finds
pointless and arbitrary, unjust in its effects and apt that petitioners failed to show any grave abuse of
to lead to absurd and unintended consequences. discretion on the part of the NLRC in rendering its
questioned decision and resolutions of November
Disposition: The public respondent's Decision 23, 1994 and March 30, 1995, respectively.
complained of is REVERSED and SET ASIDE.
Respondent Alegre's contract of employment with ISSUE: WON the employment was on a fixed term
Brent School having lawfully terminated with and by basis.
reason of the expiration of the agreed term of
period thereof, he is declared not entitled to RULING: Yes.Article 280 of the Labor Code, the law
reinstatement and the other relief awarded and on the subject of regular employment, reads:
confirmed on appeal in the proceedings below.
Page 52 of 5
The provisions of the written agreement to the and metal transistors and glass diods. It caters to
contrary notwithstanding and regardless of the oral domestic and foreign corporations that
agreement of the parties, an employment shall be manufacture computers, telecommunications
deemed to be regular where the employee has equipment and cars.
been engaged to perform activities which are
usually necessary or desirable in the usual business Aside from contractual employees, the petitioner
or trade of the employer, except where the employed 1,029 regular workers. The employees
employment has been fixed for a specific project or were subjected to periodic performance appraisal
undertaking the completion or termination of which based on output, quality, attendance and work
has been determined at the time of the attitude.[2]One was required to obtain a
engagement of the employee or where the work or performance rating of at least 3.0 for the period
services to be performed is seasonal in nature and covered by the performance appraisal to maintain
the employment is for the duration of the season. good standing as an employee.

An employment shall be deemed to be casual if it is On May 8, 1992, respondent Eloisa Fadriquela


not covered by the preceding paragraph: executed a Contract of Employment with the
Provided, That any employee who has rendered at petitioner in which she was hired as a production
least one year of service, whether such service is operator with a daily salary of P118. Her initial
continuous or broken shall be considered a regular contract was for a period of three months up
employee with respect to the activity in which he is to August 8, 1992,[3] but was extended for two
employed and his employment shall continue while months when she garnered a performance rating
such actually exists. (Emphasis added). of 3.15.[4] Her contract was again renewed for two
months or up to December 16, 1992,[5] when she
The principal test for determining whether an received a performance rating of 3.8.[6] After the
employee is a project employee and not a regular expiration of her third contract, it was extended
employee is whether he was assigned to carry out anew, for three months,[7] that is, from January 4,
a specific project or undertaking, the duration and 1993 to April 4, 1993.
scope of which were specified at the time he was
engaged for that project. After garnering a performance rating of 3.4,[8] the
respondents contract was extended for another
It is quite evident that petitioners were employed three months, that is, from April 5, 1993 to June 4,
for a specific project or projects undertaken by 1993.[9] She, however, incurred five absences in the
respondent corporation. The component projects month of April, three absences in the month of May
of the latters Five Year Expansion Program include and four absences in the month of June.[10] Line
the setting up of a Cold Rolling Mill Expansion supervisor Shirley F. Velayo asked the respondent
Project, establishing a Billet Steel-Making Plant, why she incurred the said absences, but the latter
installation of a Five Stand TDM and Cold Mill failed to explain her side. The respondent was
Peripherals Project. In the case of ALU- warned that if she offered no valid justification for
TUCP v. NLRC, we held that the same Five Year her absences, Velayo would have no other
Expansion Program (or more precisely, each of its recourse but to recommend the non-renewal of her
component projects) constitutes a distinct contract. The respondent still failed to respond, as a
undertaking identifiable from the ordinary business consequence of which her performance rating
and activity of NSC, which is the production and declined to 2.8. Velayo recommended to the
marketing of steel products. Even if, as admitted by petitioner that the respondents employment be
the parties, petitioners were repeatedly and terminated due to habitual absenteeism,[11]in
successively re-hired on the basis of a contract of accordance with the Company Rules and
employment for more than one year, they cannot Regulations.[12] Thus, the respondents contract of
be considered regularized. Length of service is not employment was no longer renewed.
the controlling determinant of the employment
tenure of a project employee.[16] As stated earlier, it The respondent filed a complaint before the
is based on whether or not the employment has National Capital Region Arbitration Branch of the
been fixed for a specific project or undertaking, the National Labor Relations Commission (NLRC) for
completion of which has been determined at the illegal dismissal against the petitioner, docketed as
time of the engagement of the NLRC Case No. NCR-07-04263-93. She alleged, inter
employee. Furthermore, the second paragraph of alia, that she was illegally dismissed, as there was no
Article 280, providing that an employee who has valid cause for the termination of her
rendered service for at least one (1) year, shall be employment. She was not notified of any infractions
considered a regular employee, pertains to casual she allegedly committed; neither was she
employees and not to project employees such as accorded a chance to be heard. According to the
petitioners. respondent, the petitioner did not conduct any
formal investigation before her employment was
terminated. Furthermore, considering that she had
PHILIPS SEMICONDUCTORS (PHILS.), INC., petitioner, rendered more than six months of service to the
vs. ELOISA FADRIQUELA, respondent. petitioner, she was already a regular employee and
could not be terminated without any justifiable
Facts: The petitioner Philips Semiconductors (Phils.), cause. Moreover, her absences were covered by
Inc. is a domestic corporation engaged in the the proper authorizations.[13]
production and assembly of semiconductors such
as power devices, RF modules, CATV modules, RF
Page 53 of 5
On the other hand, the petitioner contended that four times, to the same position, with the same
the respondent had not been dismissed, but that chores. Such a continuing need for the services of
her contract of employment for the period of April the respondent is sufficient evidence of the
4, 1993 to June 4, 1993 merely expired and was no necessity and indispensability of her services to the
longer renewed because of her low performance petitioners business. While at the start, petitioner
rating. Hence, there was no need for a notice or was just a mere contractual employee, she
investigation. Furthermore, the respondent had became a regular employee as soon as she had
already accumulated five unauthorized absences completed one year of service. It is not difficult to
which led to the deterioration of her performance, see that to uphold the contractual arrangement
and ultimately caused the non-renewal of her between private respondent and petitioner would,
contract. On June 26, 1997, the Labor Arbiter in effect, be to permit employers to avoid the
rendered a decision dismissing the complaint for necessity of hiring regular or permanent
lack of merit. On October 11, 1999, the appellate employees. By hiring employees indefinitely on a
court rendered a decision reversing the decisions of temporary or casual status, employers deny their
the NLRC and the Labor Arbiter and granting the right to security of tenure. This is not sanctioned by
respondents petition. law. [34]

ISSUE: WON the petitioners are still contractual Even then, the petitioners reliance on the CBA is
employees whose employment was fixed. misplaced. For, as ratiocinated by the appellate
court in its assailed decision:
RULING: No, the petitioners became regular
employees. Art. 280. Regular and Casual Obviously, it is the express mandate of the CBA not
Employment. The provisions of written agreement to to include contractual employees within its
the contrary notwithstanding and regardless of the coverage. Such being the case, we see no reason
oral argument of the parties, an employment shall why an agreement between the representative
be deemed to be regular where the employee has union and private respondent, delaying the
been engaged to perform activities which are regularization of contractual employees, should
usually necessary or desirable in the usual business bind petitioner as well as other contractual
or trade of the employer, except where the employees. Indeed, nothing could be more unjust
employment has been fixed for a specific project or than to exclude contractual employees from the
undertaking the completion or termination of which benefits of the CBA on the premise that the same
has been determined at the time of the contains an exclusionary clause while at the same
engagement of the employee or where the work or time invoke a collateral agreement entered into
services to be performed is seasonal in nature and between the parties to the CBA to prevent a
the employment is for the duration of the season. contractual employee from attaining the status of a
regular employee.
An employment shall be deemed to be casual if it is
not covered by the preceding paragraph;
Provided, That, any employee who has rendered at PHILIPPINE TOBACCO FLUE-CURING & REDRYING
least one year of service, whether such service is CORPORATION, petitioner, vs. NATIONAL LABOR
continuous or broken, shall be considered a regular RELATIONS COMMISSION
employee with respect to the activity in which he is
employed and his employment shall continue while Facts: These refer to the consolidated cases for
such activity exists. payment of separation pay lodged by [the] Lubat
Group, and for illegal dismissal and underpayment
The two kinds of regular employees under the law of separation pay by [the] Luris group, with prayers
are (1) those engaged to perform activities which for damages and attorneys fees against the above
are necessary or desirable in the usual business or respondents.
trade of the employer; and (2) those casual
employees who have rendered at least one year of The record reveals that all complainants in both
service, whether continuous or broken, with respect cases were former workers of respondent with their
to the activities in which they are employed.[20] The respective periods of employment and latest
primary standard to determine a regular wages stated in the parties pleadings/[a]nnexes.
employment is the reasonable connection
between the particular activity performed by the On August 1, 1994, due to supposed serious
employee in relation to the business or trade of the financial reverses and losses suffered by respondent
employer. The test is whether the former is usually and its desire to prevent further losses, a notice of
necessary or desirable in the usual business or trade permanent closure of its red[r]ying operations at
of the employer. In this case, the respondent was Balintawak, Quezon City and transfer [of] the same
employed by the petitioner on May 8, 1992 as to Candon, Ilocos Sur was served to the DOLE.
production operator. She was assigned to
wirebuilding at the transistor division. There is no On August 3, 1994, complainants were also notified
dispute that the work of the respondent was of the said decision to close and transfer.
necessary or desirable in the business or trade of
the petitioner.[24] She remained under the employ of On August 16, 1994, their separation benefits were
the petitioner without any interruption since May 8, given to them but allegedly [based on] wrong
1992 to June 4, 1993 or for one (1) year and twenty- computation when management did not consider
eight (28) days. The original contract of 3/4 of their length of service as claimed by
employment had been extended or renewed for complainants (Luris group).
Page 54 of 5
Moreover, respondent points out that as the Luris
While the Lubat group were not granted xxx group and the DOLE were served a written notice
separation pay as their previous seasonal service at least one (1) month before the intended date of
[was] not continuous, and as of August, 1994, they closure effective on Sept. 15, 1994, the due process
were not employed ther[e]with as declared by requirement was met.
respondent. Viewed from the above, respondent cannot
prosper.
Based on the complaint and from the above facts,
the issues are as follows: On the other hand, the Lubat group declare that
1) Whether or not the Lubat Group are entitled to originally there were seven complainants but eight
the payment of separation pay[;] were added.
2) Whether or not the Luris Group can be legally Being seasonal workers, they were hired by
awarded separation pay differentials[,] or whether respondent to operate the Balintawak factory from
or not the computation adopted by respondent in January to September, averaging 6 to 8 months
granting complainants separation pay is annually.
erroneous[;] and
3) Whether or not the Luris group can be properly As alleged by them, when they reported for their
allowed backwages and damages by reason of annual shift, respondent refused to extend them
their alleged illegal dismissal, and for both groups, assignment for no apparent reason up to the end
attorneys fees[.] of the season in August, 1994. When they ask[ed]
In [its] position paper respondent maintains that for separation pay, respondent told them that
[the] Lubat group are not entitled to separation because they were not in the payroll for 1994, no
pay for the reason that they were not among those such benefit would be paid to them.
separated or could not have been separated from
employment on August 3, 1994 due to such closure It is their contention that complainants are entitled
and transfer as they were not employed or did not to separation pay [of] at least one-half month pay
report for work at the plant for the 1994 tobacco for every year of service[,] as they were illegally
season as shown by [the] companys records. dismissed[,] to be computed each season ranging
from 6 to 8 months [which] should be considered as
As to the Luris group, although being questioned by one year, contrary to the respondents basis which is
this group, respondent considers the following the total no. of days they actually rendered service.
formula in determining the length of service in years To back up the above, complainants cite a case
as basis for computing the separation pay of this wherein the Supreme Court held that seasonal
group to be fair and reasonable and xxx supported employees are not strictly speaking, separated from
by Article 283 of the Labor Code, as amended, the service but merely considered on leave of
such as the total number of working days actually absence without pay until reemployed. Their
worked over total number of working days in a year employment relationship is never severed but only
(303 days), multipl[ied] by the daily rate and further suspended.
multipl[ied] by 15 days.
For the prosecution of this case, complainants were
Respondent explains that this is so because forced to hire the services of counsel for which they
complainants nature of work is seasonal as they are claim xxx attorneys fees.
employed every year only during the tobacco As far as the Luris group are concerned, they state
season which may fall within the months of February that they were factory workers of
to November but actually work for a period of less respondents numbering one hundred (100) whose
[than] six (6) months for each season. The law names, periods of employment and latest salaries
qualifies tenure for purposes of separation benefits are contained in the lists attached to their position
as based on service and not employment. paper.

With these considerations, respondent claims that As claimed by this group, on August 3, 1994,
complainants relief for separation pay respondents told them that their services were
differentials must fail. already terminated and all of them dismissed as the
On the charge of illegal dismissal by the Luris group, factory would be transferred to Candon, Ilocos Sur.
respondent asserts that complainants were
separated from employment for [a] just cause that Letter-notices dated August 3, 1994, (Annexes F, F-1
is the closure of its REDRYING operations at the and F-2 to their position paper) showing that the
Balintawak plant and the transfer of the same to date when they were notified of the closure was
Candon, Ilocos Sur which was authorized by the the same date they were instantly dismissed
law and the parties CBA. although it is admitted in the notice that their
decision to transfer was made as early as March 5,
The decision of management to close and transfer 1994.
its tobacco processing and REDRYING operations
was based on the fact that it had consistently Furthermore, complainants question the basis of the
incurred a net loss from these operations, its computations of their separation benefits which
principal line of business, although its audited should include the period when there [was] no work
financial statement showed a net profit after tax to be done in a year. [B]ecause of necessity, they
from 1990 to 1993 based on over-all operations. received the short amount as their separation pay
by way of voucher but under protest as shown in
Annexes C-C-1 to C-5 to their pleading.
Page 55 of 5
Decision, Philippine Tobacco and the complainants
With the sudden transfer of the machiner[y] of filed their respective appeals before the NLRC.[5]
respondents without giving them advance notice
leaving them with insufficient separation pay, As noted earlier, the NLRC affirmed the labor
complainants experienced serious anxiety and arbiters Decision. Before this Court, only Philippine
wounded feelings for which they p[r]ay for Tobacco filed the present recourse, as the
damages including attorneys fees. complainants did not question the NLRC Decision.

Consequently, complainants also pray for ISSUE: WON the employees being seasonal
backwages, allowance and other benefits from the employees were illegally dismissed.
date of their illegal dismissal up to the final
disposition of the case. RULING: Yes, these seasonal employees were
illegally dismissed.
Furthermore, complainants maintain that since the
company is being transferred to the province, the Petitioner relies upon our ruling in Mercado v.
formers separation may be considered compulsory NLRC[11] hat the employment [of seasonal
retirement under R.A. 7641, providing for one-half employees] legally ends upon completion of the x x
month pay benefit for every year of service, and x season, a statement which was subsequently
under Section 3, Rule V, Book III of the Labor Code, reiterated in Magcalas v. NLRC.[12] Thus, petitioner
as amended for which they also demand payment argues that it was not obliged to rehire the
thereof. members of the Lubat group for the 1994 season,
because their employment had been terminated
Complainants also submitted the computation of at the end of the 1993 season. Since they were not
their differential in separation pay (addendum and employed for the 1994 season when the Balintawak
supplemental addendum to their position paper) plant was closed, it follows that petitioner has no
Annex G, G-1 to G-4. obligation to award them separation pay due to
the said closure.
To state the facts simply, there are two groups of
employees, namely, the Lubat group and the Luris We are not persuaded. From the facts, we are
group. The Lubat group is composed of petitioners convinced that petitioner illegally dismissed the
seasonal employees who were not rehired for the members of the Lubat group when it refused to
1994 tobacco season. At the start of that season, allow them to work during the 1994 season.
they were merely informed that their employment
had been terminated at the end of the 1993 This Court has previously ruled in Manila Hotel
season. They claimed that petitioners refusal to Company v. CIR[13] that seasonal workers who are
allow them to report for work without mention of called to work from time to time and are
any just or authorized cause constituted illegal temporarily laid off during off-season are not
dismissal. In their Complaint, they prayed for separated from service in said period, but are
separation pay, back wages, attorneys fees and merely considered on leave until reemployed, viz.:
moral damages.
The nature of their relationship x x x is such that
On the other hand, the Luris group is made up of during off season they are temporarily laid off but
seasonal employees who worked during the 1994 during summer season they are re-employed, or
season. On August 3, 1994, they received a notice when their services may be needed. They are not
informing them that, due to serious business losses, strictly speaking separated from the service but are
petitioner planned to close its Balintawak plant and merely considered as on leave of absence without
transfer its tobacco processing and redrying pay until they are re-employed.
operations to Ilocos Sur. Although the closure was Prescinding from the above, we hold that petitioner
to be effective September 15, 1994, they were no is liable for illegal dismissal and should be
longer allowed to work starting August 4, responsible for the reinstatement of the Lubat group
1994. Instead, petitioner awarded them separation and the payment of their back wages. However,
pay computed according to the following formula: since reinstatement is no longer possible as
total no. of days actually worked petitioner has already closed its Balintawak plant,
----------------------------------------------------- x daily rate x respondent members of the said group should
15 days instead be awarded normal separation pay (in lieu
total no. of working days in one year. of reinstatement) equivalent to at least one month
pay, or one month pay for every year of service,
In their Complaint, they claimed that the whichever is higher. It must be stressed that the
computation should be based not on the above separation pay being awarded to the Lubat group
mathematical equation, but on the actual number is due to illegal dismissal; hence, it is different from
of years served. In addition, they contended that the amount of separation pay provided for in
they were illegally dismissed, and thus they prayed Article 283 in case of retrenchment to prevent losses
for back wages. or in case of closure or cessation of the employers
business, in either of which the separation pay is
Against these factual antecedents, the labor equivalent to at least one (1) month or one-half
arbiter ordered the petitioner to pay complainants (1/2) month pay for every year of service,
separation pay differential plus attorneys fees in the whichever is higher.
total amount of P3,092,896.76. Dissatisfied with said

Page 56 of 5
However, despite the fact that the respondent Reform Program (CARP) over the land owned by
members of the Lubat group were entitled to petitioner Starke.[5]
separation pay equivalent to at least one (1) month
pay, or one (1) month pay for every year of service, For her part, petitioner Starke recounted that the
whichever is higher, they cannot receive more than companys Board of Directors petitioned the
the amount awarded to them in the NLRC Decision Sangguniang Bayan of Kabankalan for authority to
-- at least one (1) month or one-half (1/2) month re-classify, from agricultural to industrial,
pay for every year of service, whichever is higher -- commercial and residential, the whole of Hacienda
because they did not appeal from the said Bino, except the portion earmarked for the CARP.
Decision.[21] Therefore, no affirmative award can be She asserted that half of the workers supported the
given to them. In the same manner, although re-classification but the others, which included the
respondents should have been entitled to back herein respondents, opted to become beneficiaries
wages because petitioner illegally deprived them of the land under the CARP. Petitioner Starke
of work during the 1994 season, no such award can alleged that in July 1996, there was little work in the
be given to them, since they did not appeal the plantation as it was off-season; and so, on account
NLRC Decision. The elementary norms of due of the seasonal nature of the work, she issued the
process prevent the grant of such awards, as the order giving preference to those who supported
employer was not given notice that its filing of its the re-classification. She pointed out that when the
own Petition for Certiorari would put it in jeopardy milling season began in October 1996, the work was
of such relief. plentiful again and she issued notices to all workers,
including the respondents, informing them of the
availability of work. However, the respondents
HACIENDA BINO/HORTENCIA STARKE, refused to report back to work. With respect to the
INC./HORTENCIA L. respondents money claims, petitioner Starke
STARKE, petitioners, vs. CANDIDO CUENCA submitted payrolls evidencing payment thereof.

Facts: Hacienda Bino is a 236-hectare sugar On October 6, 1997, Labor Arbiter Ray Allan T. Drilon
plantation located at Barangay Orong, rendered a Decision,[6] finding that petitioner
Kabankalan City, Negros Occidental, and Starkes notice dated July 18, 1996 was tantamount
represented in this case by Hortencia L. Starke, to a termination of the respondents services, and
owner and operator of the said hacienda. holding that the petitioner company was guilty of
The 76 individual respondents were part of the illegal dismissal. The dispositive portion of the
workforce of Hacienda Bino consisting of 220 decision reads:
workers, performing various works, such as
cultivation, planting of cane points, fertilization, WHEREFORE, premises considered, judgment is
watering, weeding, harvesting, and loading of hereby rendered declaring the dismissal of the
harvested sugarcanes to cargo trucks.[2] complainants illegal. Both the petitioners and the
respondents appealed the case to the NLRC. On
On July 18, 1996, during the off-milling season, July 24, 1998, the NLRC affirmed with modification
petitioner Starke issued an Order or Notice which the decision of the Labor Arbiter.
stated, thus:
ISSUE: WON the employees are seasonal
To all Hacienda Employees: employees.
Please bear in mind that all those who signed in
favor of CARP are expressing their desire to get out RULING: No, they are regular emoployees.
of employment on their own volition. Indeed, in a number of cases, the Court has
Wherefore, beginning today, July 18, only those recognized the peculiar facts attendant in
who did not sign for CARP will be given the Mercado case. In Abasolo v. NLRC,[24] and
employment by Hda. Bino. earlier, in Philippine Tobacco Flue-Curing &
(Sgd.) Hortencia Starke[3] Redrying Corporation v. NLRC,[25] the Court made
the following observations:
The respondents regarded such notice as a
termination of their employment. As a In Mercado, although respondent constantly
consequence, they filed a complaint for illegal availed herself of the petitioners services from year
dismissal, wage differentials, 13th month pay, to year, it was clear from the facts therein that they
holiday pay and premium pay for holiday, service were not in her regular employ. Petitioners therein
incentive leave pay, and moral and exemplary performed different phases of agricultural work in a
damages with the NLRC, Regional Arbitration given year. However, during that period, they were
Branch No. VI, Bacolod City, on September 17, free to work for other farm owners, and in fact they
1996.[4] did. In other words, they worked for respondent, but
were nevertheless free to contract their services
In their Joint Sworn Statement, the respondents as with other farm owners. The Court was thus
complainants alleged inter alia that they are emphatic when it ruled that petitioners were mere
regular and permanent workers of project employees, who could be hired by other
the hacienda and that they were dismissed without farm owners.[26]
just and lawful cause. They further alleged that they
were dismissed because they applied as Recently, the Court reiterated the same
beneficiaries under the Comprehensive Agrarian observations in Hacienda Fatima v. National
Federation of Sugarcane Workers-Food and
Page 57 of 5
General Trade[27] and added that the petitioners in Financial Officer and Chief Actuary; and Anthony
the Mercado case were not hired regularly and Sotelo as the Senior Vice-President and Head of the
repeatedly for the same phase/s of agricultural Human Resources Department.
work, but on and off for any single phase thereof. Petitioner was employed on October 28, 1997 by
In this case, there is no evidence on record that the private respondent as Assistant Vice President and
same particulars are present. The petitioners did not Head of the Pensions Department and in
present any evidence that the respondents were concurrent capacity as Trust Officer of Philam
required to perform certain phases of agricultural Savings Bank, a Philam Life subsidiary. She was to
work for a definite period of time. Although the be paid P750,000.00 per annum and is entitled to
petitioners assert that the respondents made their the benefits given by private respondent to its
services available to the neighboring haciendas, employees.
the records do not, however, support such Working as Assistant Vice President of Pensions
assertion. Department of Philamlife, petitioner was offered an
additional position by respondent Cuisia, which was
The primary standard for determining regular then resolved and approved by Philam Savings
employment is the reasonable connection Banks Board of Directors, for the position of Head of
between the particular activity performed by the Trust Banking Division or AVP-Trust Officer on a
employee in relation to the usual trade or business concurrent capacity and under a separate
of the employer.[28] There is no doubt that the compensation.
respondents were performing work necessary and
desirable in the usual trade or business of an Effective January 1998, however, petitioners
employer. Hence, they can properly be classified as marketing manager and marketing officer were
regular employees. immediately transferred to Group Insurance
Division. Petitioner, thereafter, was never given
For respondents to be excluded from those replacements for the marketing manager and
classified as regular employees, it is not enough that marketing officer, contrary to private respondent
they perform work or services that are seasonal in Cuisias assurance. Thus, petitioner ran the Pensions
nature. They must have been employed only for the Department single-handedly with only one
duration of one season.[29] While the records administrative assistant as her staff. Petitioner did
sufficiently show that the respondents work in the field work, the desk work (administrative, legal,
the hacienda was seasonal in nature, there was, finance, marketing), the out of town meetings, the
however, no proof that they were hired for the client presentations, aside from her work with the
duration of one season only. In fact, the Philam Savings Bank as fund manager, wherein
payrolls,[30] submitted in evidence by the petitioners, private respondent Cuisia offered to her for a
show that they availed the services of the separate compensation, but has still remain [sic]
respondents since 1991. Absent any proof to the unpaid.
contrary, the general rule of regular employment
should, therefore, stand. It bears stressing that the Sometime in November, 1998, petitioner availed of
employer has the burden of proving the lawfulness her housing and car benefits and applied for a car
of his employees dismissal.[31] loan and housing loan.

On the procedural issue, petitioner Starke avers that On November 18, 1998, however, private
the CA should not have denied outright her motion respondent through Centeno and Sotelo, offered
for reconsideration, considering its timely filing and her P250,000.00 for her to vacate her position by
the huge amount involved. This contention is December 1998. Petitioner declined the offer
already moot. Petitioner Starke has already aired in considering that there was no valid reason for her
this petition the arguments in her motion for to leave. Private respondents Centeno and Sotelo
reconsideration of the CA decision, which have admonished her that her filing of suit would prompt
been adequately addressed by this Court. respondent Cuisia to blacklist her in companies
Assuming arguendo that the CA indeed failed to where he holds directorships and advised her that
consider the motion for reconsideration, petitioner Philamlife is big and can stand the long ordeal of
Starke was not left without any other recourse.[32] justice system, whereas she may not withstand the
IN LIGHT OF ALL THE FOREGOING, the petition is phase of the trial. Evidence that this meeting and
DENIED. matter took place was the formal letter of rejection
dated November 25, 1998 sent by petitioner and
duly received by the offices of respondents Cuisia,
THE PHILIPPINE AMERICAN LIFE AND GENERAL Centeno and Sotelo.
INSURANCE CO., petitioner, vs. ANGELITA S.
GRAMAJE, respondent. Pertinent portion of the November 25, 1998 letter is
hereby quoted:
Facts: Private respondent Philippine American Life
and General Insurance Company is a corporation [T]his shall summarize the discussion of meeting held
duly organized and existing under Philippine laws. at Mr. Centenos Office last November 18, 1998.
Individual respondents occupy the following Briefly, an offer of Two Hundred Fifty Thousand
positions, namely: Maurice Greenberg, as president [Pesos] (P250,000) has been made as Settlement
of the Company; Jose Cuisia, Jr. as Chairman of the fee so that Philamlife will not resort to transferring
Board; Maria Haas and Gardon Watson as Regional undersigned to another department for reasons
Coordinating Pensions Officers, Reynaldo C. only known to management and which
Centeno as Executive Vice-President, Chief undersigned is still not fully aware in writing. In so
Page 58 of 5
doing, it has been emphasized that Mr. Centeno This refers to your 9 December 1998 memorandum
and Mr. Sotelo is (sic) sparing undersigned of the regarding your request for a car loan. I have earlier
hardships that undersigned will undergo in the said discussed your application for a car loan with both
other department which is intended to make Mr. Anthony B. Sotelo, FVP and Corporate HR
undersigned inefficient and eventually serve as Director and Mr. Jose L. Cuisia, Jr., President and
basis for her termination or as claimed non-election CEO. Considering your present employment status,
by March 1999. Further, it has been requested and which has been the subject of several discussions
categorically stated by Mr. Sotelo that undersigned between you and Messrs. Jose L. Cuisia, Jr. and
forgive Maria Haas for whatever she has done Anthony B. Sotelo and myself, we deem it prudent
to defer action on your loan request until such time
On December 6, 1998, respondent Cuisia met with that the issue is resolved with definitiveness.
petitioner and cajoled her to reconsider and
accept the offer of settlement. Cuisia even On December 16, 1998, petitioner, while on Official
volunteered to help her look for another job. Sick Leave, received a message in her pager that
Petitioner declined, and reiterated that the the Pensions Department, which was then holding
actuations of respondents clearly intended to office at the fifth floor of the Philamlife Building at
harass and humiliate her and have caused her and United Nations Avenue was assumed to be headed
her family extreme emotional stress. by Corine Moralda as her successor, and the
Pensions Department was to be immediately
On December 8, 1998, two days after the aforesaid physically transferred on said date at the Philamlife
December 6 meeting, respondents issued her a Gammon Center in Makati City. Though sick and on
memorandum instructing her to transfer to the official sick leave, petitioner went to the office on
Legal Department effective December 14, 1998 December 17, 1998 to verify, and upon seeing the
and to make proper turnover and submit the status Pensions Department totally dark, without any staff
report not later than December 11, 1998. and with left over fixtures, petitioner, emotionally
shattered, opted to just leave the premises.
By her letter dated December 10, 1998, petitioner On December 18, 1998, respondent Cuisia through
protested the sudden unexplained transfer, more so a memorandum appointed Ms. Corine Moralda as
a non-existing position, and stressed that she was replacement of petitioner as Head of the Pensions
hired because of her marketing, finance, and fund Department effective December 14, 1998. It was
management skills, not her legal skills. She also only at that time that petitioner learned that as
made of record that her department surpassed the early as August 23, 1998, respondents had
target fund level volume set by the company, thus: advertised in the Manila Bulletin for her
Undersigned wish to inform you that your directive replacement.
for the transfer of undersigned to the legal Also, although, it is the tradition of Philamlife to give,
department is being contested on the ground of during the Christmas Season, officers and
outright violation of undersigneds rights. employees a traditional Seasons giveaways, i.e.,
ham and queso de bola, petitioner then, thru her
Undersigned believe that the transfer will not make authorized representatives, asked for her share, but
her efficient in her work. Undersigned was hired she was not in the list of recipients. Petitioners name
primarily because of her marketing, finance, and was not in the Legal Department, not in the
fund management skills. Her legal skills are Pensions Department, and not in the list of
secondary and supplementary in nature. Thus, employees of Philamlife when verified with the
transfer to the legal department, which is primarily Personnel Department.
legal, is not acceptable for it will only make
undersigned less efficient and negates her Hence, on December 23, 1998, petitioner filed the
productivity and contribution to the company. instant case for illegal or constructive dismissal
against herein private respondents.
Let it be on record, that as of today, the
Department has surpassed its P15 Million target, The Labor Arbiter, in his decision[4] dated 01 June
which was originally at P12 Million, as set by no less 2000, found that respondent was not illegally
than the president of Philamlife during the budget dismissed.
preparation and as duly reviewed and approved
by the head of the corporation planning ISSUE: WON the respondent was constructively
department, as fully documented. For the records, dismissed.
we are almost hitting the P20 Million fund level
volume, and we are just waiting for the confirmed RULING: Yes. Constructive dismissal exists when an
P109 Million placement of Adamson University act of clear discrimination, insensibility or disdain by
Retirement Fund. an employer has become so unbearable to the
employee leaving him with no option but to forego
With the above, by December 14, 1998 with his continued employment.[32] The
undersigned will continue to be the head of the circumstances which prevailed in the working
Pensions Department until this new issue and the environment of the respondent clearly demonstrate
other issues raised are fully resolved. this. The failure of the Labor Arbiter to resolutely
consider these prevailing circumstances before
Atty. Angelita S. Gramaje respondent was asked to transfer was a major flaw
AVP-Pensions Department in his decision. Clearly, had the Labor Arbiter
Also, on December 10, 1998, respondent Centeno considered them, he would have concluded that
declined the car loan benefit of petitioner, thus: the transfer of respondent from the Pensions
Page 59 of 5
Department to the Legal Department was not a petitioner by abandoning her work and failing to
legitimate exercise of management prerogative on report accordingly. This argument is apocryphal.
the part of petitioner. Before the order to transfer Respondent, on 23 December 1998, already filed a
was made, discrimination, bad faith, and disdain case for illegal dismissal against petitioner.[37] For
towards respondent were already displayed by petitioner to anticipate respondent to report for
petitioner. work after the latter already filed a case for illegal
dismissal before the NLRC, would be absurd. We
Petitioner has repeatedly asserted that the have already laid down the rule that for
performance of respondent did not meet the abandonment to exist, it is essential (1) that the
expectation of the company and did not comply employee must have failed to report for work or
with accepted standards for a pension profit center must have been absent without valid or justifiable
manager, as she lacked the skill, as well as the reason; and (2) that there must have been a clear
willingness, to perform her duties and responsibilities. intention to sever the employer-employee
Allegedly, based on the evaluation of her relationship manifested by some overt acts.[38]Both
performance, respondent proved to be so inept in these requisites are not present here. There was no
the performance of her obligations, viz: abandonment as the latter is not compatible with
a. Failure to prepare and submit a budget plan; constructive dismissal.[
b. Failure to prepare and submit a Pension
Production Report on time;
c. Strained relations with clients; DUNCAN ASSOCIATION OF DETAILMAN-PTGWO and
d. Failure to prepare an Operations Manual for the PEDRO A. TECSON, petitioners,
Department; vs. GLAXO WELLCOME PHILIPPINES,
e. Inability to develop and maintain a good INC., Respondent.
working relationship with her colleagues;
f. Inability to communicate her ideas; and FACTS: Petitioner Pedro A. Tecson (Tecson) was
g. Others.[33] hired by respondent Glaxo Wellcome Philippines,
Inc. (Glaxo) as medical representative on October
It is rather peculiar that the alleged ineptness of 24, 1995, after Tecson had undergone training and
respondent did not prompt petitioner to issue any orientation.
Inter-office Memorandum reprimanding, Thereafter, Tecson signed a contract of
admonishing, or warning the former about her employment which stipulates, among others, that
performance. The solemnity of respondents alleged he agrees to study and abide by existing company
non-performance was so immense, considering rules; to disclose to management any existing or
that the Pensions Department is a profit center, future relationship by consanguinity or affinity with
which was so imperative to the existence of co-employees or employees of competing drug
petitioner in terms of raising revenue. The officers of companies and should management find that such
petitioner should have been very much troubled relationship poses a possible conflict of interest, to
about this. resign from the company. Code of Conduct of
Glaxo similarly provides these conditions; that
This now puts into question the alleged ineptness of otherwise, the management and the employee will
respondent as posited by petitioner. As aptly explore the possibility of a transfer to another
declared by the Court of Appeals: department in a non-counterchecking position or
. . . We recall that what triggered petitioners transfer preparation for employment outside the company
was her alleged inefficiency and ineptness in her after six months.
work in the Pensions Department. Records, Tecson was initially assigned to market Glaxos
however, reveal otherwise. Petitioner produced a products in the Camarines Sur-Camarines Norte
fund level of 1000% over the previous year (her sales area. Subsequently, Tecson entered into a
predecessors year of 1997 with a fund level of romantic relationship with Bettsy, an employee of
about P2 Million generated for two years or an Astra Pharmaceuticals3(Astra), a competitor of
average of P1 Million per year then) in the amount Glaxo. Bettsy was Astras Branch Coordinator in
of P19,248,320.31 as a result of a meager 3 months Albay. She supervised the district managers and
marketing efforts, although private respondents medical representatives of her company and
instructed her to stop marketing sometime in April prepared marketing strategies for Astra in that area.
1998 for no apparent reason. All these were never
rebutted nor disproved by private respondents. Even before they got married, Tecson received
They merely alleged her inefficiency without several reminders from his District Manager
concrete and sufficient proof. But allegation is regarding the conflict of interest which his
different from proof. Hence, we cannot relationship with Bettsy might engender. Still, love
countenance their allegations.[34] (emphasis ours) prevailed, and Tecson married Bettsy in September
1998.
Petitioner maintains that it was respondent who
severed her working relationship with it.[35] Per letter, Tecsons superior reminded him that he and Bettsy
dated 11 January 1999, issued by petitioners Legal should decide which one of them would resign
Department, respondent was asked to report from their jobs. Tecson requested for time to comply
immediately to her new assignment and submit to a with the company policy against entering into a
medical examination, and that the latter took no relationship with an employee of a competitor
heed of this.[36] It seems that the point impliedly company. He explained that Astra, Bettsys
being raised by petitioner is that respondent employer, was planning to merge with Zeneca,
disengaged her employment relationship with another drug company; and Bettsy was planning to
Page 60 of 5
avail of the redundancy package to be offered by under the circumstances because relationships of
Astra. that nature might compromise the interests of the
company. In laying down the assailed company
Tecson again requested for more time resolve the policy, Glaxo only aims to protect its interests
problem. Thereafter, Tecson applied for a transfer in against the possibility that a competitor company
Glaxos milk division, thinking that since Astra did will gain access to its secrets and procedures.
not have a milk division, the potential conflict of
interest would be eliminated. His application was That Glaxo possesses the right to protect its
denied in view of Glaxos least-movement- economic interests cannot be denied. No less than
possible policy. the Constitution recognizes the right of enterprises
to adopt and enforce such a policy to protect its
Glaxo transferred Tecson to the Butuan City-Surigao right to reasonable returns on investments and to
City-Agusan del Sur sales area. Tecson asked Glaxo expansion and growth.
to reconsider its decision, but his request was
denied. Tecson defied the transfer order and Indeed, while our laws endeavor to give life to the
continued acting as medical representative in the constitutional policy on social justice and the
Camarines Sur-Camarines Norte sales area. protection of labor, it does not mean that every
labor dispute will be decided in favor of the
DEVELOPMENT OF THE CASE: Because the parties workers. The law also recognizes that management
failed to resolve the issue at the grievance has rights which are also entitled to respect and
machinery level, they submitted the matter for enforcement in the interest of fair play.21
voluntary arbitration, but Tecson declined the offer.
On November 15, 2000, the National Conciliation EQUAL-PROTECTION: Glaxo does not impose an
and Mediation Board (NCMB) rendered absolute prohibition against relationships between
its Decision declaring as valid Glaxos policy on its employees and those of competitor companies.
relationships between its employees and persons Its employees are free to cultivate relationships with
employed with competitor companies, and and marry persons of their own choosing. What the
affirming Glaxos right to transfer Tecson to another company merely seeks to avoid is a conflict of
sales territory. interest between the employee and the company
that may arise out of such relationships.
CA sustained; MR denied.
Moreover, records show that Glaxo gave Tecson
Petitioners Contention: that Glaxos policy against several chances to eliminate the conflict of interest
employees marrying employees of competitor brought about by his relationship with Bettsy.
companies violates the equal protection clause of
the Constitution because it creates invalid PETITION DENIED.
distinctions among employees on account only of
marriage. They claim that the policy restricts the
employees right to marry; that Tecson was d. Guidelines on Imposition of Penalty
constructively dismissed
GLAXO argues: that the company policy prohibiting Associated Labor Union v. NLRC, 302 SCRA
its employees from having a relationship with 708 (1999)
and/or marrying an employee of a competitor
company is a valid exercise of its management FACTS:
prerogatives and does not violate the equal This is a petition for certiorari to annul the
protection clause; decision of the National Labor Relations
The policy is also aimed at preventing a competitor Commission in NLRC NCR 00-09-06075-93 on the
company from gaining access to its secrets, ground that it was rendered with grave abuse
procedures and policies; that Tecson can no longer of discretion.
question the assailed company policy because
when he signed his contract of employment, he Petitioner Renato Felizardo was employed
was aware that such policy was stipulated therein. at respondent Republic Flour Mills-Selecta Ice
Cream Corporation as jet printer operator on
ISSUE: WON Glaxos policy against its employees November 16, 1991. He was dismissed from
marrying employees from competitor companies is employment on September 13, 1993 for
valid dishonesty and theft of company property. He
and petitioner Associated Labor Union-TUCP, of
HELD: The Court finds no merit in the petition. which he is the secretary, filed a complaint for
illegal dismissal, unfair labor practice, and non-
Glaxo has a right to guard its trade secrets, payment of 13th month pay against respondent
manufacturing formulas, marketing strategies and company. He was ordered reinstated without
other confidential programs and information from backwages by the Labor Arbiter, but the NLRC
competitors, especially so that it and Astra are rival reversed the said decision and dismissed his
companies in the highly competitive complaint. Hence, this petition.
pharmaceutical industry.
The Labor Arbiter found that with the
The prohibition against personal or marital exception of the pair of boots, the articles
relationships with employees of competitor which petitioner took from the company were
companies upon Glaxos employees is reasonable mere scraps which were of no value to
Page 61 of 5
respondent company. He ruled that dismissal offense. Consequently, the Labor Arbiters order of
was too harsh a penalty to be imposed on a reinstatement of petitioner without backwages may
first-time offender and that his unemployment be considered appropriate.
for about eleven (11) months was sufficient
penalty for what he had done. Accordingly, the The instant petition is GRANTED. The decision of the
Labor Arbiter ordered petitioners reinstatement National Labor Relations Commission is hereby SET
without backwages. ASIDE and that of the Labor Arbiter is REINSTATED.

On appeal, the NLRC reversed the LAs


decision. Philippine Long Distance Telephone Company v.
NLRC, 303 SCRA 9 (1999)
ISSUE: Whether or not Renato Felizardo was
illegally dismissed. FACTS:
This petition for certiorari under Rule 65 of
RULING: YES. There is no question that the the Rules of Court assails the Resolution
employer has the inherent right to discipline, dated June 29, 1992 of the National Labor
including that of dismissing its employees for just Relations Commission ordering petitioner to
causes. This right is, however, subject to reinstate private respondent, Enrique
reasonable regulation by the State in the Gabriel, with full backwages, benefits, and
exercise of its police power.[5] The finding of the proportionate privileges; as well as the
NLRC that an employee violated the company Order dated August 19, 1992, denying the
rules and regulations is subject to scrutiny by the motion for reconsideration. The challenged
Court to determine if the dismissal is justified ruling reversed the decision of the Labor
and, if so, whether the penalty imposed is Arbiter dated May 3, 1991, which dismissed
commensurate to the gravity of his offense. the complaint for lack of merit.

In this case, the Court agree with the Labor Arbiter Private respondent, Enrique Gabriel, was
that dismissal would not be proportionate to the employed by petitioner Philippine Long
gravity of the offense committed by petitioner Distance Telephone Company (PLDT), as a
considering the value of the articles he pilfered and foreman in Dansalan Area 2, M-3. As a
the fact that he had no previous derogatory record supervisor, his territorial responsibility
during his two (2) years of employment in the covered Camp Crames First to 20th
company. The Labor Arbiter is certainly mistaken in Avenues and portions of Project 4, all in
regarding the articles taken to be mere scraps and Quezon City.
hence without value to the company. They were of On September 5, 1989, Enrique Gabriel
some value but not enough to warrant dismissal. ordered Medel Mercado, an installer, to set-
up two telephone units at Unit R, Facilities
Moreover, it should also be taken into account that Center Building, located at Shaw Boulevard,
petitioner is not a managerial or confidential Mandaluyong, Metro Manila. The telephone
employee in whom greater trust is placed by numbers of the units were 78-88-41 and 79-
management and from whom greater fidelity to 98-46, in favor of a certain Mr. Marlon
duty is correspondingly expected. It is easy to see Aquino.
why an unfaithful employee who is holding a On October 16, 1989, private respondent,
position of trust and confidence in a company again ordered Juancho Jocson, another
poses a greater danger to its security than a mere installer, to set-up additional units with
clerk or machine operator like petitioner. telephone numbers 78-40-70 and 79-40-98
for the same subscriber.
There is another reason why violations by non- Later, both installation activities were
confidential employees of company rules and investigated because (a) the Facilities
regulations such as that involved in this case are Center Building had no entrance cable
considered minor. Such employees are generally facilities or conduit wires for telephone
mere wage earners whose dismissal from connection, (b) Mandaluyong was not
employment can have severe financial within respondents area of jurisdiction, and
consequences on their families especially at a time (c) installers Mercado and Jocson were not
like the present when unemployment is quite under his direct supervision.
high. Consequently, whatever missteps may have In the administrative investigation
been committed by them ought not to be visited conducted by PLDT, where a confrontation
with a consequence so severe as dismissal. between private respondent and installers
Mercado and Jocson took place, private
Dismissal as a measure to protect the interests of respondent tried to explain his side to clear
respondent company is unwarranted under the certain issues taken against him, adding
facts of this case. Suspension would have that his intention in ordering the installation
sufficed. Without deciding for how long the of the telephone units was to provide
suspension should be in cases such as this, customer satisfaction. However, on February
considering that petitioner has been prevented 1, 1990, the petitioner still required the
from working in respondent company since private respondent to submit a written
September 13, 1993, we hold that, for all purposes, report of the incident. He submitted his
he has served a reasonable period of suspension explanation and, although admitting the
commensurate to the gravity of his responsibility of his actions, reiterated his
Page 62 of 5
rationalization that his sole intention was to were not under his direct supervision. Yet based on
serve the customer, thereby earning his instruction, he secured OK numbers for the
goodwill for the company. telephones and performed call back at the panel
On September 3, 1990, private respondent box while misrepresenting himself to the dispatch
was dismissed from employment on the clerk as the subscriber.
ground that he committed grave Despite these circumstances, the NLRC reversed
misconduct, breach of trust, and violations the Labor Arbiters order of dismissal imposed
of company rules and regulations when he against private respondent Enrique
ordered the unwarranted installation Gabriel. According to the NLRC, it found no written
activities. rule of PLDT which provides that such unwarranted
On September 6, 1990, private respondent, installation of telephone lines is subject to the
as complainant below filed an illegal penalty of dismissal. Nor was there any proof that
dismissal case against herein petitioner, the private respondent profited from the said
PLDT. setting up of telephone lines. Neither was there a
showing that PLDT suffered losses from the
ISSUES: 1. Whether or not Gabriel is guilty of serious telephone service in favor of Mr. Marlon
misconduct and/or breach of trust anent the Aquino. Finally, as claimed by the private
irregular installation of the afore-numbered respondent in his memorandum, which was not
telephones. rebutted on this point by the petitioner, the subject
telephones were installed only after the documents
2. Whether or not NLRC abused its discretion of approval were issued by PLDT. Given these
amounting to lack or excess of jurisdiction in circumstances, a substantial doubt as to the validity
reversing the decision of the Labor Arbiter, of the termination appears, and the employees
and ordering the reinstatement of private claim of illegal dismissal accordingly gains
respondent with full back wages and other credence because such doubt must be resolved in
benefits. his favor.

RULING: 1. NO. At the outset, it must be recalled 2. NO. Assailed NLRC Resolution is AFFIRMED
that the basic requisite for dismissal on the ground with MODIFICATION. Petitioner is directed to
of loss of confidence is that the employee reinstate the private respondent to his position held
concerned must be one holding a position of trust at the time of the complained dismissal. Petitioner is
and confidence. However, loss of confidence must likewise ordered to pay private respondent his full
not be indiscriminately used as a shield by the backwages including thirteenth month pay due
employer against a claim that the dismissal of an him based on his last salary, computed from the
employee was arbitrary. date of promulgation of the NLRC Resolution on
June 29, 1992, until his actual reinstatement.The
Likewise, it must be noted that willful defiance of other awards of unspecified benefits and
company rules must be characterized by perverse proportionate privileges in the said Resolution are
attitude that would be considered as inimical to the set aside for lack of merit.
interest of his employer. Even when an employee is
found to have transgressed the employers rules, in The Court have to concede that in ordering the
the actual imposition of penalties upon the erring reinstatement of private respondent, Gabriel, the
employee, due consideration must still be given to public respondent, NLRC, is not entirely without
his length of service and the number of violations good and justifiable reason. Thus it could not be
committed during his employ. said that this portion of the assailed Resolution of
the NLRC is tainted with grave abuse of discretion.
Dismissal is the ultimate penalty that can be meted
to an employee. Where a penalty less punitive Now, since Gabriels dismissal has been found to be
would suffice, whatever missteps may have been of doubtful justification in law and policy, the award
committed by the worker ought not to be visited by the NLRC of full backwages in his favor could not
with a consequence so severe such as dismissal be said as erroneous. It has to be sustained, but
from employment. For, the Constitution guarantees only from the date of the NLRCs promulgation of its
the right of workers to security of tenure. The misery Resolution on June 29, 1992.
and pain attendant to the loss of jobs then could
be avoided if there be acceptance of the view Backwages including thirteenth month pay are a
that under certain circumstances of the case the form of relief that restores the income that was lost
workers should not be deprived of their means of by reason of unlawful dismissal. Note that in contrast
livelihood. to the MERALCO case, infra, where the award of
backwages was not granted because the
In the present case, there is no dispute that the employer was in good faith when it dismissed the
private respondent ordered the installation of the employee who received P7,000.00 purportedly in
telephone units in favor of Marlon Aquino, a consideration of his facilitation of the electrical
telephone subscriber. As found by the Labor Arbiter, connection in favor of an applicant, in the case at
private respondents orders were irregular. The bar the installation of four telephone units was
orders of telephone connection were pursued even not per se illegal because of the previous approval
if there were no entrance cable facilities for by the PLDT of these units for installation. The
telephone connection. Moreover, Mandaluyong connections were only irregular because they
was not within the area of private respondents violated certain standard operating procedures of
jurisdiction. The installers, Mercado and Jocson, PLDT. Private respondent in this case had facilitated
Page 63 of 5
the service connections of the telephone units reinstated considering that the period of his
irregularly but he was apparently of the wrong preventive suspension had expired.
impression that he was cultivating a customers
goodwill. He did not receive any monetary or other Sometime in September 1995, petitioner
material benefit at all for the facilitation. Thus, on manifested to RCPI his willingness to settle his
one hand, the cited irregularity here could not case provided he is given his retirement
justify the ultimate penalty of dismissal from benefits. However, RCPI informed petitioner that
employment. And, in our view, reinstatement (with his employment had already been terminated
full backwages and thirteenth month pay earlier as contained in the letter dated
computed beginning June 29, 1992 until actual November 22, 1993. The conflict was submitted
reinstatement) would provide sufficient relief for the to the grievance committee. Despite the lapse
loss suffered by him. of more than two years, the case remained
unresolved before the grievance committee,
On the other hand, the award of unspecified other hence, it was submitted for voluntary arbitration.
benefits and proportionate privileges to the private
respondent by the NLRC appears to us already ISSUE: Whether or not Farrol was illegally dismissed.
unwarranted. Private respondent is not entirely
faultless. As a supervisor, he is required to act RULING: YES. The assailed decision of the Court of
judiciously and to exercise his authority in harmony Appeals is REVERSED and SET ASIDE and new one
with company policies. When he jeopardized the entered REINSTATING the decision of the Voluntary
status of the rank-and-file employees whom he Arbitrator subject to the MODIFICATION that
ordered to by-pass the standard operating petitioners separation pay be recomputed to
procedures of the company, to the detriment of his include the period within which backwages are
employer, he was not entirely blameless. The due. For this purpose, this case is REMANDED to the
irregularity attributable to him could not be entirely Voluntary Arbitrator for proper computation of
disregarded. He must not be further rewarded, in backwages, separation pay, 13th month pay, sick
fairness to the employers own legitimate concerns leave conversion and vacation leave conversion.
such as company morale and discipline. Anent the procedural requirement, Book V,
Rule XIV, of the Omnibus Rules Implementing the
Farrol v. Court of Appeals, 435 SCRA 543 (2000) Labor Codeexisting at the time petitioner was
discharged from work, outlines the procedure
FACTS: Petitioner Wenifredo Farrol was for termination of employment, to wit:
employed as station cashier at respondent "Sec. 1. Security of tenure and due process. -
RCPIs Cotabato City station. On June 18, 1993, No worker shall be dismissed except for a just or
respondent RCPIs district manager in Cotabato authorized cause provided by law and after due
City informed their main office that "Peragram process.
funds" from said branch were used for the "Sec. 2. Notice of Dismissal. - Any employer
payment of retirement benefits of five who seeks to dismiss a worker shall furnish him
employees. On October 1, 1993, petitioner a written notice stating the particular acts or
verified as correct RCPIs Field Auditors report omissions constituting the grounds for his dismissal. In
that there was a shortage of P50,985.37 in their cases of abandonment of work, the notice shall be
branchs Peragram, Petty and General Cash served at the workers last known address.
Funds. Consequently, petitioner was required by xxxxxxxxx
the Field Auditor to explain the cash shortage "Sec. 5. Answer and hearing. - The worker
within 24 hours from notice. The next day, may answer the allegations stated against him in
petitioner paid to RCPI P25,000.00 of the cash the notice of dismissal within a reasonable period
shortage. from receipt of such notice. The employer shall
afford the worker ample opportunity to be heard
On October 16, 1993, RCPI required petitioner to and to defend himself with the assistance of his
explain why he should not be dismissed from representatives, if he so desires.
employment. Two days thereafter, petitioner "Sec. 6. Decision to dismiss. - The employer
wrote a letter to the Field Auditor stating that shall immediately notify a worker in writing of a
the missing funds were used for the payment of decision to dismiss him stating clearly the
the retirement benefits earlier referred to by the reasons therefor.
branch manager and that he had already paid "Sec. 7. Right to contest dismissal. - Any
P25,000.00 to RCPI. After making two more decision taken by the employer shall be without
payments of the cash shortage to RCPI, prejudice to the right of the worker to contest the
petitioner was informed by the district manager validity or legality of his dismissal by filing a
that he is being placed under preventive complaint with the Regional Branch of the
suspension. Thereafter, he again paid two more Commission.
sums on different dates to RCPI leaving a xxxxxxxxx
balance of P6,995.37 of the shortage. "Sec. 11. Report on dismissal. - The employer
shall submit a monthly report to the Regional Office
Respondent RCPI claims that it sent a letter to having jurisdiction over the place of work all
petitioner on November 22, 1993 informing him dismissals effected by him during the month,
of the termination of his services as of specifying therein the names of the dismissed
November 20, 1993. Unaware of the termination workers, the reasons for their dismissal, the dates of
letter, petitioner requested that he be commencement and termination of employment,
the positions last held by them and such other
Page 64 of 5
information as may be required by the Ministry Valiao v. Court of Appeals, 435 SCRA 543 (2004)
(Department) for policy guidance and statistical
purposes." (Underscoring supplied). FACTS: For review on certiorari is the Decision
As set forth in the foregoing procedures, the dated August 22, 2000 of the Court of Appeals
employer must comply with the twin requirements in CA-G.R. SP No. 55133, and its Resolution
of two notices and hearing. The first notice is that dated November 22, 2000 denying the motion
which apprises the employee of the particular acts for reconsideration. The Court of Appeals
or omissions for which his dismissal is sought, and dismissed the petition for certiorari filed by
after affording the employee an opportunity to be petitioner and affirmed the Resolution dated
heard, a subsequent notice informing the latter of July 7, 1999 of the National Labor Relations
the employers decision to dismiss him from work. Commission (NLRC)-Fourth Division in NLRC Case
As regards the first notice, RCPI simply No. V-000134-98 (RAB Case No. 06-01-10026-95),
required petitioner to "explain in writing why he which sustained the Decision of Labor Arbiter
failed to account" for the shortage and demanded Benjamin E. Pelaez, directing private
that he restitute the same. On the assumption that respondent West Negros College (WNC) to pay
the foregoing statement satisfies the first notice, the petitioner Rene P. Valiaos salary during the
second notice sent by RCPI to petitioner does not period of his preventive suspension and
"clearly" cite the reasons for the dismissal, contrary attorneys fees, while dismissing all other claims.
to the requirements set by the above-quoted
Section 6 of Book V, Rule XIV of the Omnibus Rules. Rene P. Valiao was appointed by private
A perusal of RCPIs dismissal notice reveals respondent West Negros College (WNC) as
that it merely stated a conclusion to the effect that Student Affairs Office (SAO) Director, with a
the withholding was deliberately done to hide starting salary of P2,800 per month. On May 14,
alleged malversation or misappropriation without, 1990, he was assigned as Acting Director,
however, stating the facts and circumstances in Alumni Affairs Office. He was transferred to a
support thereof. It further mentioned that the staff position and designated as Records Chief
position of cashier requires utmost trust and at the Registrars Office but was again re-
confidence but failed to allege the breach of trust assigned as a typist. The latest re-assignment
on the part of petitioner and how the alleged was due to his tardiness and absences, as
breach was committed. On the assumption that reflected in the summary of tardiness and
there was indeed a breach, there is no evidence absences report, which showed him to have
that petitioner was a managerial employee of been absent or late for work from a minimum of
respondent RCPI. It should be noted that the term seven (7) to a maximum of seventy-five (75)
"trust and confidence" is restricted to managerial minutes for the period March to October 31,
employees. It may not even be presumed that 1991, and to have reported late almost every
when there is a shortage, there is also a day for the period November to December
corresponding breach of trust. Cash shortages in a 1991.
cashiers work may happen, and when there is no
proof that the same was deliberately done for a Copies of his tardiness/absences reports
fraudulent or wrongful purpose, it cannot constitute were furnished petitioner, along with
breach of trust so as to render the dismissal from memoranda requiring him to explain but his
work invalid. explanations were either unacceptable or
Assuming further that there was breach of unsatisfactory. Subsequent reports also showed
trust and confidence, it appears that this is the first that he did not change his habits resulting in
infraction committed by petitioner. Although the tardiness and absences. He was even caught
employer has the prerogative to discipline or dismiss one time manipulating the bundy clock, thus
its employee, such prerogative cannot be necessitating another memorandum to him
exercised wantonly, but must be controlled by asking him to explain his dishonest actuations in
substantive due process and tempered by the accomplishing the daily attendance logbook
fundamental policy of protection to labor enshrined and in using the bundy clock.
in the Constitution. Infractions committed by an
employee should merit only the corresponding On January 18, 1993, petitioner was again
sanction demanded by the circumstances. The absent from work without permission or notice
penalty must be commensurate with the act, to his immediate superior. It turned out that he
conduct or omission imputed to the went to Bacolod City and on January 28, 1993,
employee[17] and imposed in connection with the the petitioner was one of those arrested during
employers disciplinary authority. a raid in the house of one Toto Ruiz, a
The Court thus holds that the dismissal suspected drug pusher and was brought to the
imposed on petitioner is unduly harsh and grossly Bacolod Police Station along with four (4) other
disproportionate to the infraction which led to the suspects. Upon further search and investigation
termination of his services. A lighter penalty would by the Narcotics Control Division, the petitioner
have been more just, if not humane. In any case, was found possessing two (2) suspected
petitioner paid back the cash shortage in his marijuana roaches (butts) which were placed
accounts. Considering, however, that the latter is inside his left shoe. The event was widely
about to retire or may have retired from work, it publicized, focusing on petitioners position as an
would no longer be practical to order his Economics teacher of WNC, and considering
reinstatement. further that one of his fellow suspects was a
member of the Philippine Army, who was
caught with an unlicensed firearm, a tooter and
Page 65 of 5
other shabu paraphernalia. The petitioner and
other suspects were then charged with violation FACTS: Vicente C. Ectuban, Jr. was employed by
of the Dangerous Drugs Act of 1972 (Republic the respondent on January 30, 1978 until his
Act No. 6425, as amended). dismissal on June 10, 1994 for loss of trust and
confidence. At the time of his dismissal, the
WNC alleged that petitioner was dismissed petitioner was the Chief Purser of the M/V Surigao
on charges of serious misconduct, and gross Princess receiving a monthly salary of P5,000.00. As
and willful neglect of duty. WNC said his the Chief Purser, the petitioner handled the funds of
dismissal was effected after due notice and the vessel and was the custodian of all the passage
prior hearing. It claimed also that since tickets and bills of lading. It was his responsibility,
petitioner was terminated for a valid cause after among other things, to issue passage tickets and to
a due hearing, the latters claim for moral and receive payments from the customers of the
exemplary damages, and attorneys fees had respondent, as well as to issue the corresponding
no basis in fact and in law. official receipts therefor. He was also tasked to
disburse the salaries of the crewmen of the vessel.
ISSUE: Whether or not Valiao was illegally dismissed.
Sometime in the last week of May 1994, the newly
RULING: NO. The Labor Arbiter found no justifiable designated jefe de viaje of the M/V Surigao
reason to place the petitioner under preventive Princess, in a surprise examination, discovered that
suspension as there was no serious or imminent several yellow passengers duplicate original of yet
threat to the life or property of his employer or co- to be sold or unissued passage tickets already
workers. contained the amount of P88.00 the fare for adult
passengers for the Cagayan de Oro to Jagna,
However, the Labor Arbiter found the dismissal of Bohol route. He noticed that three other original
the petitioner from WNC to be valid due to copies which made up the full set did not bear the
absenteeism and tardiness and after he was same impression, although they were supposed to
accorded the procedural due process aspect of have been prepared at the same time. Acting on
the law as reflected in the records showing that the what appeared to be a strong evidence of short-
petitioner was formally investigated and given the changing the company, the jefe de viaje dug
opportunity to refute the alleged findings by the deeper on what he uncovered. As expected, he
management of WNC. The Labor Arbiter held that found inordinate amount of ticket issuances for
frequent absenteeism and tardiness of the children at half the fare of P44.00 in Voyage 434 of
petitioner constituted not only willful disobedience the vessel. When word of the anomaly reached the
but also gross and habitual neglect of duties, which respondent, it waited for the petitioner to return to
are valid grounds for termination of Cebu City in the hope of shedding more light on
employment. He stressed that the petitioners the matter.
frequent absences without proper leave of
absence was not only unfair to WNC and the On May 30, 1994, shortly after disembarking
petitioners co-employees but also set an from the M/V Surigao Princess at the port of Cebu,
undesirable example to the employees under his the petitioner received a memorandum of even
supervision, considering that the petitioner was not date from Personnel Officer Artemio F. Aiga relative
a mere rank-and-file employee but one who owed to the irregularity in the alleged involvement in
more than the usual fealty to the organization. anomaly of ticket issuance, instructing him to
forthwith report to the main office and to explain in
On appeal to the NLRC, the latter affirmed the writing why no disciplinary action should be meted
decision of the Labor Arbiter. on him or to submit himself to an investigation. The
memorandum warned the petitioner that his failure
The Court of Appeals held that the petitioner was to comply with the aforementioned instructions
validly dismissed for serious misconduct and gross would be construed as a waiver of his right to be
habitual neglect of duties, which was aggravated heard. It also informed the petitioner of his
by his arrest for violation of Rep. Act No. 6425, as immediate preventive suspension until further
amended [the January 28, 1993 incident] and that notice. The petitioner, however, refused to
he was afforded the twin requirements of notice acknowledge receipt of the memorandum which
and hearing and the opportunity to defend himself was personally served on him, prompting the
by the investigating committee. The appellate respondent to mail the same, and which the
court noted that WNC had presented sufficient petitioner received days later.
evidence to support petitioners termination from
employment after taking into consideration the Meanwhile, upon his arrival at the office, the
totality of the infractions or the number of violations petitioner was questioned by Mr. Carlo S. Go, Senior
committed by petitioner during the period of Executive Vice-President and General Manager of
employment and stressed that it properly exercised respondent. Thereafter, petitioner was preliminarily
its management prerogative by observing due investigated by Mr. Aiga wherein his statements
process. Finally, the Court of Appeals ruled that the were taken down. After the initial investigation, the
NLRC correctly denied the claim for damages and petitioner was told to sign its minutes but he
attorneys fees for lack of evidentiary support. adamantly refused, claiming the same to be self-
incriminatory. The next day, the petitioner was
replaced by Mr. Felix Almonicar as the Chief Purser
Etcuban v. Sulpicio Lines, Inc. 448 SCRA 516 of the M/V Surigao Princess. As a result of his
(2005)
Page 66 of 5
replacement, the petitioner thought he was fired necessarily the same as those applicable to the
from his job. termination of employment of ordinary employees.
Employers, generally, are allowed a wider latitude
Labor Arbiter Ernesto F. Carreon rendered his of discretion in terminating the employment of
Decision dated March 13, 1995, finding the managerial personnel or those of similar rank
petitioners dismissal illegal. performing functions which by their nature require
the employers trust and confidence, than in the
NLRC affirmed the challenged decision with the case of ordinary rank-and-file employees.
modification that the backwages to be paid to
the petitioner shall be reckoned from the time The fact that the petitioner has worked with the
of his actual dismissal on June 10, 1994, up to respondent for more than 16 years, if it is to be
the issuance of the writ of execution on the considered at all, should be taken against him. The
finality of the decision, but not to exceed five infraction that he committed, vis-a-vis his long years
(5) years. of service with the company, reflects a regrettable
lack of loyalty. Loyalty that he should have
The Court of Appeals reversed and set aside the strengthened instead of betrayed. If an employees
NLRC decision. It ruled that there was valid and length of service is to be regarded as a justification
just cause for the petitioners dismissal, as there for moderating the penalty of dismissal, it will
was sufficient basis for loss of trust and actually become a prize for disloyalty, perverting
confidence on him. The appellate court the meaning of social justice and undermining the
amplified that in cases of dismissal for loss of efforts of labor to cleanse its ranks of all
trust and confidence, it is not required that undesirables.
there is proof beyond reasonable doubt.
The argument that the petitioner was not guilty of
ISSUE: Whether or not Vicente C. Ectuban, Jr. was anything because the tickets were never issued or
illegally dismissed. that he had received nothing from the passengers
that he could short-change the company would
RULING: NO. The petition is DENIED and the assailed not mitigate his liability, nor efface the respondents
Decision and Resolution of the Court of Appeals are loss of trust and confidence in him. Whether or not
hereby AFFIRMED in toto. the respondent was financially prejudiced is
immaterial. Also, what matters is not the amount
The requirement that there be some basis or involved, be it paltry or gargantuan; rather the
reasonable ground to believe that the employee is fraudulent scheme in which the petitioner was
responsible for the misconduct was sufficiently met involved, which constitutes a clear betrayal of trust
in the case at bar. As Chief Purser, the petitioner and confidence. In fact, there are indications that
cannot feign ignorance on the irregularity as he this fraudulent act had been done before, and
had custody of the tickets when the anomaly was probably would have continued had it not been
discovered. It would not be amiss to suppose that discovered.
the petitioner, who would benefit directly or
indirectly from the fruits of such fraudulent scheme, Moreover, the records show that the petitioner is
was a party to such irregularity. That there were not as blameless as he claimed to be. In 1979 and
other pursers who could have done the irregularity 1980, he was suspended by the respondent for
is of no moment. It bears stressing that the petitioner several company infractions, which the petitioner
was the Chief Purser who was tasked to directly did not deny. It must also be stressed that when an
supervise each and every purser under him. While, employee accepts a promotion to a managerial
indeed, it was not proved that he was the one who position or to an office requiring full trust and
made the irregular entries on the tickets, the fact confidence, he gives up some of the rigid
that he did not lift a finger at all to determine who it guaranties available to an ordinary worker.
was is a sad reflection of his job. In fact, even if the Infractions which, if committed by others, would be
petitioner had no actual and direct participation in overlooked or condoned or penalties mitigated
the alleged anomalies, his failure to detect any may be visited with more serious disciplinary action.
anomaly in the passage tickets amounts to gross It cannot be over emphasized that there is no
negligence and incompetence, which are, likewise, substitute for honesty for sensitive positions which
justifiable grounds for his dismissal. Be that as it may, call for utmost trust. Fairness dictates that the
to our mind, it is no longer necessary to prove the respondent should not be allowed to continue with
petitioners direct participation in the irregularity, for the employment of the petitioner who has
what is material is that his actuations were more breached the confidence reposed on him. Unlike
than sufficient to sow in his employer the seed of other just causes for dismissal, trust in an employee,
mistrust and loss of confidence. once lost, is difficult, if not impossible, to regain.
There can be no doubt that the petitioners
The Court was not unmindful of the foregoing continuance in the extremely sensitive fiduciary
doctrine, but after a careful scrutiny of the cited position of Chief Purser would be patently inimical
cases, the Court is convinced that the petitioners to the respondents interests. It would be oppressive
reliance thereon is misplaced. It must be stressed and unjust to order the respondent to take him
that in all of the cases cited, the employees back, for the law, in protecting the rights of the
involved were all rank-and-file or ordinary workers. employee, authorizes neither oppression nor self-
As pointed out earlier, the rules on termination of destruction of the employer.
employment, penalties for infractions, insofar as
fiduciary employees are concerned, are not
Page 67 of 5
Anent the petitioners request for separation pay,
the Court is constrained to deny the same. Well-
settled is the rule that separation pay shall be
allowed only in those instances where the
employee is validly dismissed for causes other than
serious misconduct or those reflecting on his moral
character. Inasmuch as reason for which the
petitioner was validly separated involves his
integrity, which is especially required for the position
of purser, he is not worthy of compassion as to
deserve at least separation pay for his length of
service.

Page 68 of 5

You might also like